Anda di halaman 1dari 205

DATA BANK SOAL KOBE

TO 11 FEBRUARY UGM
Ujian MCQ/ CBT Lokal Prodi Neurologi FK-KMK UGM

1. Anak laki-laki, usia 5 tahun dibawa ke IGD karena pasien tampak mengantuk terus sepanjang
hari (16 jam). Pasien adalah penderita epilepsi yang rutin kontrol di poli saraf. Pasien juga
mengalami gangguan tumbuh kembang. Menurut ibu pasien, pasien tampak mengurangi
aktivitasnya sejak 2 hari yang lalu. dan sejak tadi pagi pasien sulit dibangunkan. Riwayat
terakhir kejang 6 bulan yang lalu. pasien rutin mengkonsumsi phenobarbital dan oxcarbazepine.
Pada pemeriksaan neurologi didapatkan GCS 225, denyut jantung 70 kali/menit, frekuensi
napas 18 kali/menit dan tekanan darah 80/50 mmHg. Apakah temuan lain yang kemungkinan
besar akan ditemui pada pasien tersebut ? *
a. dilatasi pupil
b. peningkatan bising usus
c. hiperrefleksia
d. hipotermia (gejala negatif) yang lain gejala positif
e. tremor
2. Seorang laki-laki usia 53 tahun, dibawa ke IGD, dari hasil anamnesis dan pemeriksaan fisik
neurologi pasien didiagnosis Stroke Iskemik. Hasil pemeriksaan penunjang CT-scan
menunjukkan lesi di teritori a. serebri media kanan. Gejala yang bisa ditemukan pada pasien
ini adalah? *
a. Ketidakmampuan mengenali wajah orang  PCA (girus fusiform)
b. Perubahan perilaku  ACA
c. Paresis tungkai kiri  ACA motorik primer falx (kakiiiiiiiiiiiiiiiiiiiiiiiiiiiii)
d. Neglek sisi kiri (lesi non dominan hemisfer)
e. Afasia Broca  MCA kiri

3. Laki-laki, usia 30 tahun dibawa ke IGD dengan keluhan kejang seluruh tubuh menghentak-
hentak selama 40 menit. Saat di IGD pasien masih kejang. Dokter IGD segera memasukkan
injeksi diazepam intravena 10 mg. Bila kejang masih berlanjut, kapankah waktu yang tepat
untuk melakukan pengulangan injeksi diazepam intravena ? *
a. 30 – 45 detik
b. 10 – 15 menit
c. 30 – 45 menit
d. 1 – 2 jam
e. 2 – 3 jam
4. Laki-laki, usia 70 tahun dibawa ke poliklinik saraf karena mudah lupa sejak 1 tahun. Keluhan
dirasa semakin memberat. Awalnya pasien hanya lupa menaruh barang, sering mengulang

1
TO UGM 17 FEBUARI 2022
pertanyaan, namun saat ini mulai lupa dengan anaknya. Aktivitas sehari-hari sebagian besar
membutuhkan bantuan. Riwayat stroke, hipertensi, DM disangkal. Pada pemeriksaan
neurologis diperoleh MMSE 18 dan tidak ditemukan defisit neurologis fokal. Pasien
memperoleh donepezil. Bagaimanakah mekanisme kerja terapi farmakologis yang diperoleh
pasien tersebut ? *
a. Inhibitor asetilkolinesterase
b. Prekursorkolinergik 
c. Antagonis NMDA glutaminergik  memantine
d. Dopamin depleting agent  neuroleptik
e. Meningkatkan sekresi asetilkolin

5. Laki-laki, 34 tahun datang ke IGD dengan keluhan sakit kepala sejak 3 minggu. Sakit kepala
dirasakan di seluruh kepala, tidak berdenyut, seringkali memberat saat batuk. Sakit kepala
mengganggu aktivitasnya sebagai atlet tinju. Pasien jadi tidak dapat konsentrasi untuk
bertanding. Demam disangkal. Pada pemeriksaan fisik diperoleh early papiledema dan tidak
didapatkan defisit neurologis. Gambaran CT scan kepala sebagai berikut (terlampir). Apakah
diagnosis yang paling sesuai pada kasus tersebut? *

a. Post concussion syndrome : Defisit kognitif (atensi atau memori) dan ditambah minimal 3
dari berikut : fatigue, ggn tidur, sakit kepala, dizzi, irrtability, affective disturbance, apatis,
perubahan kepribadian (kepribadian berlangsung lebih dari 3 bulan)
b. primary exertional headache
c. primary thunderclap headache
d. sefalgia sekunder et causa perdarahan subdural (Bilateral kronik)
e. sefalgia sekunder et causa edema serebri

2
TO UGM 17 FEBUARI 2022
6. Laki-laki, 63 tahun didiagnosis dengan Parkinson Disease dan mendapat obat levodopa. Pada
saat kontrol pasien mengeluhkan kepalanya bergoyang-goyang sulit di kontrol. Apakah jenis
obat yang paling tepat untuk kasus tersebut ? *Dystonia
a. Levodopa
b. Dopamin Agonis
c. Amantadine
d. Antikolinergik
e. Halloperidol
Dystonia related parkinson :
- Off distonia : naikkan dosis levodopa
- Peak dose distonia : kurangi levodopa, intensifkan DA, pembedahan, antikolinergik

7. Seorang peneliti ingin mengetahui faktor-faktor risiko dari penyakit Myastenia gravis.
Dana yang diperoleh untuk penelitian tersebut tidak terlalu besar dan subyek penelitian yang
terbatas. Diharapkan penelitian tersebut selesai dalam waktu singkat. Desain penelitian
apakah yang paling cocok dipilih oleh peneliti tersebut? *
a. Potong lintang (cross-sectional)
b. Kasus kontrol
c. Kohort prospektif
d. Analisis kesintasan
e. Meta analisis

8. Seorang perempuan usia 56 tahun datang ke rumah sakit dengan keluhan nyeri kepala
mendadak tanpa disertai adanya kelemahan satu sisi tubuh 1 hari sebelum masuk rumah sakit.
Menurut pasien nyeri kepala ini dirasakan paling hebat seumur hidupnya dan muncul setelah
pasien bersin. Pasien kemudian dilakukan pemeriksaan CT-scan namun didapatkan hasil yang
normal. Kemudian dilakukan pemeriksaan lanjutan. Hasil apakah yang diharapkan didapat
pada pasien ini? *
a. Pemeriksaan Transkranial doppler didapatkan peningkatan flow velocities lebih dari 200
m/s yang menunjukkan adanya vasospasme  onset vasospasm terjadi 3–21 hari
b. Pemeriksaan MRI polos didapatkan gambaran diffusion-restriction pada sekuen DWI 
infark

3
TO UGM 17 FEBUARI 2022
c. Pemeriksaan lumbal pungsi didapatkan cairan berwarna kuning (5-6 hari post onset baru
kuning)
d. Pemeriksaan fungsi pembekuan darah didapatkan PT dan aPTT memanjang
e. Pemeriksaan rangsang meningeal didapatkan kuduk kaku positif (CT scan normal
PSA pada premesenfalik)

9. Seorang laki-laki usia 65 tahun dibawa ke UGD RS oleh keluarga dengan keluhan sering
ngompol dan berjalan sempoyongan. Tangan pasien sering gemetar. Bila berjalan, langkah
pasien lebar supaya tidak jatuh. Pada pemeriksaan didapatkan tensi : 90/55 mmHg, nadi =
70x/menit. Resting tremor (+). Apakah diagnosis yang paling mungkin pada pasien ini? *
a. Normal pressure hydrocephalus
b. Progressive supranuclear palsy
c. Vascular Parkinson
d. Multiple system atrophy (pasien ada hipotensi, inkontinensia dan parkinson  MSA
Parkinson)
e. Corticobasal degeneration

10. Seorang perempuan penyandang epilepsy katamenial datang ke poliklinik saraf. Pasien
mengeluh masih mengalami kejang pada saat menstruasi. Saat ini pasien minum fenitoin
2x100mg. Bagaimanakah penatalaksanaan yang tepat bagipasien tersebut ? *
a. Menaikkan dosis Phenitoin menjadi 2x lipat
b. Memberikan tambahan OAE Luminal 1x 60 mg 2-4 hari sebelum, selama dan setelah
menstruasi
c. Memberikan tambahan OAE Clobazam 10 mg pada saat menstruasi dan setelah menstruasi
(20-30 mg)
d. Memberikan Asetazolamid 250 mg 1-2x sehari selama 5-7 hari sebelum, selama dan
setelah menstruasi (250-500mg)
e. Memberikan Diazepam 3x 5 mg perhari, 2-5 hari sebelum, selama, dan setelah menstruasi

Epilepsi katamenial : peningkatan kejang 2x lipat saat perimenstruasi


- Diagnosis dengan pengukuran suhu tubuuh basal
- OAE kerja cepat : clobazam 20 – 30 mg / hari 10 hari selama periode mens
- Asetazolamide 250-500 mg / hari (5-7 hari sebelum dan selama mens)
- Terapi hormon : progesteron, antagonis estrogen, metabolit progest

11. Seorang ibu membawa hasil rekaman EEG (terlampir) anak laki-lakinya yang berusia 15 tahun
dengan keluhan sering bengong. Apakah jenis terapi OAE yang dihindari pada kasus
tersebut ? * JAE – Elava : etox, lamo , valroat
JME : lalevato

4
TO UGM 17 FEBUARI 2022
a. Carbamazepine / dan fenitoin /clonazepam (memperburuk) CCF
b. Fenitoin
c. Oxcarbazepine
d. Asam valproate
e. Phenobarbital

12. Perempuan 40 tahun dirawat di bangsal saraf dengan SAH spontan. Pasien memperoleh
manitol, nimodipin dan valsartan. Pada visit hari ini tampak perburukan kesadaran. Pada
pemeriksaan fisik terdapat diaforesis dan turgor menurun. Status neurologis terdapat
penurunan GCS 2 poin dan tidak ditemukan defisit neurologis baru. Funduskopi normal.
laboratorium sbb (terlampir). Apakah penyebab penurunan kesadaran yang paling mungkin
pada pasien ini? *

CSWS

a. Hiponatremi ec cerebral salt wasting (CSWS)


b. Hiponatremi ec syndrome of inappropriate antidiuretic hormone
c. Cerebral pontine myelinolisis
d. Pseudohiponatremia
e. Hiponatremi ec infus manitol
PSA CSWS
Meningitis  SIADH

5
TO UGM 17 FEBUARI 2022
13. Perempuan, usia 13 tahun dilakukan perekaman EEG. Pada saat dilakukan hiperventilasi
didapatkan gelombang yang lambat dengan amplitude tinggi hampir di seluruh lapangan
EEG. Pada saat perekaman berlangsung 1 menit setelah HV dihentikan, ternyata gelombang
lambat tersebut masih ada. Apakah penyebab yang paling mungkin terhadap munculnya
gelombang lambat pada kasus tersebut ? *
a. Hipoksia
b. Hipoglikemi  pada kasus
c. Hipotermi
d. Hipokapnea : ini utk perlambatan scr umum terjadi pada hiperventilasi
e. Hipoksemia
Hiperventilasi memicu absan , carbondiaksoda menumpuk pada neuron

14. Perempuan, 26 tahun, penyandang epilepsi. Saat ini pasien sedang hamil. Oleh dokter yang
merawat pasien diberitahui bahwa dosis OAE yang diminum akan dinaikkan dosisnya. Dasar
pemikiran dokter menaikkan dosis karena pada saat hamil kemungkinan akan terjadi
peningkatan frekuensi bangkitan. Apakah yang sangat mungkin menyebabkan peningkatan
frekuensi bangkitan pada kondisi pasien tersebut ? *
a. Perubahan hormonal  Trimester III (estrogen tinggi)
b. Penurunan konsentrasi albumin : albumin sbg alat transportasi obat2an
c. Peningkatan berat badan
d. Belum terkontrolnya bangkitan sebelum kehamilan
e. Retensi cairan tubuh

15. Anak perempuan, 15 tahun dibawa oleh orang tuanya ke poliklinik saraf dengan keluhan tiba-
tiba ada gerakan menyentak pada lengan dan bahu. Keluhan terutama pada pagi hari
setelah bangun tidur. Saat usia 8 tahun pasien pernah didiagnosis bangkitan lena.
Bagaimanakah gambaran EEG yang diharapkan muncul pada pasien tersebut ? *JME : biasanya
ada riwayat absans (Absans juga sebaliknya ada riwayat myoclonic)
a. 2,5Hz SWC : LGS (atipikal absans)
b. 3Hz SWC : tipikal absan
c. Polispike general
d. Polispike fokal : TLE FLE
e. Spike centrotemporal : BECTS/rolandic : hemifasial spasm, orolaringofaringeal

16. Perempuan, 65 tahun, SMA,ibu RT, paska stroke infark datang untuk kontrol ke poli Saraf.
Hasil Evaluasi Neurologis T: 140/90 mmHg. N: 80x/mnt, R: 18x/mnt regular. Moca-Ina 25,
Murung, mudah tersinggung, depresi + , Motivasi rendah, isolasi sosial, pikiran negatif
meningkat, insomnia. Dimanakah letak lesi kasus ini ? *Demensia vaskular
a. Ganglia basalis
b. Girus Cingulate
c. Sistim limbik Dalam
d. Lobus temporal
e. Prefrontal bilateral

6
TO UGM 17 FEBUARI 2022
17. Seorang perempuan usia 56 tahun datang ke rumah sakit dengan keluhan nyeri kepala
mendadak tanpa disertai adanya kelemahan satu sisi tubuh 1 hari sebelum masuk rumah sakit.
Menurut pasien nyeri kepala ini dirasakan paling hebat seumur hidupnya dan muncul
setelah pasien bersin. Pasien kemudian dilakukan pemeriksaan CT-scan namun didapatkan
hasil yang normal. Pemeriksaan penunjang paling tepat apakah yang berikutnya dapat
dilakukan untuk menegakkan diagnosa pasien ini? *
a. Pemeriksaan Transcranial doppler
b. Pemeriksaan MRI polos
c. Pemeriksaan Lumbal pungsi
d. Pemeriksaan fungsi pembekuan darah
e. Pemeriksaan rangsang meningeal

18. Laki-laki, usia 58 tahun, datang ke RS dengan keluhan gerakan gemetar pada tangan sejak 3
tahun yang lalu. Pasien didiagnosa dokter dengan Parkinson Disease. Dalam beberapa bulan
ini pasien mengeluhkan gejala penyakit Parkinson nya semakin bertambah kemudian pasien
mengalami Freezing atau hesitation. Pilihan terapi apakah yang tepat kita berikan pada pasien
tersebut ? * ON OF fenomenom
a. Antikolinergik
b. Levodopaman
c. Dopamin Agonis
d. Propanolol
e. Selegiline

19. Perempuan 70 tahun dirujuk ke klinik memori, dengan keluhan pelupa sejak 2 tahun terakhir
dan makin memberat, sering bingung, dan tersesat, penurunan fungsi intelektual dan aktivitas
harian perlu bantuan. Apakah Standar pemeriksaan untuk penegakan diagnosis kasus ini ? *
a. Autoanamnesis, esessmen neuropsikologi, neuroimaging
b. Allo Anamnesis, pemeriksaan Fisik, neuropsikologi dan biomarker (Biomarker tambahan)
c. Allo anamnesis, pemeriksaan neuropsikologi dan neuroimaging
d. Pemeriksaan fisik, neuropsikologi, laboratorium, neuroimaging
e. Anamnesis, pemeriksaan fisik, MMSE , laboratorium dan Head CT-Scan

7
TO UGM 17 FEBUARI 2022
20. Laki-laki berusia 63 tahun, bekerja sebagai seorang dosen perguruan tinggi, dibawa ke ahli
saraf setelah dia mengalami kesulitan membaca materi kuliah saat mengajar. Pada
pemeriksaan, pasien bisa berbicara normal. Dia bisa membaca huruf tapi tidak seluruh
kata. Dia bisa menulis kalimat lengkap saat didikte tetapi kemudian tidak dapat membaca
apa yang telah ditulisnya. Pemahaman, pengulangan, penulisan, dan penamaan adalah juga
normal. Ketajaman visual adalah normal. Saat uji konfrontasi, didapatkan pasien memiliki
hemianopia homonim. Pemeriksaan saraf kranial, motorik, dan sensorik normal. MRI-nya
sesuai gambar dibawah (terlampir). Gejala pasien ini sesuai dengan? *

a. Afasia wernike
b. Disleksia psikogenik
c. Aleksia tanpa agrafia (calcarina sama corpus callosum)
d. Sindrom neglek selektif
e. Buta kortikal

21. Perempuan 40 tahun, dibawa ke IGD dengan keluhan penurunan kesadaran sejak 3 hari yang
lalu semakin memberat. Pasien merupakan penderita cirosis hepatis kronis. Hasil
pemeriksaan laboratorium SGOT 450, SGPT 675. pasien diusulkan dilakukan pemeriksaan
EEG. Bagaimanakah hasil EEG yang diharapkan muncul pada kasus tersebut ? *Ensefalopati
a. Continous slow activity
b. Triphasic wave

8
TO UGM 17 FEBUARI 2022
c. BIPLEDS
d. Intermittent slow activity
e. BECTs : Rolandic
PLEDS : ensefalitis

22. Seorang laki-laki usia 56 tahun datang ke tempat praktek RMC anda dengan keluhan rasa
nyeri seperti terbakar pada kedua ujung tangan dan kaki sejak 1 tahun yang lalu. Keluhan
disertai dengan rasa kesemutan dan rasa seperti tersengat listrik. Pasien memiliki riwayat
diabetes melitus tipe II sejak 10 tahun yang lalu dan tidak rutin mengkonsumsi obat
antidiabetik oral . Pada pemeriksaan fisik didapatkan Tekanan darah 120/70, nadi 83 kali/menit,
laju nafas 20 kali/menit, suhu badan 36.3. Pemeriksaan neurologis didapatkan gloves and
stoking hipestesia, pemeriksaan propioseptif dalam batas normal. Pada pemeriksaan
penunjang didapatkan HbA1C 16,5 g/dl. Tatalaksana yang paling tepat diberikan pada pasien
adalah? *
a. Pregabalin 1x75 mg
b. Amitriptillin (lini 2)
c. Okskarbamazepine
d. Tramadol
e. Gabapentin

Diabetic neuropati : pregabalin, gabapentin, karbamasepin, okskarbasepin

23. Perempuan, 45 tahun, datang ke poli saraf dengan keluhan tangan kanan bergerak-gerak sendiri
tanpa dapat dikontrol. Keluhan tersebut timbul mendadak 1 minggu lalu dan menetap hingga
saat ini. Pada observasi tampak tangan bergerak-gerak tanpa terkendali, kasar, seperti
terlempar-lempar, terutama melibatkan bagian ekstremitas proksimal. Gangguan pada
struktur manakah yang mendasari patofisiologi terjadinya kasus tersebut? *balismus
(proximal)
a. Nukleus kaudatus kiri  korea
b. Nukleus subtalamikus kiri
c. Substansia nigra kiri  parkinson
d. Putamen kiri  distonia
e. Globus palidus kiri

24. Perempuan, usia 55 tahun dibawa anaknya ke IGD RS dengan keluhan lemah lengan dan
tungkai kanan yang dialaminya secara tiba-tiba saat bangun tidur di pagi harinya. Sejak 5
tahun terakhir pasien menderita diabetes mellitus dan tidak teratur minum obat. Pada
pemeriksaan dijumpai kesadaran compos mentis, tensi 190/100 mmHg, nadi 86x/menit,
pernafasan 20x/menit dan suhu 36,9o C. Sudut mulut tertarik ke sisi kiri, tetapi masih bisa

9
TO UGM 17 FEBUARI 2022
mengerutkan kening dan menutup mata. Dijumpai hemineglect kiri. Dimanakah letak
kelainan pada kasus tersebut ? *
a. Arteri serebri anterior
b. Arteri serebri media
c. Arteri serebri posterior
d. Arteri basilaris
e. Arteri serebelli anterior inferior

25. Seorang anak usia 13 th, gemuk, datang ke IGD karena nyeri kepala disertai muntah. Nyeri
kepala dirasakan setiap hari seperti ditekan, terkadang disertai pandangan kabur. Pemeriksaan
neurologi : papil edema 2 dioptri. Pemeriksaan penunjang apakah yang juga bisa digunakan
sebagai terapi ? *IIH idiopatik intrakranial HT
a. Pemeriksaan neuroimaging
b. Pemeriksaan darah lengkap
c. Pemeriksaan goniometri
d. Pemeriksaan lapang pandang
e. Pemeriksaan liquor cerebro spinal

26. Laki-laki, 25 tahun datang ke poliklinik saraf dengan keluhan rasa panas pada punggung
tangan kanan sejak 1,5 bulan. Keluhan dirasakan seperti rasa terbakar dan terjadi terus
menerus. Pada saat disentuh dirasakan perih sehingga sangat mengganggu. Beberapa hari
sebelumnya pasien mengalami avulsi saraf radialis distal kanan. Pada pemeriksaan fisik
tampak kulit kering, merah, edema dan ditumbuhi rambut. Apakah diagnosis yang paling tepat
untuk pasien tersebut ? *
a. Nyeri fantom (phantom limb pain)
b. Cervical root syndrome
c. Complex Regional Pain Syndrome (CRPS) tipe 1
d. Nyeri Terowongan Radial (Radial Tunnel Syndrome)
e. Kausalgia CRPS 2 jawaban CW

27. Seorang laki-laki 54 tahun dengan klinis recurent TIA. Pasien menjalani pemeriksaan TCD.
Salah satu protokol dilakukan pemeriksaan transforaminal. Pembuluh darah apa saja yang bisa
dilihat dari protokol tersebut? *
a. A Opthalmika dan Siphon (transorbital)
b. ACA, MCA dan PCA (trans temporal)
c. A. Vertebral dan Bassilar (transforaminal)
d. A. Vertebral dan PCA
e. MCA dan PCA

10
TO UGM 17 FEBUARI 2022
28. Seorang anak 9 tahun dibawa oleh orang tuanya karena gangguan berjalan. pada pemeriksaan
didapatkan gangguan keseimbangan disertai ketidakmampuan mengukur jarak. Dari
struktur intracranial berikut, manakah yang paling mungkin terkena pada anak ini? *
a. Ganglia basalis
b. Thalamus
c. Medulla oblongata
d. Lobus parietalis
e. Cerebellum

29. Laki-laki 52 tahun datang ke klinik dengan keluhan tremor. Tremor disadari istri beberapa
tahun terakhir, terutama saat istirahat misalnya saat menonton TV. Dia juga mengeluh lebih
lambat berjalan, dan istrinya mengatakan bahwa suaranya lebih lembut. Pada
autoanamnesis, tulisannya juga semakin mengecil. Selain itu dia merasa sehat dan tidak ada
keluhan kognitif. Pada pemeriksaan didapatkan tremor istirahat sedang di tangan kanan dengan
cogwheel ringan pada tangan kanan. Pada pemeriksaan gait didapatkan postur sedikit kaku
dan dia lebih mengayunkan tangan kanan daripada kiri. Pada ketukan jari lebih lambat pada jari
kanan. Pemeriksaan lain dalam batas normal. Diagnosis yang paling tepat? *
a. Idiopathic Parkinson’s disease
b. DLB
c. Vascular parkinsonism
d. Drug-induced parkinsonism
e. Corticobasal ganglionic degeneration

30. Seorang laki-laki umur 20 tahun datang ke Poliklinik Saraf RSUP dengan keluhan kelemahan
telapak kaki kiri sulit diangkat sejak 3 minggu y.l. pasca terpeleset. Dari status neurologis
didapatkan kelemahan M. Tibialis Anterior, M. Peroneus Longus, dan M. Ekstensor
Digitorum Brevis sinistra, hipestesi di daerah lateral tungkai kiri bawah dan dorsum pedis
sinistra. Apakah hasil pemeriksaan NCV – EMG yang diharapkan? *
a. CMAP N. Tibialis abnormal
b. CMAP N. Peroneus Profundus dan Superfisialis abnormal (motorik)
c. SNAP N. Suralis abnormal
d. EMG: terdapat aktivitas spontan pada M. Biceps Femoris
e. EMG : terdapat aktivitas spontan pada M. Gastrocnemius

31. Seorang laki-laki umur 24 tahun datang ke Poliklinik Saraf RSUP dengan keluhan kelemahan
telapak tangan kanan sejak 1 bulan y.l. pasca terjatuh. Dari status neurologis didapatkan
kelemahan M. Brachioradialis dan M. Ekstensor Digitorum Communis dextra, RF
Brachioradialis menurun, hipestesi di daerah lateral dorsum manus dextra. Dimanakah
kemungkinan letak lesi pada kasus tersebut? *
a. Spiral Groove
b. N. Interosseus Posterior  motor murni
c. Aksila

11
TO UGM 17 FEBUARI 2022
d. Elbow
e. Pleksus brachialis

32. Seorang laki-laki umur 20 tahun datang ke Poliklinik Saraf RSUP dengan keluhan kelemahan
telapak kaki kiri sulit diangkat sejak 3 minggu y.l. pasca terpeleset. Dari status neurologis
didapatkan kelemahan M. Tibialis Anterior, M. Peroneus Longus, dan M. Ekstensor Digitorum
Brevis sinistra, hipestesi di daerah lateral tungkai kiri bawah dan dorsum pedis sinistra.
Dimanakah kemungkinan letak lesi pada kasus tersebut? *
a. Radiks L4-5, L5-S1
b. N. Ischiadicus
c. N. Peroneus Communis
d. N. Tibialis
e. N. Femoralis

33. Seorang pria, 43 tahun dibawa ke rumah sakit dengan keluhan kejang. Bentuk kejang umum.
Keluhan dirasakan sejak 4 bulan yang lalu. Pasien tinggal di daerah di mana babi dipelihara di
pekarangan rumah. Pada CT-scan didapatkan gambaran lesi kistik multiple dengan kalsifikasi
multiple. Pada beberapa lesi kistik didapatkan gambaran serupa scolex. Manakah obat yang
paling cocok diberikan pada pasien ini? *
a. Amfoterisin B
b. Albendazol (1 1nya obat cacing) 15mg/kg bb utk 1 bulan; bisa prazikuantel
50mg/kgbb; 2 minggu
c. Flukonazol
d. Metronidazol
e. Pirimetamin

34. Laki-laki 35 tahun datang ke tempat praktek dengan riwayat tergigit anjing di daerah endemis
rabies, saat ini anjing yang mengigit masih hidup status General :Tekanan Darah : 130/70
mmHg HR: 107 x/’. RR: 24 x/menit. Tax : 36C.Kepala, thorak, abdomen dalam batas normal .
Extremitas : luka gigitan di tangan kiri menembus epidermis. Status neurologi GCS 456 MS ; -,
Motorik : lateralisasi –Reflek patologis : -/- Reflek fisiologis : normal. Lain-lain dalam batas
normal. Apa tindakan untuk memberi tatalaksana terbaik? *
a. Rawat luka, SAR Human imunoglobullin 40 IU/kgBB atau Equine Rabies
Immunoglobulin adalah 20 IU/kgBB dan vaksin antirabies 4 kali
b. Rawat luka, SAR Human imunoglobullin 20 IU/kgBB atau Equine Rabies
Immunoglobulin 40 IU/kgBB dan vaksin anti rabies 4 kali
c. Rawat luka, SAR Human imunoglobullin 20 IU/kgBB atau Equine Rabies
Immunoglobulin 40 IU/kgBB dan vaksin anti rabies 2 kali
d. Rawat luka, SAR Human imunoglobullin 40 IU/kgBB atau Equine Rabies
Immunoglobulin adalah 20 IU/kgBB dan vaksin antirabies 2 kali
e. Rawat luka, SAR Human imunoglobullin 50 IU/kgBB atau Equine Rabies
Immunoglobulin adalah 20 IU/kgBB dan vaksin antirabies 2 kali

12
TO UGM 17 FEBUARI 2022
35. Seorang wanita 45 tahun dibawa ke IGD karena tiba-tiba mengeluh nyeri kepala disertai
kelemahan lengan dan tungkai kanan. Riwayat hipertensi kronis. CT Scan dibawah ini. Arteri
yang terlibat pada kasus tersebut adalah? *

a. Arteri. Lentikulostriata (capsula interna)


b. Arteri Pons perforator
c. Arteri serebelaris superior
d. Arteri khoroidalis posterior
e. Arteri serebri media segmen kortikal

36. Seorang perempuan 80 tahun datang ke poli saraf dengan keluhan gangguan ingatan. Pada
anamnesis ditemukan status fungsional penderita yang menurun secara drastis. Keluhan ini
berlangsung kurang lebih 1 tahun. Gejala awal adalah penderita sering lupa, gampang menangis, dan
sering melihat objek dan mendengar suara-suara yang sebenarnya tidak ada. Riwayat penyakit
dahulu disangkal. Pada pemeriksaan fisik umum dalam batas normal. Pemeriksaan skrining MMSE
adalah 19.Terapi yang dapat diberikan pada kasus ini adalah Donepezil maupun Memantine.
Bagaimana cara kerja dari Memantine? *
a. Merupakan golongan inhibitor asetilkolinesterase yang berguna untuk mengatasi cedera
neuronal karena defisit kolinergik.
b. Merupakan antagonis NMDA nonkompetitif yang berguna untuk mengurangi defisit
kolinergik.
c. Merupakan golongan inhibitor asetilkolinesterase yang berguna untuk mengurangi cedera
neuronal karena toksisitas glutamat.
d. Merupakan antagonis NMDA nonkompetitif yang berguna mengurangi cedera neuronal
karena toksisitas glutamat.
e. Merupakan golongan inhibitor asetilkolinesterase yang berguna untuk mengurangi defisit
kolinergik.
37. Seorang perempuan umur 40 tahun datang ke Poliklinik Saraf RSUP dengan keluhan nyeri seperti
ditusuk-tusuk jarum pada jari 4 dan 5 tangan kanan(ulnaris). Keluhan ini seringkali terasa bila
bangun pagi. Terdapat kelemahan tangan kanan sehingga pasien sulit melebarkan jari-jari tangan
(wartenberg) dan menjepit dengan jempol (froment). Terdapat gangguan sensibilitas pada sisi palmar
jari 4 dan 5. Apakah diagnosis dari pasien tersebut? *
a. Cubital tunnel syndrome : sensorik palmar dan dosum manus terganggu

13
TO UGM 17 FEBUARI 2022
b. Carpal tunnel syndrome
c. Ulnar tunnel syndrome
d. Guyon canal syndrome (pergelangan tangan) : sensorik palmar saja yg terganggu
e. Saturday night palsy

38. Seorang wanita usia 32 tahun datang ke Emergensi dengan keluhan gangguan ketajaman
penglihatan dan rasa nyeri pada mata kanan. Pada pemeriksaaan didapatkan penurunan visus pada
mata kanan yang tidak dapat dikoreksi dengan penggunaan pinhole. Didapatkan relative afferent
pupillary defect (RAPD) pada mata kanan. Di manakah letak kelainan yang paling mungkin? *
a. Chiasma opticum
b. Nervus opticus (aferen eferen RAPD)
c. Tractus opticus
d. Radiatio optica
e. Korteks oksipital
39. Perempuan, 37 tahun mengalami penurunan kesadaran paska KLL. Saat kecelakaan terjadi pasien
baik-baik saja. 2 jam kemudian pasien mendadak mengalami penurunan kesadaran. Pemeriksaan
fisik = midriasis mata kanan dan refleks cahaya mata kanan (-). Tipe herNiasi yang manakah yang
terjadi pada pasien di atas ? *
a. Tonsil cerebellar masuk ke dalam foramen magnum (tonsilar)
b. Cerebellum upward melalui tentorium (transtentorial)
c. Girus cingulate ke bawah falx (subfalcin)
d. Medulla masuk ke foramen magnum (tonsilar)
e. Lobus temporal masuk ke tentorium (uncal)

40. Seorang wanita 28 tahun dibawa ke UGD oleh suaminya karena keluhan panas badan dan nyeri
kepala. Panas badan dirasakan sejak 2 hari yang lalu. Pada pemeriksaan fisik didapatkan adanya kaku
kuduk, pemeriksaan fisik neurologi lain tidak menunjukkan kelainan. Hasil pemeriksaan LP
menunjukkan jumlah sel 1.900 dengan predominansi PMN (khas bakterial/khas), protein 83 mg/dL(0-
50), dan glukosa 25 mg/dL(40-80). Glukosa darah 110 mg/dL. Apakah diagnosis yang paling mungkin
pada penderita ini? *
a. Meningitis bakterialis akut
b. Meningitis viral
c. Meningitis jamur
d. Meningitis TB
e. Meningitis eosinofilik

41. Seorang laki-laki 57 tahun mendadak mengalami kelemahan lengan dan tungkai sebelah kiri sejak 1
jam yang lalu. Tensi 200/110 mmHg dengan NIHSS 12. Pada pemeriksaan laborat hasil faal hemostasis
trombosit 120.000. Hasil CT Scan didapatkan perdarahan minimal daerah kapsula interna kanan.
Riwayat cedera kepala tidak serius 2 bulan lalu(3 bulan). Apakah Kontraindikasi untuk dilakukan
trombolisis intravena pada kasus tersebut ? *
a. Tensi 200 mmHg
b. NIHSS hanya 12

14
TO UGM 17 FEBUARI 2022
c. Cedera kepala 2 bulan sebelumnya
d. CT Scan adanya perdarahan minmal
e. Trombosit relatif kurang
42. Seorang laki-laki 45 tahun dibawa ke IRD RS karena mendadak mengeluh nyeri kepala hebat disertai
muntah. Pemeriksaan fisik didapatkan tanda Brudzinki I dan II (+). Apakah sistem skoring yang tepat
Untuk menilai derajat keparahan pada kasus tersebut ? *
a. Skor Hunt – Hess
b. Skor Siriraj (stroke perdarahan /iskemik)
c. Skor Splitz Martin (AVM)
d. Skor Fisher (menilai hasil CT scan)
e. Skor Cognard  DAVF dan Borden classification

43. Seorang wanita 60 tahun sering mengeluh pusing dengan riwayat hipertensi terkontrol Pada
pemeriksaan fisik ditemukan gangguan koordinasi dan pemeriksaan sonologi dan angiografi ditemukan
stenosis pada arteri basilaris. Apakah Jalur kolateral dari sistem ekstrakranial yang berperan pada
untuk menjaga stabilitas CBF pada kortikal posterior kasus tersebut ? *
a. Arteri maksilaris (bagian interna)  menjadi arteri temporalis superrfisal
b. Arteri oksipital
c. Arteri temporalis superfisial  anastomosis dgn arteri meningea media
d. Arteri fasialis  arteri angulariss  anastomosis dgn oftalmika
e. Arteri asending faringeal

44. Sorang ibu usia 31 th mengeluh anak lelakinya yang berusia 7 th mengalami kesulitan menaiki tangga
sejak 1 tahun yang lalu. Sejak usia 3 th, anak tersebut ketika berdiri dari duduk selalu terlihat tangannya
bertumpu pada benda di sekitarnya (gower sign). Setelah ditelusuri, ternyata paman ibu tersebut juga
mengalami hal serupa pada saat kecil, dan meninggal ketika berusia 18 tahun. Pada pemeriksaan needle
EMG anak tersebut, gelombang apakah yang sangat diharapkan muncul ? *DMD
a. Positive sharp wave.  neuropatik
b. Polyphasic.  bisa dua dua
c. Fibrilasi.  neuropatik
d. Amplitudo rendah
e. Myokimic discharge.  penyakit Miotonia

15
TO UGM 17 FEBUARI 2022
45. Seorang laki-laki usia 20 th mengeluh lengan kiri tidak bisa digerakkan dengan sempurna sejak 2 bulan
yang lalu . Pada pemeriksaan didapatkan abnormalitas pada abduksi lengan atas (deltoid :
N.aksilaris), fleksi pada siku (Biceps : N.musculocutaneus), dan supinasi lengan bawah .
Pemeriksaan reflek trisep masih normal. Di manakah kemungkinan besar letak lesi yang dialami
pasien tersebut ? *
a. Upper trunk pleksus brachialis (ERB palsy)/waiter tip) (C5,6)
b. Middle trunk pleksus brachialis.  C7  radial dan median
c. Lower trunk pleksus brachialis.  kelainan n.ulnaris (C8,T1)
d. Posterior cord pleksus brachialis  kelainan aksila dan radialis
e. Medial cord pleksus brachialis.  kelainan n.ulnaris

46. Seorang pria 40 tahun kontrol ke poliklinik setelah perawatan karena kecelakaan lalu lintas yang
menyebabkan trauma kepala. Pada pemeriksaan gangguan saraf VII dan VIII (meatus akustikus
internus/MAI) kiri yang menetap. Di antara struktur tengkorak berikut, manakah yang mungkin
mengalami kerusakan akibat kecelakaan tersebut? *
a. Lamina cribrosa  anterior NI
b. Canalis opticus  anterior NII
c. Fissura orbitalis superior : Nervus 346 51
d. Meatus acusticus interna  NVII, VIII
e. Foramen jugularis  N IX,X,XI

47. Laki-laki berusia 30 tahun datang dengan keluhan nyeri kepala yang memberat sejak 1 bulan
terakhir. Nyeri terasa di seluruh kepala, tidak berdenyut, tidak menjalar, memberat dengan
aktivitas dan membaik saat tidur. Keluhan mual dan muntah proyektil disangkal. Pemeriksaan fisik
awal didapatkan tekanan darah 120/70mmHg, nadi 82x/menit, nafas 20x/menit, suhu badan 36.7C,
saturasi oksigen 99%. Pemeriksaan neurologis didapatkan GCS 15, pupil isokor, slight paresis N. VII
UMN dextra, dan status motorik dan sensorik dalam batas normal. Riwayat hipertensi, diabetes,
infeksi paru disangkal. Pasien bekerja sebagai peternak babi. Dilakukan pemeriksaan CT scan kontras
pada pasien ini dan didapatkan gambaran dinding kista yang tampak menebal dengan
penyangatan yang samar, serta edema perilesional yang minimal. Kemungkinan besar pasien
sedang berada dalam tahap penyakit? *
a. Vesicular stage : larva (cyst with dot sign : enhance sedikit, tanpa edema)
b. Colloidal stage : Gambaran kista  degenerasi sistiserkus awal, kerusakan skoleks (respon
inflamasi  kista ring enhancement disekitarnaya)
c. Granular stage : degenerasi sistiserkus yg mengecil (enhancement nodular dengan edema
ringan)
d. Nodular calcified stage : tanda kalsifikasi (hiperdens)
e. Residual stage

16
TO UGM 17 FEBUARI 2022
48. Laki-laki 62 tahun datang dengan nyeri pada lengan, dan terasa kaku dan disertai tremor saat
istirahat. Pasien mengeluh sulit berjalan tapi tidak jatuh. Pada pemeriksaan didapatkan distonia di
lengan kanan dengan postur fleksi di siku dan pergelangan tangan. Kadang-kadang didapatkan
hentakan seluruh badan. Sensasi terhadap sentuhan, nyeri, dan modalitas sensorik lain normal namun
dengan matanya tertutup, tidak bisa mengidentifikasi benda yang diletakkan di tangan kanan atau
nomor yang digambarkan di telapaknya (astereognosia dan agrefestesia). Diagnosis yang paling
tepat? *
a. Penyakit Parkinson Idiopatik
b. Multiple-system atrophy
c. Progressive supranuclear palsy
d. Normal-pressure hydrocephalus
e. Corticobasal ganglionic degeneration : asimetris, apraksia, mioklonus dan tanda
piramidal, ggn sensorik

49. Seorang pria 78 tahun ini datang ke kantor, dibawa oleh personil rumah jompo karena agitasi yang
semakin memburuk dan gangguan neurologis lainnya. Atas dasar temuan dan riwayat yang ada, Anda
mendiagnosa DLB. Tiga temuan klinis utama di DLB adalah? *
a. Parkinsonism, gangguan memory, halusinasi visual
b. Halusinasi Visual, aphasia, parkinsonism
c. Fluctuating fungsi cognitif, parkinsonism, agresi
d. Parkinsonism, fluctuating fungsi cognitif, Halusinasi visual
e. Halusinasi Visual, Halusinasi auditory, fluctuating fungsi cognitif

50. Seorang laki-laki umur 24 tahun datang ke Poliklinik Saraf RSUP dengan keluhan kelemahan telapak
tangan kanan sejak 1 bulan y.l. pasca terjatuh. Dari status neurologis didapatkan kelemahan M.
Brachioradialis dan M. Ekstensor Digitorum Communis dextra, RF Brachioradialis menurun,
hipestesi di daerah lateral dorsum manus dextra. Apakah hasil pemeriksaan status neurologis lain
yang diharapkan? *
a. Ditemukan kelemahan M. Deltoid
b. Ditemukan kelemahan M. Biceps

17
TO UGM 17 FEBUARI 2022
c. Ditemukan kelemahan M. Triceps  entrapment setinggi axilla
d. RF Biceps menurun
e. RF Triceps normal

51. Perempuan, 72 tahun diperiksa CT-scan kepala karena sakit kepala. Didapatkan hasil masa di hemisfer
kiri dengan hiperostosis diatas tulang tengkorak. Apakah diagnosis yang paling mungkin pada kasus
tersebut ? *
a. Meningioma
b. Pituitari adenoma
c. Astrocytoma
d. Schwannoma
e. Hemangioblastoma

52. Laki-laki 30 tahun mengeluh nyeri dan lemah pada lengan kanannya. Pasien baru saja mendapat tetanus
antitoksin karena luka tusuk 6 hari yang lalu. Apakah penyebab munculnya keluhan kelemahan dan
nyeri pada pasien tersebut ? *
a. Tetanus
b. Septicemia
c. Botulism
d. Neuritis brachial  ada kasus neuritis post ATS
e. Bukan akibat ATS

53. Seorang laki-laki 36 tahun dengan penurunan kesadaran yang terjadi perlahan. Riwayat nyeri
kepala kronik progresif dialami sejak 2 tahun yang lalu. Pemeriksaan fisik neurologi menunjukkan
hemiparesis kiri. Pemeriksaan imejing menunjukkan gambaran tumor primer intrakranial.

18
TO UGM 17 FEBUARI 2022
Pemeriksaan histopatologi otak menunjukkan sel yang atipik tanpa adanya nekrosis dan
proliferasi endotel. Gambaran histopatologi yang sesuai dengan klasifikasi WHO adalah? *
a. pilocytic astrositoma (WHO Grade I)
b. fibrillary astrositoma (WHO Grade II)  mitosis rendah, perjalanan lambat 5-8 thn
c. anaplastic astrositoma (WHO Grade III)  mitosis tinggi
d. glioblastoma Multiforme (WHO Grade IV)  ada nekrosis dan invasi vaskular
e. pleomorphic xanthoastrocytoma (WHO Grade II)

- Derajat I : lesi tumor berbatas tegas, contohnya astrositoma pilositik


- Derajat II (astrositoma difusa) : tumor astrositik tampak infiltratif dan disertai atipia sitologis
- Derajat III (astrositoma anaplastik) : tumor mulai menunjukan anaplasia dan aktivitas mitosis
- Derajat IV (glioblastoma) : menunjukkan adanya proliferasi mikrovaskular & atau nekrosis

54. Perempuan, 57 tahun, datang ke IGD dengan keluhan kelemahan kedua tungkai sejak 1 tahun yang
lalu. pemeriksaan fisik = tonus spastik, fasikulasi (+ LMN), respon plantar ekstensor (+) dan
hiperreflexia. Dimanakah letak kelainan yang paling mungkin pada kasus tersebut ? *ALS
a. Dorsal spinal root  ggn sensorik dermatomal
b. Ventral spinal root  LMN
c. Fasiculus arcuatus  afasia konduksi
d. Motor neuron
e. Purkinje sel

55. Wanita umur 36 tahun datang ke poliklinik saraf membawa surat pengantar dari dokter spesialis
mata dengan keluhan mata kanan kabur mendadak sejak 4 hari yang lalu. Dibagian mata tak
dijumpai kelainan. Visus tak terhingga, tak bisa dikoreksi. Jika dilakukan pemeriksaan Visual
evoked potensial hasil apakah yang akan didapatkan ? *
a. Latensi P100 normal., amplitude menurun  ggn okular (VEP monookuler)
b. latensi P100 memanjang dengan amplitude menurn. : ggn okular (VEP
Kata mas dion ga ada
monookuler)
c. Latensi N.145 normal, amplitude menurun : ggn okular (VEP monookuler) jawabanya
d. Latensi P100 dan N145 normal. : normal
e. Latensi N75 memanjang.
f. Siapakah mas dion???

Amplitudo menurun :
- 1 mata : ggn okular
- 2 mata : bisa banyak lesi
Latensi memanjang :
- Lesi nervus optik
Absence VEP : neuropati optik / ggn okular
N75 : korpus genikulatum lateral sampai radiasio optik

19
TO UGM 17 FEBUARI 2022
P100 : Radiasio optik sampai visual korteks
N145 : Visual korteks ke kortek asosiasi visual

56. Seorang pasien wanita usia 34 dikonsulkan ke bagian penyakit saraf dengan keluhan nyeri kepala
progresif sejak 6 bulan yang lalu, paralisis gerakan konjugat ke atas kedua mata, nistagmus
konvergensi, gangguan reflek akomodasi dan retraksi eyelid (collier lid). Hasil patologi
anatomi didapatkan kesan pinealoma. Kumpulan gejala dari penyakit di atas dikenal dengan? *
a. Sindroma weber
b. Sindroma benedict
c. Sindroma parinaud
d. Sindroma dejerine
e. Sindroma Wallenberg

57. Seorang anak usia 5 tahun dibawa oleh ibunya ke poli anak dengan keluhan anaknya sulit berjalan,
jika berjalan pasien jinjit dan tidak bisa jongkok, dan kesulitan untuk berdiri sendiri dari posisi
duduk. Pasien dibawa ke RS, dari pemeriksaan fisik didapatkan psudohipertrofi dan gower sign.
Pasien juga dilakukan pemeriksaan gen, dan didiagnosa dengan Duchene Muscular Distrofi, apa
hasil yang didapatkan dari pemeriksaan gen tersebut? *
a. Mutasi Dystrophin
b. Mutasi gen sitokrom p450
c. Mutasi gen SNM 2
d. Mutasi gen TRVP4
e. LGMD type 2A-2S

58. Seorang laki-laki 56 tahun datang dengan keluhan kelemahan keempat anggota gerak sejak 5 bulan
yang lalu. 1 bulan kemudian mengalami suara sengau diikuti bila minum kadang kadang tersedak.
Baal dan kesemutan tidak dikeluhkan. Pemeriksaan fisik didapatkan tetra paresis, atrofi lidah
dan otot-otot tangan. Refleks babinski (+), hoffman tromer (+). Apa pemeriksaan fisik khas yang
berkaitan dengan kasus ini? * ALS
a. Atrofi
b. Tetraparesis
c. Fasikulasi
d. Tremor
e. Hoffman tromer

59. Laki-laki 25 tahun dating ke poliklinik dengan keluhan kelemahan dimulai dari tungkai bawah
menjalar ke kedua lengan disertai perasaan kebas/tebal pada tangan dan kaki sejak 6 hari lalu.
Pemeriksaan fisik ; kekuatan extremitas atas 3, bawah 3. Sensibilitas hipoestesi glove and stocking
Pada peneriksaan electrodiagnostik, hasil apakah yang akan didapatkan ? GBS : asscending paralysis
a. Distal latensi motorik dan sensorik memanjang 10 harian
b. KHST motorik dan sensorik memanjang.

20
TO UGM 17 FEBUARI 2022
c. F-Wave dan H-Wave memanjang. (early GBS)
d. Amplitudo CMAP dan SNAP menurun.
e. Decrement test positif.  MG

60. Laki-laki, 40 tahun datang ke poliklinik syaraf dengan membawa hasil MRI kepala yang
menunjukkan adanya massa fossa posterior bilateral. Pada pemeriksaan fisik didapatkan café au lait
(NF2) dan riwayat keluarga gangguan pendengaran bilateral pada usia yang relatif muda. Apakah
kromosom yang mengalami kelainan pada kasus tersebut ? *Scwanoma vestibularis
a. 5
b. 13 : Wilson
c. 17 (NF1) tidak ada riwaayt keluarga  optic glioma, moyamoya
d. 21 : Amyloid protein precursor
e. 22 (swanoma + meningioma)  NF2 MISME

61. Seorang wanita 34 tahun, pasien rutin poliklinik saraf dengan epilepsi absance sejak 1 tahun yang
lalu. Pasien terkontrol kejangnya dengan Asam Valproat 2 x 500 mg. Saat kontrol pasien mengatakan
terlambat menstruasi 1 minggu kemudian disarankan pp test dan hasilnya positif. Bagaimana
tindak lanjut pengobatan epilepsi absance pada pasienn tersebut? Lalot
a. Mengganti asam valproat dengan phenitoin
b. Mengganti asam valproat dengan primidone
c. Mengganti asam valproat dengan lamotrigine (krn as valproat itu teratogenik)
d. Mempertahankan terapi rutin asam valproat  perika AFP dan asetilkolinterase (14-16
mgg), dan USG (16-20mgg)
e. Menambahkan terapi phenobarbital bersama asam valproat

62. Perempuan, 45 tahun datang ke poliklinik syaraf dengan keluhan nyeri kepala sisi kanan yang
semakin memburuk. Pemeriksaan signifikan untuk drift sisi kiri. Brain MRI dan CT scan
mengungkapkan homogeneously enhancing, round, dural based, calcified lesion mengompresi
lobus frontal kanan. Apakah terapi yang paling tepat pada kasus tersebut ? *Meningioma
a. Antikoagulasi
b. Terapi dengan isoniazid, rifampisin, etambutol.
c. Reseksi bedah  radioterapi bila residu tumor, dan ada rekurensi
d. Kemoterapi
e. Craniospinal axis irradiation
63. Seorang laki-laki 5 tahun dengan keluhan nyeri kepala, pusing, gangguan keseimbangan, dismetria,
dan disdiadokokinesia. Keluhan ini berlangsung kronik progresif sejak 5 bulan. Gambaran imejing
menunjukkan lesi tumor primer daerah serebelum. Pemeriksaan histopatologi mengesankan suatu
meduloblastoma. Manakah kriteria di bawah ini yang menunjukkan resiko tinggi angka rekurensi
kasus medulloblastoma? *
a. Ukuran tumor >2 cm
b. Lokasi tumor di fossa posterior
c. Tumor yang tidak direseksi dengan baik
d. Penurunan aktivitas reseptor C-tyrosine kinase
e. Usia <4 tahun dan >21 tahun saat terdiagnosis

21
TO UGM 17 FEBUARI 2022
Risio rendah rekurensi :
- Tidak ada tanda metastasis
- Resid volume kecil (<1,5 cm)
- Histologi tipe klasik atau desmoplastik

64. Seorang perempuan 45 tahun dengan kelemahan keempat anggota gerak yang dirasakan terjadi
perlahan sejak 6 bulan yang lalu. Keluhan nyeri menjalar juga dirasakan. Riwayat trauma disangkal.
Gambaran menunjukkan lesi metastasis berupa kompresi pada epidural torakal 6-8. Manakah
organ asal tumor primer yang paling sering menyebabkan metastasis? *
a. lymphoma
b. paru-paru (POKDI)
c. payudara (buku UI pertama payudara)
d. tulang
e. kolon
Secara keseluruhan penyebab paling umum : kanker paru-paru (25%), kanker prostat (16%), mieloma
(11%), limfoma non-Hodgkin (8%), dan kanker payudara (7%)

65. Anak perempuan 16 tahun memiliki riwayat hipotonia neonatal, dan gangguan “suck reflex” saat
menyusu. Saat beranjak dewasa beberapa kelainan dijumpai seperti keterlambatan perkembangan,
mulut yang lebar dan kaki yang kecil. Saat ini ia mengalami morbiditas obesitas,dan belum
mengalami menarche. Apakah diagnosa yang tepat pada pasien ini? *
a. Fragile x syndrome : tremor, ataksia, parkinsonism, disotonomia, ggn kognitif
b. Prader willi syndrome : hipotonus infantil, membutuhkan NGT saat neonatus, stuunting
(pendek, wajah dismofik dgn mulut lebar, kaki kecil), hipogonadisme, hiperfagi  obesitas,
menarke
c. Angelman syndrome : ggn intelektual, mikrosefal, epilepsi intractable, ataksia, (happy puppet
 inapropriate laugh), flail arm, wide based gait, prominent jaw
d. Sturge weber syndrome : port wine stain di wajah
e. Epidermal nevus :

66. Perempuan, 23 tahun, datang dengan keluhan pandangan kabur sejak 2 hari disertai nyeri pada
mata kanan. Menurut keluarga pasien mengalami ngompol sejak 2 tahun. Pemeriksaan neurologis
= dismetria lengan kanan, plantar ekstensor pd kaki kiri dan pelo. Apakah pemeriksaan
penunjang yang diperlukan untuk kasus tersebut ? *Multiple sclerosis (Mcdonald)
a. Visual evoked potential
b. Sural nerve biopsi
c. Elektroensefalografi
d. Elektromiografi
e. MRI kepala

67. Seorang laki-laki 56 tahun datang dengan keluhan kelemahan keempat anggota gerak sejak 5
bulan yang lalu. 1 bulan kemudian mengalami suara sengau diikuti bila minum kadang kadang

22
TO UGM 17 FEBUARI 2022
tersedak. Baal dan kesemutan tidak dikeluhkan. Pemeriksaan fisik didapatkan tetra paresis, atrofi
lidah dan otot-otot tangan. Refleks babinski (+), hoffman tromer (+). Apa komplikasi tersering
penyebab kematian pasien ini? *ALS
a. Fraktur berat tulang vertebra
b. Dekubitus terinfeksi
c. Gagal nafas akibat aspirasi
d. Penurunan kesadaran
e. Kejang tidak teratasi

68. Perempuan, 29 tahun dengan riwayat melanoma maligna mengeluhkan nyeri kepala ke dokter
perawatan primernya. Pemeriksaan neurologi dalam batas normal. Hasil pemeriksaan CT scan
berupa perdarahan kecil multiple pada otak. Dimanakah kemungkinan tersering lokasi lesi pada
kasus tersebut ? *
a. Gray-white junction
b. Thalamus (Ganglia basal DAN korteks serebri)
c. Fossa posterior
d. Caudal
e. Sella turcica
Khusus melanoma, lokasi metastasis lebih banyak terjadi di ganglia basalis dan korteks serebral
dibandingkan white-gray matter junction

69. Seorang anak 16 tahun datang ke poliklinik neurologi dengan keluhan oleh saksi mata adanya riwayat
kejang kelojotan berulang sejak 3 bulan lalu sebanyak 3 x, kira-kira 1x/bulan dengan pola serupa,
yang diawali pandangan kosong, lengan tertekuk kemudian diikuti kelojotan seluruh tubuh, mata
mendelik keatas, mulut mengeluarkan busa, berlangsung 2-3 menit. Setelah serangan pasien
tertidur 10 menit. Serangan biasanya terjadi saat pasien sedang duduk menonton TV. Ada riwayat
kejang demam saat kecil. Menurut klasifikasi seizure/epilepsi yang baru oleh ILAE 2017, keadaan ini
dapat digolongkan sebagai ... *
a. Generalized onset motor seizure
b. Focal onset to bilateral tonic clonic seizure
c. Generalized onset non motor seizure
d. Focal onset aware nonmotor seizure
e. Focal onset impaired awareness motor seizure

70. Seorang laki-laki 56 tahun datang dengan keluhan kelemahan keempat anggota gerak sejak 5 bulan
yang lalu. 1 bulan kemudian mengalami suara sengau diikuti bila minum kadang-kadang tersedak.
Baal dan kesemutan tidak dikeluhkan. Pemeriksaan fisik didapatkan tetra paresis, atrofi lidah dan
otot-otot tangan. Refleks babinski (+), hoffman tromer (+). Di antara pernyataan di bawah ini,
manakah hasil pemeriksaan elektrofisiologi yang paling mungkin untuk kasus ini? *ALS
a. NCS normal, EMG giant potensial
b. NCS normal, EMG polifasik

23
TO UGM 17 FEBUARI 2022
c. NCS abnormal, EMG normal
d. NCS abnornal, EMG polifasik
e. NCS normal, EMG normal

71. Seorang laki-laki 56 tahun datang dengan keluhan kelemahan keempat anggota gerak sejak 5 bulan
yang lalu. 1 bulan kemudian mengalami suara sengau diikuti bila minum kadang kadang tersedak.
Baal dan kesemutan tidak dikeluhkan. Pemeriksaan fisik didapatkan tetra paresis, atrofi lidah dan
otot-otot tangan. Refleks babinski (+), hoffman tromer (+). Apa pemeriksaan elektroneurofisiologi
terpilih untuk kasus ini ? *
a. Latensi distal-KHS dan H-refleks
b. Latensi distal-KHS dan EMG needle 3 ekstremitas (4 regio, craniobulbar, cervical,
thoracal, lumbosacral)
c. Latensi distal-KHS dan F-wave
d. Latensi distal-KHS dan SEP
e. Latensi distal-KHS dan EMG needle pada lidah

72. Anak laki-laki, 15 tahun mengalami gangguan penglihatan. Pada pemeriksaan penunjang ditemukan
angiomatosis multipel pada retina serta kista ginjal dan pankreas. Apakah diagnosis yang paling
mungkin pada kasus tersebut ? *
a. Glioblastoma multiforme
b. Meningioma
c. Hemangioblastoma
d. Ependymoma
e. Pinealoma

24
TO UGM 17 FEBUARI 2022
73. Perempuan, 9 tahun dibawa ke poliklinik syaraf dengan keluhan nyeri kepala dan pubertas
prekoks. Pemeriksaan neurologi didapatkan edema papil. Apakah kemungkinans diagnosis pada
kasus tersebut ? *
a. Tumor regio pineal
b. Oligodendroglioma
c. Kernohan kelas II astrocytoma
d. Brainstem glioma
e. Ependymoma
74. Perempuan, 9 tahun, dibawa ke poliklinik syaraf dengan keluhan gangguan tumbuh kembang. Pasien
juga mengalami pubertas prekoks dan kejang. Kejang diderita sejak 5 tahun yang lalu dan belum
terkontrol . Kejang biasanya didahului oleh episode tawa tak terkendali. Apakah penyebab tersering
keluhan pasien tersebut ? *Gellastic seizure
a. Craniopharyngioma
b. Choroid plexus papilloma
c. Giant aneurysma
d. Karsinoma metastase
e. Hypothalamic hamartoma (emosi, tawa, )

75. Seorang anak usia 5 tahun dibawa oleh ibunya ke poli anak dengan keluhan anaknya sulit berjalan,
jika berjalan pasien jinjit dan tidak bisa jongkok. Dari pemeriksaan fisik didapatkan pseudohipertrofi

25
TO UGM 17 FEBUARI 2022
dan gower sign. Apa pemeriksaan laboratorium yang dibutuhkan untuk membatu penegakkan diagnosa
tersebut? *DMD
a. Creatinin Kinase 100x
b. Ureum keratinin
c. Elektrolit
d. Troponin I
e. Glukosa

76. Perempuan, 25 tahun, datang ke poliklinik saraf dengan keluhan sakit kepala sisi kiri sejak 5 hari yang
lalu. Nyeri kepala berdenyut disertai mual/muntah dan takut terhadap cahaya. Intensitas nyeri sedang.
Kira-kira 3 menit sebelum nyeri kepala pasien merasakan penglihatannya kabur. Nyeri kepala timbul
pertama kali saat penderita duduk di kelas 3 SMP. Pemeriksaan CT scan kepala normal. Setelah diberi
obat selama 3 hari penderita merasa sembuh. Terapi abortif yang berbasis bukti (level of evidence) “A”
untuk kasus ini adalah? *Migraine aura
a. Paracetamol 500-1000 mg per 6-8 jam abortif
b. Amitriptilin 10-75 mg
c. Aspirin 500-1000 mg per 4-6 jam (cocok tapi entah LOE nya)
d. Ibuprofen 200 mg per-12 jam (400-800)
e. Kalium diklofenak 50-100 mg perhari (utk migraine frekuen, dosis cocok)

77. Seorang anak laki-laki usia 8 tahun, dibawa ke spesialis saraf dengan keluhan lemah jika berjalan.
Keluhan saat ini tidak diperhatikan mulai kapan oleh keluarga, namun sejak 2 tahun yang lalu, anak
tidak mau berolahraga. Kelemahan tidak berfluktuatif. Pemeriksaan neurologis didapatkan parapresis
dengan kekuatan 4/4. Pseudohipertrophy (+). Pemeriksaan awal yang harus dilakukan adalah ? *
a. Pemeriksaan EMG
b. Pemeriksaan Genetic
c. Pemeriksaan NCS
d. Pemeriksaan biopsy otot  definitif
e. Pemeriksaan CPK (pemeriksaan awal)

78. Seorang anak laki laki umur 14 tahun datang dengan keluhan kedutan pada wajah kanan, kadang
merasa kesemutan pada wajah kanan dan pipi kanan bagian dalam. Keluhan ini dialami sejak 2
tahun yang lalu. Riwayat penyakit keluarga terdapat saudara sepupunya yang terdiagnosis epilepsi.
Prognosis pada anak ini? * BECTS : 15-16 thn remisi  orofasial, oromotor, speech arrest, sensory
motor symptoms
a. Baik
b. Baik setelah dilakukan operasi
c. Akan berulang seumur hidup
d. Buruk
e. Tidak tahu

79. Wanita 69 tahun datang ke EMG Neurologi dengan keluhan kelemahan anggota gerak kiri sejak 2 hari
yang lalu saat pasien bangun tidur. Keluhan dirasakan memberat, mula-mula tungkai masih dapat

26
TO UGM 17 FEBUARI 2022
berjalan walaupun diseret dan lengan dapat menggenggam benda, tetapi sejak kemarin tungkai hanya
bisa digeser dan lengan tidak dapat digerakkan sama sekali. Keluhan disertai bicara rero dan mulut
mencong ke kanan. Pasien memiliki riwayat hipertensi dan DM. Pemeriksaan penunjang yang menjadi
gold standar pada pasien ini ? *
a. Lumbal pungsi
b. CT scan kepala
c. MRI kepala
d. Angiografi
e. USG karotis

80. Seorang anak usia 4 tahun dengan diagnosis Cerebral Palsy GMFCS 4. Pilihan terapi utama untuk
spastisitas pada anak ini adalah? *
a. Tizanidine
b. Diazepam
c. Botulinum toxin
d. Baclofen
e. Eperison HCl

- tingkat I : paling ringan,


- tingkat II : tidak memerlukan alat tambahan dalam mobilitas.
- tingkat III biasanya anak dapat duduk sendiri, mobilitas di lantai secara mandiri, dan bila berjalan
memerlukan ortosis dan alat untuk membantu mobilitas.
- tingkat IV : perlu bantuan untuk duduk, mobilitas terbatas dan perlu bantuan,
- tingkat V : anak sangat tergantung pada orang lain dalam kontrol postural, dan mobilitasnya

81. Seorang laki-laki berusia 60 tahun, BB: 88 kg datang ke poliklinik saraf dengan keluhan rasa kantuk
berlebihan di siang hari dan kesulitan berkonsentrasi. Menurut istrinya setiap malam pasien selalu
mendengkur, sering event napas berhenti dan terbangun dengan gerakan-gerakan motorik. Setiap bangun
pagi, pasien mengeluhkan nyeri kepala. Pemeriksaan polisomnografi semalaman didapatkan 20-40 fase
apnea per jam. Berapakah derajat beratnya OSA pada kasus di atas? *
a. Sangat berat
b. Berat
c. Sedang
d. Ringan
e. Normal

27
TO UGM 17 FEBUARI 2022
82. Seorang anak laki-laki usia 3 tahun, BB 20 kg, dengan epilepsy yang belum teratasi. Bentuk kejang,
tonik, myoklonik, tonik klonik. Saat ini terapi yang sudah diberikan adalah Asam Valproat 2 x 8 cc.
Apakah yang harus dilakukan oleh dokter spesialis saraf? *LGS - epilepsi refrakter sdh tidak mempan
dengan 1 OAE
a. Menambahkan Fenitoin 2 x 50 mg
b. Menambahkan Fenobarbital 2 x 30 mg
c. Menambahkan Clobazam 1 x 2 mg (sdh tidak mempan dengan 1 OAE)
d. Menambahkan Topiramat 2 x 50 mg (LINI III, lini II lamo)
e. Menambahkan Levetiracetam 1 x 100 mg

83. Seorang anak laki laki umur 14 tahun datang dengan keluhan kedutan pada wajah kanan, kadang
merasa kesemutan pada wajah kanan dan pipi kanan bagian dalam. Keluhan ini dialami sejak 2
tahun yang lalu. Riwayat penyakit keluarga terdapat saudara sepupunya yang terdiagnosis epilepsi.
Pemeriksaan penunjang apa yang dianjurkan pada anak ini? * BECTS
a. Tidak ada pemeriksaan penunjang yg dianjurkan
b. MRI kepala
c. EEG
d. Pemeriksaan Gula darah
e. Foto sinus

28
TO UGM 17 FEBUARI 2022
84. Seorang perempuan, 34 tahun, ibu rumah tangga mengeluh kesemutan pada tangan kanan. Pasien
dilakukan NCS. Manakah dari hasil NCS berikut yang mengalami kelainan? DL: distal latensi ; A:
amplitude; KHS: kecepatan hantar saraf *
a. NCS sensorik n. median kanan → DL: 3,6ms; A: 18mV; KHS: 36 ms. NCS motorik n.
median kanan→DL: 5,2ms; A: 4,1mV; KHS: 24 ms
b. NCS sensorik n. radial kanan → DL: 1.9ms; A: 33mV; KHS: 48ms. NCS motorik n. radial
kanan→ DL: 2,3ms; A: 7,2; KHS: 48 ms
c. NCS sensorik n. ulnar kanan → DL: 2,2ms; A: 24mV; KHS: 50 ms. NCS motorik n. ulnar
kanan→DL: 2,3ms; A: 8,1mV; KHS: 51ms
d. NCS sensorik n. sural kanan → DL: 2,1ms; A: 18mV; KHS: 44 ms. NCS motorik n. tibial
kanan→DL: 3,3ms; A: 11,2; KHS: 52 ms
e. NCS sensorik n. sural kanan → DL: 2,1ms; A: 18mV; KHS: 44 ms. NCS motorik n. peroneal
kanan→DL: 3,1ms; A: 6,2mV; KHS: 49 ms

85. Seorang anak laki laki umur 14 tahun datang dengan keluhan kedutan pada wajah kanan, kadang merasa
kesemutan pada wajah kanan dan pipi kanan bagian dalam. Keluhan ini dialami sejak 2 tahun yang lalu.
Riwayat penyakit keluarga terdapat saudara sepupunya yang terdiagnosis epilepsi. Bila dilakukan
pemeriksaan EEG Gambaran apa yang anda harapkan dari pemeriksaan EEG? *BECTS
a. Normal
b. Gelombang epileptogenik di frontal kiri
c. Gelombang epileptogenik di okssipital kiri
d. Gelombang paku di sentro temporal kiri (khas)
e. Perlambatan kontinu pada hemisfer kiri

86. Seorang wanita, 69 tahun datang ke EMG Neurologi dengan keluhan kelemahan anggota gerak kiri
sejak 2 hari yang lalu saat pasien bangun tidur. Keluhan dirasakan memberat, mula-mula tungkai masih
dapat berjalan walaupun diseret dan lengan dapat menggenggam benda, tetapi sejak kemarin tungkai
hanya bisa digeser dan lengan tidak dapat digerakkan sama sekali. Keluhan disertai bicara rero dan mulut
mencong ke kanan. Pasien memiliki riwayat hipertensi dan DM. Bila pada pasien ditemukan bruit pada
pemeriksaan A. karotis, pemeriksaan yang paling baik dilakukan? *
a. Lumbal pungsi
b. CT scan kepala
c. MRI kepala
d. Angiografi
e. USG karotis

87. Pasien laki-laki, usia 60 tahun datang dengan keluhan penglihatan ganda, gangguan pengecapan, dan
lemah tubuh sebelah kiri mendadak. Pada pemeriksaan fisik ditemukan parese N. VI kanan dan parese
N.VII perifer kanan disertai dengan adanya gangguan pengecapan 2/3 anterior lidah dan hemiparese
sinistra. Sindroma batang otak pasien di atas ? *
a. Foville syndrome  tambah PPRF, conjugate gaze palsy, 2/3 sensorik anterior lidah
b. Benedict syndrome
c. Milliard Gubler syndrome : Hanya nervus 6 dan 7 + Corticospinal tract
d. Weber syndrome

29
TO UGM 17 FEBUARI 2022
e. Cerebellar syndrome

88. Seorang anak laki laki umur 14 tahun datang dengan keluhan kedutan pada wajah kanan, kadang merasa
kesemutan pada wajah kanan dan pipi kanan bagian dalam. Keluhan ini dialami sejak 2 tahun yang
lalu. Riwayat penyakit keluarga terdapat saudara sepupunya yang terdiagnosis epilepsi. Pengobatan pada
anak ini? *BECTS : kalau perlu CBZ, Lamo, Oxca, Valpro
a. Konsul THT
b. Haloperidol
c. Tidak perlu terapi
d. Botox
e. Carbamazepine (karba valing la le)

89. Seorang wanita umur 20 tahun datang dengan keluhan kejang kelojotan seluruh tubuh, terutama saat
sedang menstruasi.. Sebelumnya merasa seperti berada ditempat yang tidak dikenal. Pada pemeriksaan
MRI otak, kelainan apa yang mungkin ditemukan? * tidak familiar tapi seharusny familiar (Jamais vu)
a. Tidak ada
b. Hipokampal sclerosis (TLE)
c. Tumor pituitary
d. Pelebaran ventrikel
e. Oedema otak

90. Seorang laki laki, 40 tahun, memiliki hobi bermain tenis lapangan, lengan kanan menjadi lemah Pasien
dilakukan NCS. Manakah dari hasil NCS berikut yang mengalami kelainan? DL: distal latensi ; A:
amplitude; KHS: kecepatan hantar saraf * Tennis elbow : Radial; Golfer elbow : ulnar
a. NCS sensorik n. median kanan → DL: 2,1ms; A: 28mV; KHS: 56 ms. NCS motorik n. median
kanan→DL: 2,3ms; A: 9,2mV; KHS: 54 ms
b. NCS sensorik n. radial kanan → DL: 2,0ms; A: 25mV; KHS: 47ms. NCS motorik n.
radial kanan→DL: 2,3ms; A: 5,2mV; KHS: 51 ms
c. NCS sensorik n. ulnar kanan → DL: 5,6ms; A: 12mV; KHS: 22 ms. NCS motorik n. ulnar
kanan→DL: 7,3ms; A: 1,9mV; KHS: 19 ms
d. NCS sensorik n. sural kanan → DL: 2,1ms; A: 18mV; KHS: 44 ms. NCS motorik n. tibial
kanan→DL: 3,3ms; A: 11,2; KHS: 52 ms
e. NCS sensorik n. sural kanan → DL: 2,1ms; A: 18mV; KHS: 44 ms. NCS motorik n. peroneal
kanan→DL: 3,1ms; A: 6,2mV; KHS: 49 ms

91. Seorang wanita, 69 tahun datang ke EMG Neurologi dengan keluhan kelemahan anggota gerak
kiri sejak 2 hari yang lalu saat pasien bangun tidur. Keluhan dirasakan memberat, mula-mula
tungkai masih dapat berjalan walaupun diseret dan lengan dapat menggenggam benda, tetapi
sejak kemarin tungkai hanya bisa digeser dan lengan tidak dapat digerakkan sama sekali.
Keluhan disertai bicara rero dan mulut mencong ke kanan. Pasien memiliki riwayat hipertensi
dan DM. Manakah obat antiagregasi di bawah ini yang bekerja menghambat ADP? *
a. Aspirin  COX
b. Clopidogrel  ADP
c. Abciximab  GpIIb IIIa inhibitor (monoklonal antibodi)

30
TO UGM 17 FEBUARI 2022
d. Cilostazol  PDE4
e. Triflusal  COX, TxA2

92. Seorang wanita umur 20 tahun datang dengan keluhan kejang kelojotan seluruh tubuh, terutama
saat sedang menstruasi.. Sebelumnya merasa seperti berada ditempat yang tidak dikenal.
Apakah diagnosis yang anda pikirkan *TLE (Jamais vu)
a. Syncope
b. Serangan panik
c. Bangkitan non epileptik psikogenik
d. Dyskinesia paroksismal
e. Bangkitan fokal
93. Seorang wanita umur 20 tahun datang dengan keluhan kejang kelojotan seluruh tubuh, terutama
saat sedang menstruasi.. Sebelumnya merasa seperti berada ditempat yang tidak dikenal.
Apakah terapi yang akan anda berikan? *TLE ini bukan katamenial
a. Paracetamol
b. Asam Valproate
c. Carmabazepin (karena bangkitan fokal)
d. Clobazam + astazolamid (katamenial)
e. Betabloker

94. Pasien laki-laki, usia 60 tahun datang dengan keluhan penglihatan ganda, gangguan
pengecapan, dan lemah tubuh sebelah kiri mendadak. Pada pemeriksaan fisik ditemukan parese
N. VI kanan dan parese N.VII perifer kanan disertai dengan adanya gangguan pengecapan 2/3
anterior lidah dan hemiparese sinistra. Dimanakah lokasi lesi pasien di atas? *
a. Pons
b. Genu interna
c. Chorda tympani  membawa sensorik 2/3 anterior
d. Cerebellum
e. Medulla oblongata

95. Seorang wanita umur 20 tahun datang dengan keluhan kejang kelojotan seluruh tubuh, terutama
saat sedang menstruasi.. Sebelumnya merasa seperti berada ditempat yang tidak dikenal. Bila
dilakukan pemeriksaan EEG, apa yang diharapkan? *TLE
a. Normal
b. Artefak gerak
c. Gelombang epileptogenik pada daerah temporal
d. Gelombang epileptogenik pada daerah frontal
e. Gelombang epiletogenik menyeluruh

96. Seorang wanita umur 20 tahun datang dengan keluhan kejang kelojotan seluruh tubuh, terutama
saat sedang menstruasi. Sebelumnya merasa seperti berada ditempat yang tidak dikenal.
Pemeriksaan penunjang yang akan dilakukan *
a. EKG
b. EEG

31
TO UGM 17 FEBUARI 2022
c. EMG
d. MRI otak
e. PSG
97. Seorang laki-laki, usia 65 tahun, pasca perawatan stroke penyumbatan 4 minggu yang lalu.
Pasien mengalami gangguan bicara tetapi masih dapat mengerti pembicaraan dan
hemiparesis kanan. Pasien bedridden. Pasien diketahui menderita hipertensi dan diabetes
mellitus sejak 15 tahun yang lalu tetapi control tidak teratur, obat antihipertensi hanya diminum
jika nyeri kepala saja. Apakah masalah utama pada pasien di atas ? *
a. Pasca stroke infark
b. Disabilitas dan kepatuhan yang buruk
c. Hipertensi dan diabetes mellitus
d. Tirah baring dan gangguan komunikasi
e. Independensi

98. Seorang anak laki laki umur 14 tahun datang dengan keluhan kedutan pada wajah kanan,
kadang merasa kesemutan pada wajah kanan dan pipi kanan bagian dalam. Keluhan ini
dialami sejak 2 tahun yang lalu. Riwayat penyakit keluarga terdapat saudara sepupunya yang
terdiagnosis epilepsi. Apakah diagnosis kemungkinan yang dipikirkan pada anak ini *BECTS
a. Hemifasial spams
b. Epilepsi fokal
c. Sinusitis
d. Movement disorder
e. Bangkitan non epileptik psikogenik

99. Seorang anak laki-laki, usia 18 tahun, datang dengan keluhan sakit kepala sebelah kiri, yang
terjadi sejak 5 hari yang lalu, serangan saat ini telah berlangsung sebanyak 5 kali dalam
seminggu. saat serangan penderita tidak bisa melakukan aktifitas rutin, sifat nyeri
berdenyut. Serangan nyeri kepala seperti ini dialami pertama kali saat umur 14 tahun. Nilai
VAS 6. Pemeriksaan CT scan kepala hasilnya normal. Obat apa yang tepat diberikan pada
penderita ini? *Migraine nya terapi abortif dan profiaksis langsung
a. Flunarizine dan asam valproat
b. NSAIDs, dan parasetamol.
c. Ergotamin dan NSAIDs
d. Ergotamin dan flunarizine spesifik abosrtif
e. Asam valproat dan verafamil.

100. Seorang laki-laki berusia 30 tahun datang ke poliklinik saraf dengan gangguan tidur.
Pasien sering terbangun lebih awal/dini daripada yang diinginkan sehingga sering muncul rasa
lelah, penurunan energi dan motivasi, perubahan mood dan penurunan produktivitas di tempat
kerja. Berdasarkan profil keluhan pasien. Manakah kemungkinan diagnosis untuk kasus di
atas? *
a. Sleep-onset insomnia

32
TO UGM 17 FEBUARI 2022
b. Sleep maintenance insomnia
c. Terminal insomnia
d. Nonrestorative sleep
e. Narkolepsi

33
TO UGM 17 FEBUARI 2022
SOAL MCQ NASIONAL APRIL 2020 DARI dr.GOFIR

1. Laki-laki 58 tahun datang ke poli Saraf dengan keluhan nyeri kepala berdenyut berulang. Pada pemeriksaan
CT angiografi didapatkan unruptur aneurisma MCA kiri. Pasien menanyakan tentang risikonya. Manakah
dari berikut ini yang paling benar ?
A. Pasien mempunyai resiko kecil untuk terjadi SAH
B. Merokok tidak berpengaruh terhadap risiko pecahnya aneurisma
C. Aneurisma sirkulasi anterior mempunyai resiko ruptur sama dibandingkan aneurisma posterior (risiko
ruptur Posterior > Anterior)
D. Usia pasien bukan menjadi pertimbangan dalam strategi tatalaksana Coiling
E. Hipertensi tidak berperan dalam risiko pecahnya aneurisma karena aneurisma murni disebabkan
kelainan anatomis
P : Populasi; H hypertension; A Age; S Size aneurism; E earlier subarachnoid hemorrhage; S Site aneurism

2. Perempuan, 35 tahun, HIV negatif datang dengan keluhan utama penurunan kesadaran sejak 5 hari yang
lalu. Terdapat riwayat demam tinggi & nyeri kepala sejak 7 hari yang lalu. GCS 13. Terdapat kaku kuduk.
Tidak ditemukan gambaran papiledema atau tanda-tanda neurologis fokal. Direncanakan untuk melakukan
pungsi lumbal. Setelah jarum spinal menembus ligamentum interspinosum maka struktur berikutnya yang
ditembus adalah?
A. Ligamentum longitudinal posterior (6)
B. Ligamentum supraspinosum (1)  interspinosum (2)
C. Ligamentum flavum (3)
D. Araknoid (5)
E. Duramater (4)
SIFDAR/SIFDAS

3. Seorang ibu mengeluh anak lelakinya yang berusia 7 tahun mengalami kesulitan menaiki tangga sejak 1
tahun yang lalu. Sejak usia 3 tahun, anak tersebut ketika berdiri dari duduk selalu terlihat tangannya
bertumpu pada benda di sekitarnya. Setelah ditelusuri, ternyata paman ibu tersebut juga mengalami hal
serupa pada saat kecil, dan meninggal ketika berusia 18 tahun. Pada pemeriksaan needle EMG anak
tersebut, gelombang apakah yang sangat diharapkan muncul ?DMD
A. Positive sharp wave
B. Polyphasic.
C. Fibrilasi.
D. Fibrilasi.
E. Myokimic discharge.

4. Seorang wanita 60 tahun sering mengeluh pusing dengan riwayat hipertensi terkontrol Pada pemeriksaan
fisik ditemukan gangguan koordinasi. Hasil pemeriksaan sonologi dan angiografi ditemukan stenosis pada
arteri basilaris. Apakah Jalur kolateral dari sistem ekstrakranial yang berperan menjaga stabilitas CBF pada
kortikal posterior kasus tersebut ?
A. Arteri maksilaris
B. Arteri oksipital
C. Arteri temporalis superfisial
D. Arteri fasialis
E. Arteri asending faringeal
BERULANG

5. Seorang bayi baru lahir, usia 5 hari, dikonsulkan dengan kecurigaan cerebral palsy. Bayi dilahirkan saat
kehamilan 32 minggu dengan Apgar score 3 dan 5. Pada pemeriksaan fisik didapatkan hipotonus dan
peningkatan deep tendon reflexes. Bagaimanakah jawaban konsul yang paling tepat untuk kasus tersebut ?
A. Diagnosis CP sangat sulit ditegakkan sebelum usia 6 bulan
B. Temuan hipotonus dan hiperrefleks mengindikasikan kelak pasien mengalami CP
C. Rendahnya Apgar score menjadi prediktif kuat terjadinya CP

34
TO UGM 17 FEBUARI 2022
D. Asfiksia perinatal; merupakan penyebab terkuat CP
E. Pasien pasti mengalami CP dan 33% di antaranya akan mengalami retardasi mental

6. Laki-laki berusia 67 tahun datang ke IGD RS dengan keluhan nyeri kepala setelah terbentur tiang listrik 2
jam yang lalu. Pasien tidak ada pingsan, muntah, perdarahan dari hidung atau telinga, dan kejang. Saat
diperiksa di IGD, VAS = 6, tampak sefal hematom di region temporal kanan. Tidak ada defisit neurologis.
Apakah diagnosis klinis pasien ini?
A. Cedera kepala ringan
B. Cedera kepala berat
C. Cedera kepala sedang
D. Konkusio cerebri
E. Kontusio cerebri

7. Seorang laki-laki 58 tahun datang ke RS dengan keluhan bila melihat ke kanan menjadi berbayang. Pada
pemeriksaan fisik didapatkan diplopia, nistagmus horizontal dextra. Gangguan adduksi mata kiri (+)
A. MLF sinistra
B. MLF sinistra
C. PPRF sinistra
D. N. Oculomotor sub medial sinistra
E. MLF bilateral

8. Laki-laki usia 52 tahun, diantar oleh keluarganya ke Poli Saraf dengan keluhan mendadak tidak mampu
menulis dan membaca apa yang telah dituliskannya. Pada pemeriksaan fungsi kortikal luhur ditemukan
adanya aleksia disertai agrafia. Dimanakah letak lesi kasus tersebut?
A. Regio temporal hemisfer kiri
B. Regio oksipital hemisfer kiri
C. Regio parietal hemisfer kiri
D. Girus angularis hemisfer kiri MCA POST
E. Girus kalkarina hemisfer kiri

9. Seorang laki-laki usia 60 tahun datang ke poliklinik dengan keluhan nyeri kepala sejak 2 bulan. Nyeri
dirasakan tiba-tiba dan memberat, kadang berdenyut kadang seperti ditusuk-tusuk. Nyeri dirasakan
unilateral terutama di sebelah kanan, intensitas menetap selama siang hari dan memberat di malam hari.
Keluhan diawali dengan serangan berulang mata sebelah kanan yang tiba-tiba menjadi gelap selama
beberapa waktu sebelumnya (amourosis fugax). Pasien juga mengatakan merasa tidak enak badan, kadang
terasa demam ringan dan mengalami penurunan berat badan. Pasien pernah berobat sebelumnya dan
diperiksakan laboratorium dengan hasil : Hb 10,5 g/dL, Ht 33, L 10.000, Tr 450.000, LED 80 mm/jam,
CRP 2,5 mg/dL. Apakah diagnosis pada pasien ini ?
A. Nyeri kepala klaster
B. Migrain retinal
C. Temporal Arteritis (Giant Cell Arteritis)
D. Nyeri kepala sekunder ec SOL
E. Paroxysmal Hemicranial

10. Laki-laki, 52 tahun, datang ke IGD dengan sakit kepala sejak 8 jam yang lalu. Sakit kepala dirasakan
muncul tiba-tiba, baru pertama kali dan sangat hebat. Pada pemeriksaan fisik diperoleh tekanan darah
150/90 mmHg, frekuensi nadi 92 x/menit regular. Pada pemeriksaan status neurologis GCS 15, pupil isokor
3mm/3mm reflex cahaya baik, kaku kuduk positif, defisit neurologis lain negatif. Hasil CT scan kepala
diperoleh perdarahan fisura interhemisferik dan fissure sylvii bilateral. Apakah tatalaksana farmakologis
yang paling tepat untuk mencegah perdarahan ulang pada pasien ini ? SAH : Target TD <160
A. Manitol
B. Nikardipin
C. Nimodipin
D. AsamTraneksamat
E. Faktor VII

35
TO UGM 17 FEBUARI 2022
11. Seorang laki-laki, usia 50 tahun, 5 hari lalu menjalani operasi amputasi tungkai kanan di bawah lutut
(below Knee) sisi kanan karena kecelakaan. Sejak 3 hari terakhir pasien merasakan nyeri tumpul dan
berdenyut pada sekitar lutut dan area luka amputasi. Apa diagnosis yang tepat pada kasus tersebut?
A. Psychogenic pain
B. Phantom limb pain
C. Stump related pain
D. Phantom limb sensation
E. Iatrogenic neuropathy

12. Seorang Laki-laki berusia 30 tahun diantar ke IGD RS dengan keluhan kejang. Kejang kelojotan ini baru
pertama kali dialami, saat kejang pasien tidak sadar, durasi kurang dari 5 menit, setelah kejang pasien
merasa kebingungan. Pasien memiliki riwayat minum alkohol. Pada pemeriksaan didapatkan babinski
positif bilateral. Apakah penyebab kejang pada pasien tersebut ?
A. Hiperglikemi
B. Hipoksia otak
C. Hipoksia global
D. Imbalance elektrolit
E. Kadar ureum yang tinggi

13. Pada tahun 2001, dilakukan identifikasi pada 100 perempuan yang bekerja di suatu pabrik arloji. Insidens
kanker otak pada kelompok perempuan tersebut sampai tahun 2005 dibandingkan dengan kanker otak pada
100 perempuan pekerja operator telepon di tahun yang sama. Apakah jenis studi ini?
A. Uji klinik
B. Studi prospektif (MAJU TAHUNNYA)
C. Studi kasus kontrol
D. Studi kros seksional
E. Studi eksperimental

14. Laki-laki usia 20 th mengeluh lengan kiri tidak bisa digerakkan dengan sempurna sejak 2 bulan yang lalu .
Pada pemeriksaan didapatkan abnormalitas pada abduksi lengan atas, fleksi pada siku, dan supinasi lengan
bawah. Pemeriksaan reflek trisep masih normal. Di manakah kemungkinan besar letak lesi yang dialami
pasien tersebut ?ERB palsy
A. Upper trunk pleksus brachialis
B. Middle trunk pleksus brachialis.
C. Lower trunk pleksus brachialis.
D. Posterior cord pleksus brachialis
E. Medial cord pleksus brachialis

15. Seorang laki-laki 58 tahun datang ke RS dengan keluhan bila melihat ke kanan menjadi berbayang. Pada
pemeriksaan fisik didapatkan diplopia. Gangguan adduksi mata kiri (+). Dimanakah letak lesi pada pasien
tersebut?
A. MLF sinistra
B. MLF dextra
C. PPRF sinistra
D. N. Oculomotor sub medial sinistra
E. MLF bilateral

16. Anak laki-laki, 10 tahun dirujuk dengan diagnosis epilepsi dan retardasi mental. Pada pemeriksaan fisik
didapatkan beberapa area hipopigmentasi ovoid pada kulit serta sejumlah papul warna merah dan kuning
di wajah, terutama sekitar mulut. Hasil pemeriksaan PA papula adalah angiofibroma. Apakah diagnosis
kasus tersebut ?
A. Acoustic neuroma
B. Hemangioblastoma kapiler
C. Stuger weber
D. Hamartoma
E. Leptomeningeal angiomatosis

36
TO UGM 17 FEBUARI 2022
17. Laki-laki 60 tahun datang dengan keluhan gangguan berjalan yang makin memberat dalam 6 bulan terakhir,
kelemahan dan parestesia pada tungkainya, dan memiliki keluhan nyeri punggung bawah.
Hasil MRI

Apakah kemungkinan diagnosis pada pasien ini?


A. Radicular arteriovenous malformations
B. Spinal dural arteriovenous fistula
C. Infark medula spinalis
D. Epidural hematoma
E. Venous angioma

18. Laki-laki, usia 60 tahun, datang ke poliklinik untuk kontrol penyakit stroke. Satu bulan yang lalu pasien
didiagnosis stroke iskemik dengan kelemahan mendadak sisi kiri dan mulut mencong. Pasien telah
mendapatkan aspirin 1 x 80 mg PO, simvastatin dan captopril. Pasien menunjukkan hasil profil lipid dalam
batas normal. Berdasarkan studi manakah yang merekomendasikan lipid-lowering agent untuk prevensi
stroke sekunder ?
A. ESPRIT
B. ACTIVE
C. SPARCL : stroke prevetion
D. CHARISMA
E. CHARISMA

19. Perempuan 42 tahun dibawa ke IGD dengan nyeri kepala mendadak yang sangat hebat disertai muntah
dan gangguan penglihatan. Demam, riwayat trauma, hiperteni dan diabetes mellitus disangkal. Pada
pemeriksaan tanda vital diperoleh tekanan darah 150/95 mmHg, frekuensi nadi 92 x/menit regular.
Pemeriksaan neurologis diperoleh GCS 13, kaku kuduk positif, kesan parese nervus III bilateral.
Gambaran MRI kepala sebagai berikut Dx. APOPLEKSI HIPOFISIS

37
TO UGM 17 FEBUARI 2022
Apakah tatalaksana kegawatan yang paling tepat pada pasien ini?
A. Antikoagulan IV
B. Kortikosteroid dan tindakan operasi segera
C. VP shunt dan kortikosteroid
D. Antikoagulan IV dan external ventricular drainase (EVD)
E. Antikoagulan IV dan kortikosteroid

20. Perempuan 44 tahun datang ke klinik saraf dengan keluhan sering pusing berputar dan pingsan dalam 1
bulan terakhir. Pemeriksaan Tensi lengan kanan 90/60, lengan kiri 180/90. Dari pemeriksaan Carotis
duplex didapatkan A vertebralis kanan mengalami reverse flow, sedangkan sistema karotis dan a
vertebralis kiri menunjukkan normal flow. Apakah diagnosis etiologi yang paling mungkin pada pasien
tersebut?Subclavian stell syndrome
A. Oklusi a brachiocefalic
B. Oklusi a. Sub-clavia kanan
C. Oklusi a. sub-clavia kiri
D. Oklusi a vertebralis kanan
E. Oklusi a vertebralis kiri

21. Laki-laki 18 tahun mengalami kelemahan tungkai dan tangan 3 hari sebelum masuk rumah sakit. Diawali
dengan kesemutan pada kaki kemudian sampai ke tangan. Sehari setelah itu kedua tangan dan kaki terasa
berat diangkat, hari ini sudah tidak bisa berjalan. RPD: Diare akut 1 minggu lalu. Pemeriksaan fisik:
Kekuatan motorik keempat anggota gerak menurun, distal lebih berat, disertai arefleks biceps. Sensorik:
hipestesi sarung tangan dan kaki. Apa rekomendasi terapi yang paling direkomendasikan untuk kasus ini?
A. Kortiko steroid dosis tinggi
B. IVIg 0.4 mg/kg BB selama 5 hari
C. IVIg 0.5 mg/kg BB selama 5 hari
D. IVIg 0.6 mg/kg BB selama 5 hari
E. Ivig 0.7 mg/kg BB selama 5 hari

22. Seorang anak laki-laki usia 6 tahun dibawa ke poliklinik saraf untuk pemeriksaan EEG karena keluhan
anak sering bengong selama beberapa detik. Apa kemungkinan hasil EEG pada pasien tersebut?CAE
A. Burst suppression (otahara)
B. Slow spike and wave complex / atypical Absans  LGS : GPFA
C. 3 HZ spike and wave complex (bisa jg doble SWC amplitudo tinggi)
D. Periodic lateralized epileptiform discharges : ensefalitis
E. Hypsaritmia west

23. Seorang perempuan berusia 55 tahun datang ke poliklinik dengan keluhan pusing berputar sejak 2 hari yang
lalu. Pusing hilang timbul, saat muncul durasi sekitar 30 menit. Pusing dirasakan hebat, disertai telinga
berdenging dan gangguan pendengaran. Pusing seperti ini sudah yang ke-3 kali dan gangguan
pendengaran semakin berat. Hipertensi, DM, trauma, batuk pilek disangkal.

38
TO UGM 17 FEBUARI 2022
Pada pemeriksaan fisik didapatkan tuli sensorineural kiri. Apakah tatalaksana medikamentosa yang
paling tepat pada pasien ini?meniere disease
A. Metilprednisolon
B. Acyclovir
C. Mecobalamin
D. Betahistin 48 mg
E. Flunarizin

24. Seorang perempuan, 18 tahun, datang ke poliklinik Neurologi dengan keluhan utama nyeri kepala sebelah
kiri. Keluhan sudah sejak 6 bulan yang lalu. Bersifat hilang timbul. Gejala diawali dengan adanya
gangguan pada lapang pandang yang berlangsung kurang lebih 30 menit diikuti nyeri yang menusuk-
nusuk di sisi kiri kepala, mual, dan silau bila melihat cahaya. Lamanya nyeri kurang lebih 6-8 jam. Ibu
pasien juga mengeluhkan keluhan seperti ini. Pemeriksaan fisik tidak ada kelainan.. Apakah diagnosis yang
paling mungkin untuk kasus di atas?
A. Migren klasik
B. Migren tanpa aura.
C. Nyeri kepala klaster.
D. Hemikrania simpleks.
E. Migren dengan aura tipikal. : aura visuall/sensorik dan/atau ggn bahasa/bicara

25. Seorang laki-laki, usia 45 tahun, dibawa berobat ke poliklinik saraf dengan keluhan kejang berulang.
Kejang pada anggota gerak kiri diikuti seluruh tubuh yang dialami sejak 2 tahun yang lalu setelah
mengalami stroke. Pada pemeriksaa fisik didapatkan hipertensi, dan gagal ginjal kronis. Apakah OAE yang
paling tepat pada pasien ini?
A. Zonisamide
B. Carbamazepin (baik untuk gagal ginjal), feni dan valpro
C. Pregabalin
D. Vigabatrin
E. Gabapentin

26. Perempuan, 35 tahun datang ke poliklinik saraf dengan keluhan nyeri dan tebal pada pergelangan tangan
kanan hingga ujung-ujung ibu jari dan jari telunjuk. Keluhan dirasakan sejak 1 tahun yang lalu. Pasien
mulai merasakan kelemahan dalam memegang bolpoin. Pemeriksaan neurologi : tinel test (+), phallen test
(+), atrofi (+), luthy sign (+). Dokter menyarankan pasien untuk operasi. Apakah alasan operasi pada kasus
tersebut ?
A. CTS kronis
B. atrofi otot-otot yang bersangkutan
C. CTS bilateral
D. keluhan ringan tapi tidak respon terhadap terapi medikamentosa lebih dari 3 bulan
E. permintaan pasien

27. Seorang laki laki berusia 65 tahun datang ke Poliklinik Saraf dengan keluhan pusing sejak 3 bulan yang
lalu. Pusing hilang timbul, tetapi biasanya berlangsung 10 – 15 detik. Ia mengatakan bahwa pusingnya
bertambah ketika ia berguling ditempat tidur atau ia bangun dari tidurnya. Suatu ketika ia merasakan
pusing berlebihan ketika menggapai sesuatu diatas kepalanya. Ia tidak mengeluh mual atau muntah
yang berhubungan dengan pusing ini. Ia mencoba menghindari tidur miring. Tidak ada mengeluhkan
gangguan pendengaran atau tinnitus. Juga tidak ada mengeluhkan adanya nyeri kepala. Tidak pernah
berobat sebelumnya dan juga tidak ada memakan obat apapun. Dari riwayat diatas, maka diagnosa yang
paling mungkin adalah;
A. Central vestibular vertigo.
B. Vestibular paroxysmal : monosymptomatic vertigo, bisa spontan bisa trigger, bisa karena
hiperventilasi, dan fenomena klinis stereotipik
C. Benign paroxysmal positioning vertigo : trigger tersering menengadah dan miring kan kepala
saat tidur
D. Vestibular neuronitis
E. Meniere’s disease

39
TO UGM 17 FEBUARI 2022
28. Perempuan 65 tahun datang ke klinik memori dengan keluhan mudah lupa, terutama ingatan baru. Hal ini
sudah dialaminya sejak 1 tahun terakhir. Dokter melakukan pemeriksaan CERAD Test terhadap pasien ini.
Domain fungsi kognitif apakah yang dapat dinilai dengan melakukan pemeriksaan CERAD Test ?
A. Atens
B. Bahasa
C. Visuospasial
D. Fungsi eksekutif
E. Memori

29. laki-laki 54 tahun, datang ke poli saraf dengan keluhan nyeri kepala yang hilang timbul, seperti ditekan,
dirasakan di seluruh bagian kepala. Pasien pernah jatuh dari kotor 1 bulan yang lalu dan sempat tidak
sadar. Saat ini dari pemeriksaan fisik tidak didapatkan kelainan neurologi. Pasien juga kadang merasakan
kepalanya berat sehingga mengganggu aktifitasnya sebagai tukang parkir. Apakah Pemeriksaan penunjang
yang paling sesuai pada kasus ini ?
A. MRI kepala
B. CT Scan kepala
C. EEG
D. X foto tengkorak
E. pemeriksaan LCS

30. Seorang laki-laki 45 tahun dibawa ke IRD RS karena mendadak mengeluh nyeri kepala hebat disertai
muntah. Pemeriksaan fisik didapatkan tanda Brudzinki I dan II (+). Seorang laki-laki 45 tahun dibawa ke
IRD RS karena mendadak mengeluh nyeri kepala hebat disertai muntah. Pemeriksaan fisik didapatkan
tanda Brudzinki I dan II (+). Apakah sistem skoring yang tepat Untuk menilai derajat keparahan pada kasus
tersebut ?
A. Skor Hunt – Hess
B. SKor Siriraj
C. Skor Splitz Martin
D. Skor Fisher
E. Skor Cognard

31. Seorang laki-laki 60 tahun datang ke poliklinik saraf karena menurut istrinya sejak 3 bulan ini sering
melakukan gerakan- gerakan berulang saat tidur, hal ini terjadi beberapa jam sebelum suaminya
bangun tidur. Selama tidur suaminya memukuli guling kemudian terbangun secara tiba-tiba dan berbicara
pada guling tersebut. Saat pagi hari suaminya mengatakan bahwa dia bermimpi diserang oleh penjahat.
Mekanisme yang mendasari munculnya kondisi tersebut adalah? RBD
A. Abnormalitas transporter dopamin pada sistem nigrostriatal  RBD
B. Abnormalitas transporter glutamat pada jalur kortikospinal
C. Abnormalitas transporter GABA di formasioretikularis
D. Abnormalitas transporter serotonin pada nukleus raphe
E. Abnormalitas transporter histamin di posterior hipotalamus

32. Perempuan, 7 tahun datang dengan keluhan gerakan pipi kiri bergerak-gerak dan keluar air liur pada
malam hari. Saat keluhan tersebut muncul, pasien tetap sadar. Kadang-kadang keluhan disertai kaku pada
lengan kiri. Riwayat tumbuh kembang normal. Riwayat keluarga : kakak laki-laki pasien mengalami
riwayat kejang demam. Manakah pertanyaan yang paling tepat untuk kasus tersebut ?
A. Herediter autosomal dominan
B. Hasil EEG spike wave amplitudo tinggi di centrotemporal
C. Usia saat onset 3 – 13 tahun dengan puncak usia 4-5 tahun
D. Frekuensi munculnya spike wave berkorelai dengan kemungkinan serangan berikutnya
E. Kejang akan mengalami remisi ketika dewasa tanpa perlu OAE (pubertas bukan dewasa)

40
TO UGM 17 FEBUARI 2022
33. Laki-laki, 31 tahun, mengalami nyeri kepala, mengantuk dan gangguan keseimbangan yang semakin
memburuk dalam seminggu terakhir. Pasien adalah penderita HIV positif, tetapi belum mendapatkan terapi
ARV dalam satu tahun ini. Pasien mengalami hemiparesis kanan dan peningkatan refleks fisiologis pada
ekstrimitas sebelah kanan. Pemeriksaan darah rutin dan kimia darah masih dalam batas normal. CT scan
kepala dengan kontras menunjukkan suatu lesi ring enhancement. Apakah mikroorganisme yang paling
mungkin pada kasus tersebut?
A. Cryptococcus neoformans : bubble soap sign; tinta india
B. Toxoplasma gondii
C. TB
D. CMV Owl Eye
E. Herpes zozter

34. Seorang perempuan 26 tahun dengan berat badan 100 kg dengan hipertensi mengeluhkan nyeri kepala yang
memberat. Nyeri kepala dikatakan terdapat pada kedua bagian depan dan belakang kepala seperti tertekan.
Pasien terkadang merasakan mual muntah saat nyeri memberat dan padangan mata mengabur. Apakah
diagnosis yang paling memungkinkan pada pasien adalah
A. Migrain
B. Idiopathic intracranial hypertension
C. Tension type headache
D. Posterior fossa mass
E. Nyeri kepala persisten

35. Laki-laki, 72 tahun, S1,pensiunan PNS di bawa ke poli Saraf, dengan keluhan sejak 9 bulan lalu berjalan
lambat pendek pendek, tangan kanan tremor. Sudah berobat ke Sp. S dan mendapat terapi Levodopa 3x 250
mg. sejak 2 bl ini muncul halusinasi. Hasil pemeriksaan neurologis (+) dijumpai Bradikinesia, face mask,
Tremor postural, refleks glabela +. Assessmen neuropsikologi dan neuropsikiatri : Demensia, Halusinasi,
dan Delusi. Apakah proses yang paling mungkin yang mendasari kelainan ini ?DLB
A. Difuse Axonal Injury
B. NMDA receptor damage
C. Gen Presenilin 1mutation
D. Αlfa-Synuclein aggregates
E. Cascade amyloid plaque

36. Laki-laki , 55 tahun, datang ke UGD dengan keluhan nyeri kepala secara tiba-tiba sekitar 2 jam
sebelumnya. Dijumpai tekanan darah 200/100 mmHg. Pada pemeriksaan funduskopi dijumpai gambaran
fundus hipertensif. Pemeriksaan motorik dan kranialis lainnya normal. Gambaran apakah yang
ditunjukkan oleh panah hitam pada funduskopi ini ?
A. Crossing sign : av nicking
B. Copper wire appearance
C. Cotton wool spots
D. Flame shaped
E. Macular star

37. Perempuan, 55 tahun, dating ke poli saraf dengan keluhan rasa nyeri di pantat kanan sejak 3 bulan yang
lalu. Keluhan nyeri terkadang menyebar hingga daerah punggung bawah dan sering dikeluhkan pada saat
pasien hendak berdiri dari posisi duduk. Keluhan tidak disertai rasa baal, kelemahan anggota gerak,
maupun gangguan BAK dan BAB. Pada pemeriksaan Laseq> 70 dan Kerniq> 135, sedangkan pada
pemeriksaan Patrick dan Compression test pasien mengeluh nyeri. Apakah topical diagnosis yang paling
mungkin terjadi pada kasus nyeri punggung bawah di atas ?
A. Nyeri Facet joint
B. Canalis stenosis
C. Nyeri Sacroiliaca joint
D. Myofacial pain
E. Sindrom piriformis

41
TO UGM 17 FEBUARI 2022
38. Laki-laki, 42 tahun datang dengan keluhan nyeri pada tangan kirinya. Pasien memiliki riwayat biciptal
tendinopathy pada ekstremitas atas dan gastro esophageal reflux diseae. Tidak didapatkan riwayat trauma.
Nyeri dirasakan mendadak dan seperti terbakar. Pada pemeriksaan fisik didapatkan bengkak, dingin dan
nyeri sentuh. Denyut radial dan ulnar normal. Apakah diagnosis kasus tersebut ?
A. Scleroderma
B. Raynaud’s phenomen
C. Complex regional pain syndrome
D. Acute arterial oclussion
E. Compartement syndrome

39. Perempuan, 10 tahun, dibawa ke poli saraf karena perburukan dari performa di sekolahnya. Dalam
beberapa minggu terakhir ini, pasien tidak bersemangat lagi tentang tugas sekolah, apatis di rumah..
Pemeriksaan didapatkan chorioretinitis, kejang GTCS, ataksia, refleks fisiologis yang meningkat, dan
tanda Babinski bilateral yang positif. Dari EEG diperoleh periodic burst of high voltage slow
waves diikuti dengan rekuren burst suppression pattern. Hasil LCS menunjukkan peningkatan dari fraksi
gamma globulin. Si anak menjadi semakin lethargi dan delirium setelah 2 bulan. Dia tetap dalam kondisi
koma selama beberapa bulan dan akhirnya meninggal. Apakah kemungkinan diagnosis dari pasien
tersebut?
A. Subacute HIV encephalomyelitis (AIDS encephalopathy)
B. Subacute sclerosis panencephalitis (SSPE)
C. Neurocystercosis
D. Bartonema henselae encephalitis
E. HTLV-infection

40. Seorang pria 34 tahun mengalami vertigo dan sakit leher setelah naik roller coaster. Pemeriksaan
menunjukkan anisocoria dengan ptosis ringan di sisi kiri, nystagmus, mengurangi sensasi di sisi kiri wajah,
dan ataksia sisi kiri. Manakah dari berikut ini adalah tes diagnostik terbaik untuk mengevaluasi penyebab
kondisi ini? Diseksi ICA (string and pearl sign)
A. Ultrasonografi karotis
B. Ultrasonografi Doppler transkranial
C. Echocardiogram transthoracic dengan bubble study
D. Angiogram kateter dari arteri servicocerebral
E. Tiime-of-flight MRA dari sirkulus Willis

41. Seorang wanita berusia 49 tahun mengalami kelemahan separuh tubuh kanan, berkembang menjadi
kelemahan keempat anggota gerak selama 2 jam berikutnya. Pada pemeriksaan dapat membuka mata; tidak
dapat berkomunikasi secara verbal. Pasian hanya mampu menggerakkan bola matanya secara vertikal dan
berkedip. Dimanakah letak lesi yang paling mungkin pada kasus tersebut?
A. ventral mesensephalon
B. medulla lateral
C. basal pons bilateral
D. ventral pons bilateral
E. dorsal mesencephalon

42. Seorang wanita usia 22 tahun, instruktur tari, mengeluh nyeri kepala yang rutin timbul saat akhir pekan.
Nyeri hampir selalu terasa di sebelah kanan kepala terutama pada tengah pelipis kanan. Ia dapat
mengetahui nyeri kepalanya akan timbul karena adanya perubahan pada penglihatannya 20-30 menit
sebelum serangan nyeri kepala. Ia melihat seperti kilatan cahaya pada mata kirinya, cahaya lalu berpendar
dan tampak bintik gelap (blind spot). Ketika bintik gelap menghilang maka nyeri kepala mulai timbul.
Biasanya jarang > 1 jam tetapi disertai mual dan muntah. Apakah diagnosis pada pasien ini ?
A. Nyeri kepala klaster
B. Migrain dengan aura
C. Temporal Arteritis (Giant Cell Arteritis)
D. Trigeminal autonomic cephalalgias (TACs)
E. Paroxysmal Hemicranial

42
TO UGM 17 FEBUARI 2022
43. Laki Laki 55 tahun pasca stroke datang ke poli saraf, mengeluh sulit tidur sejak 6 bulan. Pasien tidur baring
jam 21.00, bisa tertidur pukul 23.00, bangun pukul 04.00, tanpa terbangun sepanjang tidurnya. Pemeriksaan
Neurologis normal, Tanda vital normal, obat yang di minum saat ini candesartan 8mg/24jam dan aspilet
80mg/24jam. Depresi (-), Anxiety (-). Day time sleepiness +. Manakah Tata laksana yang paling sesuai
untuk kasus ini yang tidak menyebabkan adiktif ?
A. Pemberian Alprazolam 0,5mg sebelum tidur
B. Kombinasi Cognitive Behavioral Therapy dan lorazepam 2 mg
C. Kombinasi Cognitive Behavioral Therapy dan Zolpidem 5mg
D. Kombinasi Alpazolam 0,5mg dan Cognitive Behavioral Therapy
E. Clobazam 10mg dan Sleep Hygiene
CBT : sebaiknya d terapi dgn farmakologis untuk memeprcept perbaikan, karena CBT lama memperbaiki tidur
Obat yg dihindari utk insomnia :
Amitriptilin, antihistamin, dimenhidrinat, antipsikotik, diaz, klona, lora, alprazolam, muscle relaxant, prpegabalin
dan gabpentin

44. Seorang mahasiswi fakultas kedokteran sejak 4 bulan yang lalu mengeluh sering nyeri kepala. Mahasiswi
tersebut masih bisa beraktifitas tapi setiap 2 hari sekali selalu minum paracetamol yang dibeli di
apotek. Karena keluhan masih muncul, sejak 1 minggu yang lalu, paracetamol diminum setiap hari tapi
justru keluhannya bertambah berat. Apakah tindakan yang paling tepat pada kasus tersebut ? Nyeri MOH
terkait analgetik sederhana langsung d stop dan di obsevasi (kecuali analgetik opioid turunkan bertahap)
A. Paracetamol dihentikan dan diganti dengan asam mefenamat
B. Dosis paracetamol dinaikkan menjadi 2 kali sehari dan dievalusi nyeri kepalanya
C. Paracetamol dihentikan dan di observasi keluhan nyeri kepalanya
D. Paracetamol dilanjutkan ditambah dengan antidepresan trisiklik
E. Meminta pemeriksaan EMG spasmofili dan paracetamol diminum bila nyeri

45. Seorang pria 32 tahun datang ke unit gawat darurat dengan keluhan demam, sakit kepala, dan leher kaku.
Hasil lumbal pungsi menunjukkan CSF pleositosis limfositik, kadar protein sedikit lebih tinggi, dan
kadar glukosa normal. Pewarnaan Gram dan tinta India tidak menunjukkan organisme pada kultur. Suatu
diagnosis dugaan meningitis viral dibuat atas dasar hasil pemeriksaan CSF. Pada catatan medis pasien,
Anda menemukan bahwa ia telah dievaluasi untuk gejala yang sama beberapa waktu yang lalu. Lumbal
pungsi dilakukan pada saat ini menunjukkan temuan CSF mirip dengan yang diamati saat ini. Manakah dari
virus berikut ini kemungkinan besar bertanggung jawab untuk episode berulang ini pasien meningitis viral?
A. VZV
B. HSV-1
C. Echovirus
D. HSV-2
E. Escherechia colli

46. Seorang wanita berusia 43 tahun datang ke poli saraf dengan keluhan nyeri kepala sejak 2 tahun yang lalu,
semakin lama keluhan makin bertambah berat. Keluhan disertai dengan kelemahan anggota gerak kiri.
Tidak terdapat benjolan dibagian tubuh lainnya. Pasien mempunyai riwayat menggunakan KB suntik.
Pasien sudah membawa hasil CT scan kepala dengan gambaran hiperostosis pada os parietalis dan massa
isodens di lobus parietal. Apa terapi defenitif terhadap pasien tersebut?meningioma
A. Radioterapi dan khemoterapi
B. Radioterapi
C. Kemoterapi
D. Reseksi total jaringan tumor
E. Dexametason

47. Laki – laki 45 tahun mendadak mengalami nyeri wajah sisi kanan, rasa tebal-tebal dan kadang-kadang
seperti tersetrum. Berikut ini deskrispi patofisiologi yang paling tepat menggambarkan tipe nyeri tersebut.
A. Penyakit, disfungsi atau lesi sistem saraf perifer

43
TO UGM 17 FEBUARI 2022
B. Penyakit, disfungsi atau lesi sistem saraf central
C. Penyakit, disfungsi atau lesi sistem somatosensorik
D. Penyakit, disfungsi atau lesi nociceptor
E. Penyakit, disfungsi atau lesi neurotransmitter eksitatorik dan inhibitorik

48. Seorang pria berumur 55 tahun, datang ke RS dengan keluhan gerakan berliku-liku dan ireguler pada
tangan dan jari kanan yang semakin bertambah dalam beberapa bulan belakangan ini. Gerakan gemetar
dan perlambatan dalam berjalan juga dijumpai. Riwayat trauma kepala, hipertensi tidak dijumpai.
Berdasarkan skenario kasus diatas, daerah anatomi manakah yang mengalami gangguan pada pasien
tersebut?
A. Nukleus subthalamikus Balismus
B. Nukleus ventrolateral thalami
C. Korteks serebri
D. Putamen
E. Globus palidus

49. Seorang anak laki-laki, usia 18 bulan, dibawa ke poliklinik saraf dengan keluhan sering kaget-kaget sejak
usia 2 bulan. Riwayat tumbuh kembang terlambat. Berat badan saat ini 7 kg. Dari hasil EEG didapatkan
gambaran Hypsarrythmia. Terapi yang paling tepat pada pasien ini adalah? WEST
A. Prednison 4-5 mg/kgBB/hari (10mg 4x 2 minggu)
B. Vigabatrin 250 mg/kgbb/hari (50mg/kgbb hari / 2 dosis)
C. Topiramat 1-2 mg/kgBB/hari
D. ACTH 150 unit/m2/hari  west syndroms
E. Diazepam 0,1 mg/kgBB/hari

50. Seorang laki-laki berusia 35 tahun datang ke poli saraf dengan keluhan kejang pertama kali saat tidur.
Pasien dikeluhkan mendadak tubuh kaku dan kelonjotan seluruh tubuh dengan mata melirik kearah kanan
saat kejang. Kejang berlangsung selama 3 menit dan pasien tidak menyadari telah mengalami kejang.
Pasien diminta melakukan pemeriksaan MRI kepala dengan kontras dan didapatkan hasil hiperintens
bagian frontal pada sekuens FLAIR.

Bagaimanakah Prognosis pasien tersebut?

A. Delesi gen p16 berhubungan dengan luaran yang lebih baik


B. Reseksi tumor yang lebih besar berisiko lebih besar dengan prognosis lebih buruk
C. Delesi kromosom 1p dan 19q menunjukkan prognosis yang buruk
D. Pasien usia muda memiliki kemungkinan bertahan hidup yang lebih baik
E. Mutasi p53 terdapat pada lebih dari 50% kasus

51. Perempuan 20 tahun datang dengan sesak nafas hebat sejak 2 jam SMRS. Lima hari sebelumnya pasien
mengalami batuk pilek dan memperoleh antibiotik azitromisin dari dokter. Riwayat alergi positif. Pasien
telah terdiagnosis miastenia gravis. Pada pemeriksaan fisik diperoleh counting test 10, disfagia dan kesan
kelemahan ke-empat ekstremitas. Pasien direncanakan dirwat di ICU dengan ventilator mekanik. Apakah
tatalaksana medikamentosa yang paling tepat diberikan pada pasien ini? Krisis myasthenia
A. Plasmaferesis
B. IVIG

44
TO UGM 17 FEBUARI 2022
C. Plasmaferesis atau IVIG
D. Prednisone dosistinggi
E. Ganti piridostigmin oral dengan IV

52. Laki-laki 51 tahun sering mengeluh sempoyongan apabila berjalan. Tidak ada keluhan saat duduk
atau berbaring. Tidak ada mual muntah maupun gangguan pendengaran. Dari pemeriksaan penunjang
didapatkan Gula darah sewaktu 234. Apakah diagnosis yang paling mungkin pada kasus tersebut ?
A. Disequilibrium
B. Iskemia vertebrobasiler
C. Vertigo non vestibuler
D. Neuritis vestibularis
E. Vestibuler paroxysmia : Tidak ada mual muntah, frek >10x, <1mnt, 5-5

53. laki-laki, 56 tahun datang ke IGD dengan keluhan kelemahan sisi tubuh kiri secara mendadak sejak 1,5
jam sebelumnya. GCS 15, TD 160/100 mmHg, frekuensi nadi 92x/m, laju pernapasan 20x/m, dan saturasi
oksigen 99%. Pemeriksaan neurologis : paresis N.VII dan N.XII sentral sinistra, hemiparesis sinistra.
Funduskopi dalam batas normal. Nilai NIHSS 7. Pemeriksaan laboratorium awal menunjukkan
Hemoglobin 14,8 g/dl, Leukosit 14.000/mm3, trombosit 240.000 /mm3, gula darah sesaat 150 mg/dl. Nilai
APTT dan PTT dalam batas normal. Pemeriksaan EKG ditemukan pembesaran ventrikel kiri. Pemeriksaan
CT-Scan kepala tidak ditemukan tanda perdarahan.Apa tata laksana khusus awal yang paling tepat untuk
kasus di atas?
A. Aspirin 325 mg
B. Klopidogrel 75 mg dan aspirin 80 mg
C. rTPA 0,9mg/kgBB bolus awal 10% dari dosis total
D. rTPA 0,9mg/kgBB didahului insulin 4 unit subkutan
E. rTPA 0,9mg/kgBB diikuti insulin 4 unit subkutan

54. Laki-laki, 32 tahun datang ke IGD dengan keluhan pusing berputar sejak 2 hari yang lalu. Pusing muncul
mendadak, sangat hebat disertai mual-muntah dan berlangsung terus-menerus. Riwayat batuk-pilek 1
minggu sebelumnya.
Pada pemeriksaan fisik terdapat nistagmus unidireksional ke kanan dan tuli sensorineural kiri
Apakah diagnosis yang paling tepat untuk kasus di atas ? AVS ec Labirinitis
A. Neuritis vestibular kanan
B. Neuritis vestibular kiri  tidak ada ggn pendengaran, bila ada pertimbangkan labirinitis, infark
labirin dan fistula perlimfe
C. Labirintis kanan
D. Labirintis kiri
E. Penyakit meniere

55. Laki laki 40 tahun dengan kelemahan keempat anggota gerak. Pemeriksaan fisik tidak terdapat
gangguan sensibilitas. Didapatkan atrofi otot, Pada pemeriksaan refleks tendon ditemukan
hiperefleks. Pemeriksaan genetik biomolekular apakah yang kemungkinan besar akan positip?
A. Defek genetik pada ALS1  mengecoh
B. Defek genetik pada FUS
C. Defek genetik pada ALS2  mengecoh
D. Defek genetik pada SETX
E. Defek genetik pada SOD1 : 12%

Gen tersering ALS : C9ofr72 > SOD1 > FUS = TDP

45
TO UGM 17 FEBUARI 2022
56. Laki-laki 50 tahun dibawa ke IGD dengan penurunan kesadaran. Pasien memiliki riwayat tumor
intrakranial. Pada pemeriksaan tanda vital diperoleh tekanan darah sistolik 220 mmHg, nadi 50x/menit,
nafas ireguler. Pemeriksaan neurologis dipeoleh GCS E1M2VETT. Tidak didapatkan abnormalitas
pupil dan lateralisasi. Pasien dicurigai mengalami herniasi. Apakah herniasi yang paling mungkin terjadi
pada pasien ini?
A. Herniasi transtentorial
B. Herniasi sentral
C. Herniasi serebellar
D. Herniasi unkus
E. Herniasi eksterna

57. Seorang wanita usia 32 tahun dengan riwayat nyeri kepala sejak 6 bulan lalu, timbul beberapa kali per
minggu, biasanya bertahan sampai pasien tertidur. Nyeri dirasakan konstan dan terutama pada bagian depan
dan belakang kepala. Nyeri tidak behubungan dengan posisi dan cenderung bertambah buruk sepanjang
hari. Photophobia ringan (+). Terdapat spasme dan nyeri tekan pada leher pasien. Apakah diagnosis pada
pasien ini ?
A. Tension Type Headache (TTH) dengan pericranial tenderness
B. Cervicogenic headache : Terbukti ada lesi (klinis, lab, imejing), perbaikan atau perburukaan sesuai lesi
servikalnya, ruang gerak servikal menurun
C. Migrain tanpa aura
D. Nyeri kepala kluster
E. Paroxysmal Hemicranial

58. Seorang laki-laki berusia 63 tahun, datang dengan keluhan sering jatuh dan tidak seimbang bila
berjalan, semakin memberat sejak dua tahun terakhir. Pasien juga mengeluh sering tersandung karena
sulit melihat ke bawah, terasa kaku pada seluruh tubuh.
Pada pemeriksaan neurologis mata tidak bisa melirik kebawah. Pada pemeriksaan postur
didapatkan retroflexed posture. Apakah hasil pemeriksaan penunjang yang paling mungkin ditemukan?
A. Eye of the Tiger sign pada T2 MRI  PKAN, iron acumulation
B. Meouse ear appearance pada T2 MRI (Axial mesensefalon)
C. Atrofi Hipokampus pada T1 MRI  Kemungkinan AD
D. Degenerasi frontotemporal pada T1 MRI  FTD
E. Salt and Pepper appearance pada CT scan  kontusio awal, DAI

59. Perempuan 19 tahun datang ke poliklinik dengan keluhan pandangan dobel sejak 3 minggu yang lalu.
Pandangan dobel saat naik dan turun tangga. Selain itu pasien juga mengeluh sakit kepala yang memberat
sejak 2 bulan yang lalu. Pada pemeriksaan fisik diperoleh pupil bulat isokor 4mm/4mmm, refleks
cahaya langsung dan tak langsung negatif, near reflex positif, retraksi kelopak mata bilateral dan

46
TO UGM 17 FEBUARI 2022
papilledema. CT scan kepala menunjukkan adanya tumor primer. Apakah diagnosis yang paling tepat untuk
kasus teresebut ?
A. Hemangiomacavernosus
B. Rathke Cyst
C. Pineocytoma
D. Meningioma wing sfenoid
E. Kraniofaringioma

60. Seorang perempuan 45 tahun datang ke IGD dalam keadaan gelisah sambil memegang kepalanya. Dia
mengeluh nyeri kepala hebat disertai muntah-muntah. Peristiwa tersebut terjadi sangat mendadak.
Alloanamnesis terganggu oleh situasi pasien yang gelisah. Selang 30 menit kemudian pasien tampak lebih
tenang tetapi tidak responsif terhadap anamnesis maupun pemeriksaan fisik. Keadaan pasien menurun
tajam. Pada pemeriksaan fisik didapatkan 200/120 mmHg, nadi 60x/ menit, suhu 37,80C. Pada
pemeriksaan kedudukan bola mata: kedua bola mata di tengah, dolls eye manuver abnormal, pupil
sangat kecil, refleks cahaya positif, tampak adanya ocular bobbing. Dimanakah letak lesi pada kasus
tersebut ?
A. Talamus
B. Pons  biasanya unreactive, bedakan dengan thalamic pupil, Pinpoin tapi reactive refleks pupil
C. Serebelum
D. Supratentorial
E. Medula oblongata

61. Seorang wanita usia 55 tahun datang ke Poliklinik Saraf dengan keluhan riwayat nyeri kepala sejak 1 tahun,
yang dirasa semakin memberat walaupun sudah meminum obat-obatan nyeri kepala. Nyeri dirasakan di
sebelah kanan, berdenyut, selama beberapa jam, intensitas sedang sampai berat, bertambah dengan
aktivitas, disertai mual tapi tidak muntah, fotofobia dan fonofobia ringan. Sejak 4 bulan terakhir nyeri
dirasakan bertambah sering, timbul rata-rata 20 hari dalam sebulan, sudah mengkonsumsi Paramex dan
Neuralgin, 2-3x hampir setiap hari. Awalnya nyeri berkurang dengan obat tapi lama-lama menjadi
bertambah buruk bila minum obat. Pemeriksaan fisik dan neurologis dalam batas normal. Apakah diagnosis
pada pasien ini ?
A. Nyeri kepala sekunder ec SOL
B. Migrain kronik
C. Paroxysmal Hemicranial
D. Medication overuse headache (MOH)
E. Hemicrania continua

62. Laki-laki , usia 35 tahun dibawa ke IGD dengan keluhan kejang. Kejang mendadak saat pasien di kantor.
Sebelumnya pasien sedang beraktivitas normal. Bentuk kejang lengan kiri kaku, mata melirik ke kiri,
disertai penurunan kesadaran selama 1 (satu) menit. Setelah kejang pasien tampak bingung. Pada
pemeriksaan fisik dan neurologis dalam batas normal. Apakah pemeriksaan penunjang yang tepat untuk
menentukan penyebab kasus tersebut ? kecurigaan lateralisasi kanan, tentukan kausa pakai MRI
A. MRI kepala dengan kontras
B. CT scan kepala dengan kontras
C. Elektroensefalografi
D. Pungsi lumbal
E. Pemeriksaan toksikologi

63. Perempuan, 48 tahun, penderita gangguan jiwa, dikonsulkan ke poli saraf dengan keluhan mulut bergerak-
gerak dan lidah menjulur tidak terkendali sejak 2 hari. Pasien rutin mengkonsumsi obat-obatan neuroleptik
selama 1 tahun. Apakah obat yang paling tepat untuk mengatasi keluhan tersebut ? Tardive diskinesia :
Reserpin dan tetrabenazin; Bisa pake klona, baklo, AV, THP
A. Haloperidol
B. Trihexyphenidyl
C. Bromocriptine
D. Thioridazine
E. MAO inhibitor

47
TO UGM 17 FEBUARI 2022
64. Seorang anak perempuan usia 2 tahun di konsulkan dari TS anak dengan keluhan kesadaran menurun
dialami sejak 3 jam yang lalu. Demam (+) Sebelumnya pasien mengalami kejang fokal sebelumnya.
Terdapat ruam ruam pada kulit kepala dan bokong pasien sejak 5 hari yang lalu. Pemeriksaan CSF di
temukan lymphocytic leukocytosis, red blood cells, and an elevated protein concentration. Apakah
pilihan terapi yang paling tepat untuk pasien diatas? Ensefalitis zoster : 20mg/kgbb /8 jam (60mg) selama
21 hari
A. Acyclovir 60 mg/Kg/BB selama 14 hari
B. Acyclovir 40 mg/kg/BB selama 7 hari
C. Pirimetamin 60 mg/kg/BB selama 7 hari
D. Spiramisin 40 mg/Kg/BB selama 14 hari (TOXO u/ibu hamil
E. Spiramisin 60 mg/Kg?BB selama 14 hari

65. Seorang anak laki-laki berusia 7 tahun dibawa ibunya ke spesialis saraf dengan keluhan sulit belajar di
sekolah. Dari informasi didapatkan anak tidak dapat duduk tenang, sering mondar-mandir di kelas,
mengganggu teman sekelas, tidak bisa bermain dengan temannya. Hal ini mulai tampak sejak anak berusia
5 tahun, akan tetapi belum pernah diobati. Apakah patofisiologi terjadinya gangguan di atas?ADHD
A. Gangguan reuptake serotonin pada post sinaps
B. Kelebihan norepinefrin di lokus seruleus dan defisit dopamine di mesolimbik frontal
C. Kelebihan norepinefrin di lokus seruleus dan defisit dopamine di mesolimbik frontal
D. Kelebihan dopamin di mesolimbik frontal dan defisit serotonin post sinaps
E. Stimulasi norepinefrin yang tidak memadai pada reseptor noradrenergic

66. Perempuan usia 23 tahun dibawa ke IGD karena kejang berulang tanpa disertai perbaikan kesadaran sejak 3
jam lalu. Dua minggu sebelumnya pasien mulai bicara meracau dan tampak bingung; sesekali mengeluh
sakit kepala. Pasien tidak memiliki riwayat kejang atau penyakit lainnya. GCS E3M5V3, TD
150/90mmHg, FN 92x/menit, nafas 24x/menit, suhu 37.7oC. Tanda rangsang meningeal negatif, tidak
tampak lateralisasi. Lab: DPL dbn, elektrolit: Na 132/K 3.8/ Cl 105, GD 137, fungsi ginjal: ur 60, cr 1.5.
EEG menunjukkan gambaran delta brush. Dokter merencanakan plasmaferesis dan memberikan fenitoin
untuk mengatasi bangkitan. Pemberian fenitoin pada kasus di atas adalah atas dasar pertimbangan ...
A. Fenitoin memiliki volume distribusi yang rendah sehingga kadarnya tidak terganggu oleh
plasmaferesis
B. Fenitoin memiliki volume distribusi yang tinggi sehingga kadarnya tidak terganggu oleh plasmaferesis
C. Fenitoin memiliki fraksi terikat protein yang tinggi sehingga kadar dalam darah tidak menurun
pada plasmaferesis

48
TO UGM 17 FEBUARI 2022
D. Fenitoin tidak akan memperburuk hyponatremia
E. Hanya fenitoin yang memiliki sediaan intravena

67. Seorang laki-laki 60 tahun datang ke poliklinik saraf karena menurut istrinya sejak 3 bulan ini sering
melakukan gerakan- gerakan berulang saat tidur, hal ini terjadi beberapa jam sebelum suaminya bangun
tidur. Selama tidur suaminya memukuli guling kemudian terbangun secara tiba-tiba dan berbicara pada
guling tersebut. Saat pagi hari suaminya mengatakan bahwa dia bermimpi diserang oleh penjahat. Terapi
lini pertama untuk kasus ini adalah. RBD : Klonazepam; Lorazepam; Melatonin; pramipeksol
A. Prazosin
B. Lorazepam
C. Trazodone
D. Clomipramin
E. Paroxetine

68. Wanita 49 tahun datang dengan keluhan mendadak hemiplegia kiri dan kelemahan otot – otot wajah atas
dan bawah sisi kanan dan ditemukan Bell sign. Dimana kemungkinan letak lesi nya?
A. Infark pontin kanan
B. Infark pontin kiri
C. Infark midbrain kanan
D. Infark midbrain kiri
E. Infark MCA kanan

69. Perempuan, usia 20 tahun, dibawa ke IGD karena mengalami kesadaran menurun setelah trauma kepala.
Pada CT Scan kepala yang dilakukan 6 jam setelah trauma didapatkan hasil normal. Pemeriksaan fisik =
Sopor, tensi 130/90, nadi 80 x/menit, frekuensi nafas 24x/menit ; suhu 40,0 0C. Pemeriksaan apa lagi yang
perlu dilakukan?
A. CT Scan kepala ulangan setelah 24 jam
B. Foto schedule PA/L
C. Pemeriksaan Transcranial Doppler
D. Angiografi serebral
E. Elektroensefalografi

70. Laki-laki, 21 tahun, aktifis kampus, mengeluh sering mengantuk pada saat mengikuti kuliah. Karena
kesibukan sebagai aktifis kampus, seringkali dia tidur sampai larut malam. Pada pemeriksaan fisik tidak
didapatkan kelainan, dan tidak ada riwayat pemakaian obat-obatan tertentu. Neurotransmitter apakah yang
tidak terlibat pada kondisi mahasiswa tersebut ?
A. Norepinefrin
B. Epinefrin
C. Serotonin
D. Histamine
E. Asetil kolin

71. Bayi laki-laki usia 2.5 bulan mengalami kejang berupa spasme tonik 10 detik, frekuensi serangan 10-300
kali dalam 24 jam. Apakah hasil pemeriksaan EEG yang diharapkan muncul pada saat pasien sedang
mengalami spasme tonik ? spasme infantil  ottahara
A. Spike and double spike wave complex
B. Slow spike wave complex
C. Hypssarythmia
D. Burst suppression asimetris
E. Gambaran EEG normal

Oottahara :
o Inter-Iktal : BURST (berkembang jadi hipsaritmia usia 3-6 bulan)
o Iktal : desinkronasi difus (bloking, atenuasi) dan hilangnya burst suprression

49
TO UGM 17 FEBUARI 2022
72. Seorang pramugari sering mengeluh nyeri di daerah mata sebelah kiri dan disertai pandangan dobel. Dia
juga merasakan nyeri di sekitar mata kiri sampai ke dahi. Saat serangan muncul mendadak dan pramugari
tersebut merasa tidak ada faktor yang memicu munculnya serangan. Apakah diagnosis yang paling
mungkin pada kasus tersebut ?
A. Tolosa-Hunt sindrom : Imaging : lesi granulomatoosa pada Cavernosus biasa meluas hingga FOS
B. Hemikrania paroksismal
C. SUNCT
D. Migrain Optalmoplegik : Imaging tidak ada bukti lesi penyebab
E. Nyeri kepala Kluster

73. Laki-laki, 15 tahun datang dengan keluhan penglihatan kabur mata kiri yang semakin memberat dalam 6
tahun terakhir. Hasil pemeriksaan MRI kepala sebagai berikut

Apakah penyebab penurunan visus pada kasus tersebut ?


A. Sturge-Weber
B. Von Hippel Lindau disease : HARP : Hemangioblastoma, adrenal pheochomacytoma, Renal and
Pancreatic cyst
C. Multiple sclerosis
D. Tuberous sclerosis
E. Optic glioma  Sindrom NF1 : krmosom 17; manifestasi : kafe au lait, optic glioma

74. Perempuan 52 tahun datang ke poliklinik saraf dengan keluhan salah menyebut nama benda. Pasien dapat
berbicara dengan lancar, memahami instruksi, dan mengulang kata. Pada pemeriksaan menunjuk gambar
kursi dan sendok, pasien masing-masing menjawab dursi dan bendok. Apakah diagnosis gangguan bahasa
pada pasien tersebut?
A. Afasia konduktif
B. Afasia anomik
C. Parafasia literal/fonemik
D. Parafasia verbal
E. Parafasia semantic

75. Seorang anak perempuan berusia 5 bulan dibawa ke poli saraf dengan keluhan kejang. Kejang bisa terjadi
10 – 20 kali dalam sehari. Saat kejang pasien akan mengayunkan kepala dan kedua lengannya ke depan
secara mendadak. Pada pemeriksaan elektroensefalografi ditemukan hypsarrhytima. Apakah terapi
farmakologis yang paling tepat untuk kasus tersebut ?west
A. ACTH
B. Carbamazepine
C. Phenobarbital
D. Prednisone
E. Levetiracetam

76. Perempuan, 60 tahun, ke poli saraf dengan keluhan kedua tangan gemetar baik istirahat maupun saat
bekerja sejak 4 tahun. Bila memegang gelas airnya tumpah. Pada awalnya gejala ringan kemudian leher dan

50
TO UGM 17 FEBUARI 2022
kepala ikut bergerak. Bicara jelas dan berjalan tidak kaku. Didapatkan riwayat keluarga dengan keluhan
serupa. Pada pemeriksaan fisik didapatkan cogwheel fenomena (+), pemeriksaan finger to nose dan tes
disdiadokokinesia normal. Apakah diagnosis yang paling mungkin untuk pasien tersebut ?
A. Parkinson diseases awal
B. Trombosis Mesencefalon
C. Tremor Cerebellar
D. Essential Tremor
E. Rubral Tremor

77. Laki-laki 24 tahun mengalami kecelakaan lalu lintas, dengan crush injury pada tungkai kiri. Dokter ortoped
memutuskan melakukan amputasi. Post amputasi pasien mengeluhkan nyeri seperti kesemutan, tebal dan
tersetrum di area yang telah teramputasi. Apakah tindakan yang dapat dilakukan oleh dokter untuk
mencegah kejadian tersebut?
A. Edukasi
B. Psikoterapi
C. Roboratia/neuroprotektan
D. Analgesik preemptif : analgesik intravena atau intradural
E. OPIOID

78. Seorang wanita berusia 69 tahun dengan riwayat stroke hemisfer kanan, hipertensi, hiperlipidemia, dan
diabetes datang dengan gejala berulang berupa kebas di tubuh sisi kiri. Ultrasonografi arteri karotis
dilakukan untuk menentukan apakah prosedur endarterektomi diindikasikan. Manakah yang benar dari
pernyataan berikut?
A. Stenosis karotis kanan yang lebih dari 70% pada pasien ini harus diobati secara medis
B. Stenosis karotis kiri kurang dari 50% harus ditangani dengan pembedahan  BMT
C. Tindakan bedah direkomendasikan jika oklusi karotis kanan terdeteksi  BMT
D. Stenosis karotis kanan kurang dari 50% akan bermanfaat jika dilakukan pembedahan  BMT
E. Endarterektomi untuk stenosis karotid simtomatik 50% hingga 69% kurang menguntungkan
pada wanita dibandingkan dengan pria

79. Laki-laki, 55 tahun, berat badan 40 kg, masuk rumah sakit dengan keluhan utama penurunan kesadaran
sejak 7 hari. Ada riwayat demam tidak tinggi & nyeri kepala sejak 14 hari yang lalu. Ditemukan GCS 7.
Terdapat kaku kuduk. Terdapat hemiparesis kanan dengan tanda Babinsky positif. Hasil pemeriksaan PCR
TB cairan serebrospinalis (+).HIV (-). Dibuat diagnosis kerja meningitis tuberculosis. Bagaimanakah
penatalaksanaan pada kasus tersebut?
A. OAT dan prednisone dosis 3 mg/kgBB/hari (neuroinfeksi 2011)
B. OAT dan prednisone dosis 1 mg/kgBB/hari
C. OAT dan dexamethasone dosis 20 mg/kgBB/hari
D. OAT dan dexamethasone dosis2 mg/kgBB/hari
E. OAT dan dexamethasone dosis5 mg/kgBB/hari

80. Seorang wanita 33 tahun datang ke poli saraf dengan keluhan nyeri di seluruh bagian kepala, keluhan
hilang timbul dan berlangsung lebih dari 30 menit tapi tidak mengganggu aktifitasnya. Tidak ada mual
muntah. Dari anamnesis didapatkan 10 hari yang lalu pasien pernah dipukul suaminya sampai
bengkak di wajah. Dari vital sign tekanan darah 150/90 mmhg. Apakah diagnosis yang paling mungkin
pada kasus tersebut ?
A. Tension Tipe Headache
B. Nyeri kepala servikogenik
C. Psikosomatis
D. Psikosomatis
E. Nyeri kepala akut pasca trauma

81. Laki-laki, 55 tahun datang ke IGD dengan keluhan mulut mencong ke kanan mendadak sejak 6 jam yang
lalu. Pasien masih dapat menutup mata kirinya. Pada pemeriksaan fisik didapatkan GCS 14, pupil bulat
isokor 3mm/3mm, refleks cahaya positif, kesan deviation konjugee ke kiri, hemiparese sinistra, gangguan

51
TO UGM 17 FEBUARI 2022
proprioseptif sinistra dan dismetria pada sisi kanan. Mekanisme apakah yang paling mungkin mendasari
kelainan di atas?
A. Oklusi arteri serebeli anterior inferior (AICA) kanan
B. Oklusi arteri serebeli anterior inferior (AICA) kiri
C. Oklusi arteri serebeli posterior inferior (PICA) kiri
D. Oklusi arteri paramedian atas kanan (upper pons - mesensefalon)
E. Oklusi arteri paramedian atas kiri

82. Perempuan usia 59 tahun diantar oleh keluarganya ke Poliklinik Saraf karena mudah lupa sejak 6 bulan
yang lalu. Dari anamnesis diketahui pasien mengalami halusinasi visual dan gangguan kognisi yang
fluktuatif. Pasien juga mengalami keluhan gemetar, langkah kecil-kecil sejak 1 tahun yang lalu. Apakah
diagnosis yang paling mungkin pada kasus tersebut?
A. Demensia lewy bodies
B. Demensia frontotemporal
C. Demensia penyakit Parkinson
D. Demensia senilis
E. Demensia campuran

83. Laki-laki, 14 tahun mengalami serangan menghentak pada bahunya tiba-tiba tanpa disengaja. Serangan ini
dialami sejak kurang lebih 2 tahun terakhir. Belum diobati dan makin sering. Biasa terjadi beberapa menit
setelah bangun tidur di pagi hari. Lama serangan hanya 1 detik dan tidak berulang. Saat serangan pasien
sadar. Pemeriksaan EEG menunjukkan gambaran polispike and wave. Apakah terapi OAE yang paling
tepat untuk kasus tersebut ?JME
A. Carbamazepine
B. Fenitoin
C. Valproat (lini 1) topi,valpro lamo, leve)
D. Clobazam
E. Topiramat

84. Laki-laki 62 tahun datang ke poli saraf dengan keluhan badan gemetar sejak 1 tahun yang lalu. Awalnya
gemetar hanya dirasakan pada tangan kanan. Gemetar terjadi saat istirahat. Penderita juga mulai
mengeluhkan kaku pada seluruh tubuh. Pada pemeriksaan fisik tampak ekpresi wajah datar. Pada
pemeriksaan neurologis ditemukan tremor istirahat dengan frekuensi 4-7 siklus per detik. Rigiditas tipe
gigi roda ditemukan pada kedua tangan. Apa temuan patologi utama di substansia nigra yang dapat
ditemukan pada kasus di atas?
A. Badan inklusi intrasitoplasmik
B. Badan inklusi intranuklear
C. Neurofibrillary tangles
D. Plak amiloid.
E. Plak senilis

85. Perempuan, 56 tahun datang dengan keluhan kejang seluruh tubuh menghentak-hentak pada saat malam
hari. Kejang baru pertama kali muncul. Pada pemeriksaan MRI kepala didapatkan hasil sebagai berikut

52
TO UGM 17 FEBUARI 2022
.
Apakah penyebab kejang pada kasus tersebut ?
A. Temporal lobe glioma
B. Heterotopia
C. Cortical scarring
D. Mesial temporal sclerosis
E. Herpes simplex encephalitis

86. Perempuan, 48 tahun datang ke poli saraf karena pusing berputar disertai rasa mual dan muntah. Rasa
pusing ini timbul mendadak pagi ini dan berkurang jika pasien berbaring. Pada pemeriksaan Fisik
didapatkan : ketika mata melirik ke kanan timbul nistagmus OD dan gangguan adduksi OS. Ketika mata
melirik ke kiri : normal. Dimanakah letak lesi kelainan tersebut ?
A. N. Vestibularis kanan
B. N. Vestibularis kiri
C. Pons Kiri
D. Pons kanan
E. Serebelum

87. Seorang laki-laki 30 tahun sering mengalami kejang lobus temporalis. Pemeriksaan EEG mendukung
adanya gelombang abnormalitas dan angiografi menunjukkan gambar di bawah ini.

Apakah Tindakan neurointervensi yang paling tepat untuk hasil angiografi tersebut ? AVM
A. Stenting
B. Coiling
C. Embolisasi
D. Trombolisis

53
TO UGM 17 FEBUARI 2022
E. Trombektomi

88. Perhatikan gambar dibawah ini!

Hasil gambaran MRI-DWI dan MRA di atas menunjukkan kelainan pembuluh darah apa ?
A. Stenosis arteri serebri media
B. Stenosis arteri karotis interna
C. Stenosis arteri serebri anterior
D. Stenosis arteri karotis eksterna
E. Stenosis arteri karotis kommunis
Soal UNAS 2022

89. Seorang laki-laki 40 tahun mengeluh saat mencukur jenggot dan menggosok gigi, tiba-tiba pipi sebelah
kanan terasa nyeri. Nyeri dirasakan seperti tersengat listrik yang muncul dalam beberapa detik tapi sering
berulang. Dari pemeriksaan status neurologi tidak didapatkan kelainan, hanya dari pemeriksaan darah rutin
didapatkan HB 9,3 gr%. Apakah Terapi pilihan pertama pada kasus tersebut ? TN
A. Tramadol
B. Methylprednisolon
C. Carbamazepin (es anemia aplastik, SSJ, hepatotoksik)
D. Amitriptilin
E. Mecobalamin

90. Seorang laki-laki 52 tahun dikonsulkan dari bagian penyakit dalam Karena mengeluhkan pusing berputar.
Dari anamnesis didapatkan bahwa dia adalah seorang penderita diabetes yang mengkonsumsi banyak obat.
Manakah obat dibawah ini yang dapat menyebabkan labirintitis toksik?
A. Penicilline
B. Promethazine
C. Asam asetilsalisilat
D. Dimenhidrinat
E. Gentamycin

91. Seorang laki-laki 32 tahun, PNS, datang ke poli saraf dengan keluhan nyeri pinggang yang timbul saat
dirinya mengambil bolpoint yang jatuh 2 hari sebelum periksa. Nyeri dirasakan terus menerus namun
tidak menjalar sampai tungkai dan tidak disertai kesemutan. Pasien kemudian minum obat pereda nyeri
tapi nyeri hanya berkurang sedikit. Apakah Tindakan awal yang paling tepat untuk kasus tersebut ?
A. meminta pemeriksaan x foto vertebra lumbosakral AP/lateral
B. memberi terapi analgetik plus muscle relaxan
C. memberi edukasi mengenai aktivitas yang tidak memperberat nyeri
D. meminta pemeriksaan darah rutin dan fungsi ginjal
E. memberi terapi analgetik plus antidepresan

54
TO UGM 17 FEBUARI 2022
- Myofasial Pain : Ada trigger point, terus digunakan dengan muscle relaxnt
- LBP : Pemeriksaan penunjang perlu dilakukan jika nyeri tidak membaik dalam 3 bulan atau
ditemukan tanda defisit neurologis pada pemeriksaan fisik atau ditemukan tanda bahaya (red flag)

92. Laki-laki, usia 50 tahun datang ke poliklinik dengan keluhan pandangan gelap mata kanan mendadak
sejak 5 hari yang lalu. Nyeri daerah mata disangkal. Pasien memiliki riwayat hipertensi dengan obat
hipertensi baru dinaikan dosisnya. Pada pemeriksaan neurooftalmologi didapatkan.

N. II kiri, skotoma sentral,


edema papil  Dx. NA-AION

Lesi dimanakah yang mendasari gangguan penglihatan tersebut di atas ?


A. Arteri siliaris posterior
B. Arteri siliaris anterior
C. Arteri retina sentralis
D. Arteri oftalmika
E. Vena retina sentralis

93. Laki laki 40 tahun dengan kelemahan keempat anggota gerak. Pemeriksaan fisik tidak terdapat
gangguan sensibilitas. Didapatkan atrofi otot, Pada pemeriksaan refleks tendon ditemukan hiperefleksi.
Pathophysiologi apa yang mendasari penyakit yang tipikal tersebut. ALS
A. Kerusakan alpha motorneuron di medula spinal
B. Terjadinya demyelinisasi saraf tepi  GBS, CIDP
C. Penurunan massa otot  Myopathy
D. Degenerasi axonal saraf tepi
E. Degenerasi myofibril

94. Perempuan, 23 tahun, datang ke UGD dengan keluhan penurunan kesadaran sejak 3 hari sebelumnya. Ada
riwayat demam tinggi yang naik-turun sejak 5 hari dan keringat dingin. Pasien sedang hamil 10 minggu.
Hasil pemeriksaan tetes tebal ditemukan Plasmodium falciparum bentuk cincin. Apa terapi yang paling
tepat untuk kasus ini?
A. Chloroquine??
B. Artesunate
C. Artemeter
D. Primakuin
E. Kina

95. Perempuan, 38 tahun datang ke poli saraf dengan keluhan pusing berputar 2 hari yang lalu. Saat ini
pasien tidak ada keluhan tapi khawatir keluhan akan kambuh lagi. Pasien minta obat untuk pencegahan
penyakitnya. Pemeriksaan fisik normal. Riwayat penyakit dahulu (-). Apakah Pilihan terapi yang paling
tepat untuk kasus tersebut ?
A. Betahistin 8 mg 3 kali sehari
B. Flunarizine 5 mg 2 kali sehari
C. Dimenhirinat 50 mg 3 kali sehari

55
TO UGM 17 FEBUARI 2022
D. Diazepam 5 mg 1 kali sehari (malam)
E. Tidak diberi obat sama sekali

96. Anak perempuan 8 tahun sering terjatuh. Dokter anak hanya menemukan skoliosis ringan. Pada usia 10, ia
menunjukkan masalah bicara, kesulitan berjalan, disartria, dan tremor pada tangan bilateral. Pada
pemeriksaan neurologis, nistagmus horizontal bilateral. Tanda Romberg sangat positif. Riwayat yang sama
pada bibinya. Pasien didiagnosa sebagai Friedreich’s ataxia. Dimanakah kelainan pada pasien tersebut?
A. Mutasi kromososm 9q13
B. Mutasi kromosom 11q22,23
C. Mutasi kromosom Xq21-22
D. Mutasi kromosom 1q22-23
E. Mutasi kromosom 5q13

97. Seorang wanita, 36 tahun, mengalami vertigo episodic dalam 1 tahun ini, disertai mual muntah,
mendenging pada kedua telinga, rasa penuh pada telinga dan penurunan pendengaran. Pasien diusulkan
pemeriksaan audiometri dengan dugaan meniere disease. Kemungkinan hasil audiometri pada pasien
tersebut adalah :
A. Tuli konduksi pada nada rendah
B. Tuli konduksi pada nada tinggi
C. Tuli sensorik pada nada rendah
D. Tuli sensorik pada nada tinggi
E. Tuli campuran

98. Perempuan usia 23 tahun dibawa ke IGD karena kejang berulang tanpa disertai perbaikan kesadaran sejak 3
jam lalu. Dua minggu sebelumnya pasien mulai bicara meracau dan tampak bingung; sesekali mengeluh
sakit kepala. Pasien tidak memiliki riwayat kejang atau penyakit lainnya. GCS E3M5V3, TD
150/90mmHg, FN 92x/menit, nafas 24x/menit, suhu 37.7 oC. Tanda rangsang meningeal negatif, tidak
tampak lateralisasi. Lab: DPL dbn, elektrolit: Na 128/K 3.8/ Cl 105, GD 137, fungsi ginjal: ur 60, cr 1.5.
Diagnosis klinis pasien tersebut adalah ...
A. Focal seizure with impaired awareness
B. Psikosis
C. Progressive symptomatic seizure
D. Psikogenik non epileptic seizure
E. Epilepsi simtomatik

99. Seorang wanita 39 tahun dengan SAH grade IV hari ke lima dengan GCS E1/V1/M1, dengan hasil TCD
seperti dibawah.

Apakah diagnosis yang paling tepat pada pasien tersebut?


A. Focal vasospasm
B. General vasospasm
C. Global Ischemia
D. Brain death
E. Vasospasm resolution

56
TO UGM 17 FEBUARI 2022
100.Seorang laki-laki , usia 65 tahun, sejak 2 hari mengeluh adanya ruam vesicle dan nyeri panas intensitas
sedang berat, di bagian perut sisi kanan setinggi pusar, kemudian dokter menegakkan diagnosis acute
herpes zoster dan memberikan terapi antiviral yang sesuai. Dokter mengedukasi prognosis terkait
kemungkinan munculnya neuralgia herpetic maupun pasca herpes.
Apa tatalaksana tambahan yang perlu diberikan pada pasien tersebut?
A. Anticonvulsant
B. Steroid intravena
C. Opioid long acting
D. Lidocain patch/topical
E. Non Steroidal Anti Inflammatory Drugs

MCQ JULI 2021


NEUROBEHAVIOR

1. Seorang pasien 54 th dengan keluhan kesulitan memahami pembicaraan secara tiba-


tiba dari pemeriksaan didapatkan pasien dapat berbicara lancar. Tidak dapat
memahami isi pembicaraan namun dapat mengulangi apa yang diperintahkan kepada
pasien REPETISI BAIK. Gangguan bahasa yang terjadi pada pasien tsb?
A. Afasia Wernicke
B. Afasia transkortikal motorik
C. Afasia transkortikal sensorik
D. Afasia global
E. Afasia anomik

2. Laki-laki 55 tahun dengan keluhan anggota gerak kanan mendadak lemah sejak bangun
tidur pagi hari. Pasien sadar dapat berkomunikasi dan memahami pembicaraan orang
lain. Pemfis didapatkan pasien tidak memahami perkataan dan dapat mengulang kata-
kata. Dimanakah topis kelainan letak lesi pada kasus tersebut?
A. Lobus frontal inferolateraL ------- broca/motorik

57
TO UGM 17 FEBUARI 2022
b. Fasikulus arkuata : afasia konduksi (paham dan ngomong, tp gak bisa ngulang)
c. Gyrus cinguli ------ transkortikal motorik
d. Girus angularis  afasia transkortikal sensorik
e. Lobus temporal superior ---- afasia Wernicke/sensorik

3. Seorang laki-laki 65 tahun sering lupa sejak 2 tahun yang lalu. Pasien mulai sering lupa
arah tempat, sulit memakai baju sendiri dan sulit tidur di malam hari. Oleh dokter di
diagnosa Alzhemier berat dan telah diterapi. Apakah terapi yang paling tepat pada kasus
tersebut ?
A. Inhibitor kolinesterase + NMDA antagonis
B. MAO inhibitor + AchEI (MAO inhibitor : isokarbo oxazid)
C. AchEI
D. NMDA antagonis
E. Gingkogiloba

Ringan sedang : AchEI (DRG : donepezil, rivastigmin, galantamin)


Sedang berat : AchEI + NMDA antagonis

4. Seorang Laki-laki 65 th ke poli dengan sering lupa terutama apa saja yang baru dilakukan
dan sulit mengingat kegiatan yang sebelumnya telah direncanakan. Pasien masih bisa
menyetir dan mengurus diri dan kegiatan rumah tangga. Tidak ada riw. HT dan DM.
Apakah terapi yang paling tepat pada kasus tersebut? MMSE 25
a. Psikoterapi
b. Donepezil
c. Fisioterapi
d. Galantamin
e. stimulasi kognitif
MMSE ringan : (21-26) 24-29 MCI (terapi kognitif dan interpersonal)
MMSE sedang : (15-20) 18-23 (AchEI)
MMSE Sedang-berat : (10-14) 0-17 (AcheI + NMDA antagonis)
MMSE Berat : (<9)

5. Pasien laki-laki, usia 50 th dengan HIV (+), gangguan kognitif (+), keseharian dan pekerjaan
terganggu sampai dikeluarkan dari tempat kerja  HIV-associated dementia

6. Obat pada HIV yang mengakibatkan gangguan kognitif  efavirenz

7. Pasien laki-laki dengan keram-keram simetris distal, pasien HIV (+). Apa pemeriksaan gold
standar  Biopsi kulit

8. Pasien dengan keluhan mudah lupa dan Parkinson. Obat yang memperburuk gejala motoriknya
a. Memantin
b. Donepezil

58
TO UGM 17 FEBUARI 2022
c. Rivastigmin
d. Entacapone (COMT inhibitor)
e. Selegiline?? (MAO inhibitor)

NEUROOFTALMOLOGI

9. Seorang wanita 56 tahun dengan riw. HT dan DM mengeluhkan mata kanan kabur.
Beberapa hari kemudian pasien merasa gelap saat melihat ke atas dan ke bawah yang
makin memburuk. Hasil pemeriksaan RC didapatkan papilarry defect. Hasil funduskopi
didapatkan “flame shaped hemorrage”. Diagnosa pada kasus diatas ?
A. Anterior ischemic optic neuropathy (amaurosis fugax, ↑LED, ↑CRP) di a. ciliaris
posterior
B. Idiopatik intrakranial Hipertension
C. PAPILEDEMA due tO increased intracranial pressure
D. Optic neuritis
E. Stroke hemmoraghe
Flame shape hemoorrhage : retinopati HT

10. Gambar lapangan pandang, Letak lesi  traktus optikus

11. Pasien disinari kanan, tidak ada respon, sinari kiri, dua-dua respon. Dimanakah letak lesinya 
RAPD kanan

12. Epilepsi dengan deviasi konjugate ke kiri. Dimanakah letak lesinya  lesi iritatif kanan (deviasi
konjugate kontralateral lesi)

13. Pasien dengan GDS (geriatri depression scale) = 11. Kategori  sedang
Normal : 0-9
Mild : 10-19
Severe : 20-30

59
TO UGM 17 FEBUARI 2022
NEROOTOLOGI

14. Seorang perempuan datang dengan keluhan pusing berputar dengan durasi < 1 menit
ketika pasien melakukan perubahan posisi dari duduk kemudian telentang dan saat
pasien membungkuk ke depan, pasien didiagnosa mengalami BPPV. Saat dilakukan
manuver halpike. Manakah pernyataan yang benar dibawah ini?
A. Nistagmus upbeat torsional ke sisi lesi. Knp ??
B. Nistagmus upbeat torsional kontralateral lesi – (VOR) HRS NYA DOWN BEAT BR KE
KONTRA LAT
C. Nistagmus geotropik  lat / hori
D. Nistagmus apogeotropik  lat / hori
E. Nistagmus unidireksional kontralateral lesi

15. Seorang perempuan usia 48 tahun datang dengan keluhan pusing berputar dengan
durasi < 1 menit, pusing bertambah jika pasien melakukan perubahan posisi dari duduk,
kemudian telentang dan saat pasien membungkuk ke depan , ketika dilakukan manuver
dix halpike, ditemukan nistagmus perifer. Apakah penatalaksanaan yang paling tepat
pada kasus tersebut?
a. Manuver epley dari sisi kanan  dimulai menoleh ke sisi sakit. Baring ke sisi sakit
b. Manuver epley dari sisi kiri
c. Manuver semont dari sisi kanan  dimulai menoleh sisi sehat. Baring ke sisi sakit
d. Manuver brand Daroff dari sisi kanan  dimulai menoleh ke kiri, baring ke kanan
(tidak pakai sisi sakit sehat)
e. Manuver lampert dari sisi kiri  baring dulu, menoleh sisi sehat, baru putar badan

60
TO UGM 17 FEBUARI 2022
61
TO UGM 17 FEBUARI 2022
16. Seorang perempuan 37 tahun mengeluh pusing berputar yang hilang timbul, pasien
tidak memperhatikan secara jelas ??. OS mengaku badan terasa panas, mual dan
beberapa kali muntah. Tidak ada gangguan pendengaran. Tidak ada defisit neurologi.
Pasien dilakukan pemeriksaan neurootologi. Apakah hasil pemeriksaan neurootologi
yang paling mungkin muncul pada kasus tersebut ? EVS  Dix hallpike
dx. kasus perifer BPPV
a. Head impulse test negatif  sentral
b. Nistagmus unidireksional  perifer
c. Skew deviation positif  sentral
d. Romberg dipertajam buka mata terjatuh  sentral

62
TO UGM 17 FEBUARI 2022
e. Finger to nose terdapat dismetria  sentral

17. Pasien perempuan diantar ke IGD dengan keluhan pusing berputar. Riw flu 1 minggu
sebelumnya. Nistagmus horizontal rotational ke kiri. Apakah diagnosis pasien  Neuritis
vestibular kanan.
- HINTS : HI positif (sakadik +), N unidireksional ke kiri; Skew negatif
- Kanal horizontal. Nistagmus nya kontralat lesi.
- Terapi : dimenhidrinat, MP ( 100mg/hari –3 hari) tap off 20mg/3 hr selama 3 minggu
- Klo pake prednisone : 2x20mg . 10-14hr

18. Derajat tumor CPA. Gambar CT scan menekan batang otak. Berapa grading pasien tersebut 
T4

Derajat KOOS (Pakai stadium)


Stadium 1  didalam meatus
Stadium 2  didalam meatus sebagian keluar ke sisterna pontine
Stadium 3  keluar ke sisterna pontine hampir mengenai pons
Stadium 4  menekan batang otak

Derajat SAMII (pakai T)


T1 : INTRAMEATAL
T2 : EKSTRAMEATAL
T3 : A. EKSTRAMEATAL MENGISI CPA BELUM SAMPAI PONS;

B. EKSTRAMEATAL MENGISI CPA MENCAPAI PONS

T4 : A. KOMPRESI PONS; B : KOMPRESI PARAH, DISERTAI DISLOKASI BATANG OTAK DAN


KOMPRESI VENTRIKEL 4

T5 : Midline shift

19. Terapi pada BPPV. Manuver brand daroft  5 siklus, pagi dan sore selama 3 minggu  CUIT
HAL 32.
20. Pasien perempuan dengan keluhan pusing dan nyeri pada bahu dan leher. Apakah diagnosis
pasien tersebut  servikal dizziness  CUIT HAL 38
21. Apakah pemeriksaan pada servikal dizziness  uji torsi leher -> positif bila muncul vertigo/dizzi
atau nistagmus

63
TO UGM 17 FEBUARI 2022
Servikal dizziness : Pusing yg disebabkan pergerakan leher, disertai nyeri leher dan keterbatasan ROM
leher, biasanya disebabkan ggn proprioseptif (shg didapatkan ggn ketidakseimbangan saat berjalan)

22. Pasien dengan keluhan pusing dan berkurang jika duduk. Apakah mekanisme yang mendasari 
Gangguan propioseptif atau cerebellum
23. Pasien dengan nyeri dan kebas pada kedua kaki, riw. DM (+). Apakah mekanisme nyeri pasien 
Sensitisasi katekolamin (sensitisasi perifer)

Mekanisme perifer meliputi:


1). Aktivitas ektopik,
2). Sensitisasi nosiseptor,
3). Interaksi abnormal antar serabut saraf,
4). Sensitivitas terhadap katekolamin

mekanisme sentral meliputi:


1). Sensitisasi sentral,
2). Reorganisasi sentral,
3). Hilangnya kontrol inhibisi.

24. Mekanisme neuritis vestibular  inflamasi nervus vestibular

VASKULAR

25. Laki2 52 th, ke poli saraf dengan keluhan mata kanan bengkak, kemerahan disertai nyeri
setelah KLL. Pada pemeriksaan dijumpai proptosis,, kemosis, burit orbital dan
opthalmoplegia.. Tidak ada kelainan motorik. DX . CCF
Apakah pemeriksaan penunjang yang tepat untuk menentukan diagnosa tsb ?
A. VEP
B. TCD
C. CT Scan cerebral
D. DWI cerebral
E. Angiografi

26. Pria 38 th ke UGD dengan keluhan kejang kelojotan , 1 jam yang lalu. Kejang terjadi
selama 5 menit, saat kejang pasien tidak sadarkan diri, setelah kejang pasien tampak
mengantuk. OS mengatakan nyeri kepala hebat mendadak saat beraktivitas, mual (-),
muntah (-), pasien menyangkal adanya HT, DM. Dari pemeriksaan, GCS E3M5V3, kaku
kuduk ringan. NN cranialis dalam batas normal, hemiparese kiri. Berapakah derajat
klinis berdasarkan hunt and Hess :
Point yg diperiksa :
a. Grade I Sakit kepala ringan
b. Grade II Sakit kepala sedang defisit Nn. Craniales
c. Grade III Mengantuk
d. Grade IV Stupor
e. Grade V Koma
64
TO UGM 17 FEBUARI 2022
27. Wanita 64 tahun datang ke UGD dengan keluhan kelemahan separuh tubuh kiri sejak 2
jam sebelumnya. Skor NIHSS 16. TD 170/90 mmHg. Pasien menderita infark miokard 4
bulan sebelumnya dan menggunakan obat antikoagulan dengan INR >1.8 menjalani
operasi usus buntu 3 minggu sebelumnya. CT scan kepala menunjukkan tidak ada
perdarahan. Manakah pernyataan di bwah ini kriteria eksklusi untuk rTPA
a. Skor NIHSS 16  inklusi
b. Menderita infark miokard 4 bulan sebelumnya  inklusi tp hr nya 3 bln
c. TD 170/90 mmHg  inklusi
d. INR > 1,8  eksklusi - hrs nya sebenernya <1,7
e. Menjalani operasi usus buntu 3 minggu sebelumnya.--> inklusi . 2 minggu klo
mayor

28. Apakah diagnosis yang paling sesuai untuk kasus di atas?


A. Pseudotumor Cerebri

B. Meningioma

C. Venous Sinus Thrombosis

D. Preeklampsia

E. Neuritis optik

29. Wanita hamil 12 minggu dengan gambaran CT scan (ada gambar CT) venous sinus trombosis.
Terapi yang diberikan  LMWH

65
TO UGM 17 FEBUARI 2022
30. Laki-laki, 40 tahun, datang ke IGD dengan keluhan nyeri di sekita mata kiri sejak 3
minggu yang lalu. Nyeri dirasakan kambuh-kambuhan dan disertai penglihatan ganda
terutama jika melirik ke arah kiri. Seminggu yang lalu, pasien sempat berobat ke dokter
umum dan keluhan membaik dengan pemberian kortikosteroid. Tidak ada riwayat
trauma, diabetes, hipertensi, tumor, maupun nyeri kepala lain sebelumnya.
Hasil pemeriksaan MRI kepala sbb:

Apakah penyebab rasa nyeri pada kasus tersebut? DX. Tolosa hunt syndrome

A. Kompresi pada fissura orbitalis superior


B. Proses inflamasi yang menyebabkan lesi pada N.V-1
C. Proses inflamasi non-spesifik pada sinus cavernosus
D. Kerusakan N.III, IV, dan VI akibat inflamasi granulomatosa
E. Lesi serabut saraf simpatis pada arteri karotis interna segmen kavernosa

31. Pasien Laki-laki usia 60 tahun datang dengan keluhan penglihatan ganda, gangguan pengecapan,
lemah tubuh sebelah kiri mendadak. Pada pemeriksaan fisik ditemukan parese N VI kanan,
parese N VII Perifer kanan disertai adanya gangguan pengecapan 2/3 anterior lidah dan
hemiparese sinistra. Apakah sindrome yang sesuai dengan klinis diatas?
A) Syndrome Wallenberg  lateral medullary syndrom
B) Syndrome Foville  ada conjugate palsy ki ka . topis nya di pons dorsal .N 6 kiri dan kanan
kena karena kena dinukleus nya.
C) Syndrome Benedict  mesensefalon. Paramedian midbrain -> ada tremor
D) Syndrome Milliard Gubler  ventral pons. Kena 6,7 . hanya serabut saraf 6 aja. 1 mata
E) Syndrome Weber  basal mesensefalon  parese n3 ipsi lateral. hemiparese

32. Pasien Laki-laki usia 60 tahun datang dengan keluhan penglihatan ganda, gangguan pengecapan,
lemah tubuh sebelah kiri mendadak. Pada pemeriksaan fisik ditemukan parese N VI kanan,
parese N VII Perifer kanan disertai adanya gangguan pengecapan 2/3 anterior lidah
( sepertiganya NIX) dan hemiparese sinistra. Dimanakah letak lesinya?  pons letak nukleusnya
N.V,VI,VII, VIII.

33. Pasien dengan keluhan ptosis dan parese n. III. Dimanakah letak lesi  A. Communicans
posterior??

66
TO UGM 17 FEBUARI 2022
34. Pasien dengan laki-laki, merokok. Tidak ada riw. DM dan HT. rencana rTPA. Apakah pilihan
pemeriksaan selanjutnya  1. Ct scan kepala 2. pemeriksaan lab

A. GDS dan GD2PP


B. PT, APTT, INR
C.
35. Pasien laki-laki, usia 70 th, dengan TIA berulang, stenosis 90% pada bifurcatio A. Carotid.
Penatalaksanaan selanjutnya  CEA??  CAS (
CEA ( Carotid end aerteretomy )
 Diatas 70 tahun, dilakukan CEA bila symptomatic bila stenosis 50-69%
Asimptomatik jika 70-99%
Komplikasi infark miocard
Pria lebih bermanfaat daripada perempuan

CAS ( carotid angioplasthy stenting )


 usia 18 tahun, simtomatik bila stenosis diatas 50%
Asimptomatik diaatas 70%
komplikasi infark cerebri

kalau ada stenosis ICA intracranial dan arteri vertebralis udah diberikan obat masih stenosis
masih berat, itu pemilihannya stenting

36. Pasien anak-anak dengan nidus 4 cm (2), daerah korteks motorik (1), drainase vena dalam (1)
Berapakah skor spletzer Martin?  4

37. Perempuan wanita datang dengan keluhan nyeri kepala dan dilakukan angiografi didapatkan
aneurisma tidak ruptur. Pasien dengan hipertensi berobat rutin. Riwayat ibunya dengan
aneurisma. Apakah tatalaksana pada pasien  coiling (harus d terapi karena faktor risiko tinggi
untuk ruptur) Kalau udah rupture tetap dicoiling agar mencegah rebleeding

67
TO UGM 17 FEBUARI 2022
38. Pasien menyadari kalau yang dilihatnya hanya halusinasi,. Pasien mengaku ada masalah pada
penglihatannya. Diagnosis  Charless bonnet syndrome
Menyadari kalau dia buta dan mengakui kalau ada halusinasi

39. Pasien menyangkal masalah penglihatannya  Anton syndrome dia suka nyangkal, padahal
sering nabrak, cortical blindness dia

INFEKSI

40. Kejang, lesi pada imaging  Dx neurosistiserkosis Pemilihan terapi kejangnya? 


simetidine (menaikkan efek praziquantel) menghambat peningkatan metabolisme
Neurosistiserkosis

Praziquantel diberikannya 50mg/kgbb dibagi menjadi 3 dosis selama 14 hari ( OAE akan
meningkatkan metabolisme prazy, jadinya efeknya sedikit, jadinya diberikan simetidin)

Albendazole 15 mg/kgbb diabgai 2 dosis selama 1 bulan kontraidikasi peningkatan fungsi hati
(Amal,2022)

CBZ dan phenitoin

40 Pasien dengan hasil CT scan kepala “owl’s Eye”. Apakah diagnosis  infeksi CMV
Gambar di grup CBT

68
TO UGM 17 FEBUARI 2022
41. Seorang laki-laki usia 31 th datang ke poliklinik saraf karena mengeluh vertigo. Pasien
juga merasakan nyeri pada telinga kiri semenjak ada bintil-bintil kemerahan pada telinga
kiri. Selain itu pasien juga mengeluh mulutnya agak mencong ke kanan, dan tidak bias
berkumur. Sudah berobat ke dokter, dikatakan menderita herpes zoster di telinga.
Namun vertigo belum hilang. Sindroma apakah yang dialami pasien tersebut?
F. Wallerian syndrome proses aktif degenerasi retrograde dari ujung distal akson
yang merupakan akibat dari lesi saraf. Ini terjadi antara 7 hingga 21 hari setelah
lesi terjadi. Setelah hari ke-21, degenerasi saraf akut akan terlihat pada
elektromiograf.
G. Hunt Hess syndrome
H. Wallenberg syndrome
I. Ramsay Hunt syndrome
Terdapat riwayat infeksi herpes biasanya disekitaran infeksi telinga, karena disitu
lewat NVII dan NVIII jadi mulut mencong dan vertigo
Tx. Acyclovir 5x800mg selama 1 minggu, dan dikasih steroid
J. Herpes simpleks oticus

42. Pasien dengan kejang. Riwayat tertusuk paku, risus sardonikus positif. Berapakah skor abblet’s
 3 (severe)

43. Apakah terapi pasien diatas D/tetanus (severe)  Diazepam 50-100 mg dalam dextrose 5%
dengan kecepatan 10-15 mg/jam halaman 358 dibedakan spasme ringan, sedang, berat, pokdi
(kobe, 2022),
44. Pasien dengan positif HIV. Hasil CT scan (PCNLS )primary central nervous system lymphoma)
(diatas corpus calosum). Pada spektroskopi ditemukan?  peningkatan cholin

Lokasi tersering di ganglia bangsal, corpus calosum, lobus frontal

69
TO UGM 17 FEBUARI 2022
Epilepsi

45. Seorang wanita 25 tahun menggunakan kacamata tebal, kemudian mengeluh mata merah dan
nyeri. Dari obat antiepilepsi berikut manakah yang kemungkinan diberikan pada kasus di atas?
A. Asam valproat
B. Levetiracetam
C. Topiramate  glaukoma sudut tertutup (topiramat dan zonisamide)
D. Lamotrigine
E. Fenitoin

46. Anak usia 3 hari dengan gambaran EEG burst supression. Pernyataan yang benar mengenai
kasus ini  Penyakit ini berlanjut menjadi sindrom west (3-12 bulan), LGS (>12 bulan)
47. Anak usia 1 tahun, dengan tidak bisa berbicara setelah kejang, awalnya pasien dapat berbicara
mama, papa. Ada riwayat kejang demam umur 11 bulan. Apakah diagnosis pasien tersebut 
 Epilepsi landau kleffner (aphasia : verbal dan auditori agnosia)  prognosis baik, 15
tahun remisi, sering terjadi defisiensi bahasa menetap
 Dravet syndrome dan epilepsi lobus temporalis : ada riwayat kejang demam

70
TO UGM 17 FEBUARI 2022
48. Anak dengan keluhan tersentak-sentak pada bahu dan terjadi setiap bangun tidur (morning
jerk). Apakah Edukasi yang benar? D/ JME  pasien kemungkinan minum obat seumur hidup
(AV) LaLeVaTo;;;;; Kalau absans ELAVA

49. Pasien anak usia 8 tahun, tiba-tiba kepala tertunduk (tonik seizure), kadang bengong (absans
atipikal) Apakah gambaran EEG pasien tsb? D/ LGS  generalized slow spike waves; GPFA;
Trias LGS : bangkitan tonik , lena atipikal (wajib ada) dan gangguan kognitif
50. Pasien dirawat di ICU dengan sepsis dan di beri antibiotik. Pasien kejang. Obat antibiotik yang
menyebabkan kejang  penicillin , sefalosporin , klorokuinolon, imipenem (hindari karena bisa
menyebabkan kejang)

51. Seorang perempuan 28 tahun dengan epilepsi. Saat ini sedang hamil. Obat yang diberikan untuk
mencegah perdarahan maternal  Vitamin K 1 mg/hari intra muscular dosis tunggal (neonatus

71
TO UGM 17 FEBUARI 2022
resiko perdarahan pada ibu yang mengkonsumsi OAE enzim inducer (OAE hampir semua inducer
kecuali Asam valproate)

52. Seorang ibu hamil dengan kejang. Terapi yang diberikan  MgSO4 dosis 4 gram selama 5-10
menit dilanjutkan dosis maintenance 1-2 gram/ jam selama 24 jam post partum atau setelah
kejang terakhir.

Diazepam bisa diberikan pada saat persalinan dengan dosis 10 iv sedangkan phenytoin 15-20
mg / kgbb iv

53. Pasien pria usia 16 tahun dengan BB 35 kg, telah mendapatkan terapi fenitoin 250 mg/12 jam.
Pasien mengalami dizziness. Apakah yang menyebabkan kondisi tersebut  malnutrisi ?? dosis
4-10 mg per kg/bb (Over dosis ki ini)
- Kalau hipoallbumin  jumlah fenitoin terikat kurang  konsentrasi plasma meningkat 
toksisitas meningkat

54. Pasien ibu menyusui, konsumsi obat epilepsi. Apakah faktor yang perlu pada bayinya  relative
aman buat menyusui (carbamazepine , phenytoin, asam valproate)
Yang beresiko (phenobarbital , primidon , benzodiazepine, etosuksimide, zonisamide,
Felbamate) (Star low go slow)
a. Fungsi ginjal
b. Fungsi hepar
c. Fungsi gastrointestinal
d. Lamanya pengobatan
e. Tergantung jumlah kombinasi obat

NEUROFISIOLOGI

55. Seorang laki-laki berusia 69 tahun dibawa ke poli saraf dengan keluhan tetraparese yang timbul
perlahan-lahan sejak 2 tahun yang lalu. Kelemahan awalnya dirasakan sebagai kram di ujung
lengan kanan, kemudian menyebar ke eksteremitas lain. Pemeriksaan fisik ditemukan
tetraparese, tonus keempat eksteremitas spastis (+), fasikulasi (+, LMN), refleks tendon dalam
meningkat, dan atrofi di otot tangan dan lengan bawah bilateral (LMN). Apakah tatalaksana
farmakologi yang tepat untuk pasien tersebut?
A. Riluzole  ALS
B. Amantadine (antivirus dan anti Parkinson)- NMDA reseptor antagonis
C. 4-amynopiridine (lambert eaton syndrome)  guanidine hyrokholid atau 3-4
diaminopiridin, kortikosteroid
D. Glatiramer acetate (immunomodulator – multiple sclerosis)
E. Methylprednisolone (anti inflamasi)

56. Seorang laki-laki usia 22 tahun datang untuk mengevaluasi kelemahan pada wajah
dan anggota gerak atas yang memburuk secara progresif pada 2 tahun terakhir. Pasien
kesulitan menutup mata kanan dan kesulitan mengerucutkan bibir. Pasien kesulitan
menyisir rambut dengan tangan kanan dan telapak kaki kanan pasien selalu terlihat

72
TO UGM 17 FEBUARI 2022
jatuh. Pada pemeriksaan terdapat atrofi otot yang lebih menojol pada ekstremitas atas
dengan terdapat winged scapula. Saat inspeksi, lengan atas pasien terlihat lebih kecil
dibandingkan dengan lengan bawah namun bagian deltoid terkesan dalam batas
normal. Creatinin kinase 510 IU/L (nilai normal 350 IU/L). Kelainan distrofi muscular
yang paling mungkin pada pasien adalah *
a. Becker (CK tinggi 100-1000x)
b. Duchenne (CK Tinggi 100-1000x)
c. Emery-Dreifuss (joint contractur awal masa anak anak) (kelemahan otot
progresif–humeroperoneal ke scapula dan pelvic girdle)(cardiac involvement)
 tidak kena wajah
d. Fascioscapulohumeral (kelemahan wajah leher dan bahu) (drop foot)
(winged scapula)  wajah kena
e. Myotonic

57. Pasien dengan miastenia gravis sudah mendapatkan terapi piridostigmin dosis maksimal tapi
masih belum ada perbaikan. Terapi yang tepat diberikan
a. Kortikosteroid  imunosupresan diberikan setelah pemberian kortikosteroid
b. Imunosupresan
c. IVIG
d. Plasmafaresis

73
TO UGM 17 FEBUARI 2022
58. Pasien dengan kelemahan keempat ekstremitas yang terjadi secara perlahan-lahan selama 6
bulan yang lalu. Hasil EMG KHS menurun, latensi distal memanjang, ada blok konduksi, CMAP
normal. Apakah diagnosis pasien  CIDP (> 2bulan) AIDP (<28 hari)

NYERI KEPALA

59. Laki-laki usia 30 tahun, datang dengan keluhan nyeri hebat seperti tertusuk-tusuk di
daerah tenggorokan, lidah bagian belakang dan sampai ke telinga kanan. Pasien
pingsan. Pasien dengan riwayat neuralgia trigeminal. Apakah diagnosis pasien tersebut
diatas?
a. Neuralgia trigeminal idiopatik
b. Neuralgia trigeminal simtomatik
c. Syncope accociated with neuralgia trigeminal
d. Syncope accociated with Neuralgia Glosofaringeal Idiopatik : belakang lidah; fossa
tonsilar, faring, suduh rahang, telinga;
Pemicu : batuk, menelan, menguap, berbicara (paroksismal, durasi detik – 2
menit)
e. Neuritis optika

60. Wanita 42 th dengan keluhan nyeri kepala sebelah kanan, durasi 10 menit dengan injeksi
konjungtiva, lakrimasi, dialami 5 kali/hari. Apakah terapi fase akut  indometasin (hemicrania
Paroksismal) durasi 2-30 menit , serangan lebih dari 20x/ hari dosis : 150 mg/ hari oral bisa
ditingkatkan sampai 225 mg/hari per oral , 100-200 mg/ hari intravena
SUNA ,SUNCT, Cluster, Hemicrania- ada injeksi lakrimasi dan konjungtiva

61. Wanita, nyeri kepala terus menerus sepanjang hari dan sampai-sampai di hapal waktu kejadian.
Apakah diagnosis pasien  Probable New daily persisten headache dirasakan sepanjang hari
dan diingat dengan jelas waktu kejadian nya) yang diingat jelas dan nyata, nyeri berlangsung
terus menerus dan tidak membaik dalam 24 jam serta berlangsung kurang dari 3 bulan(probable
NDPH) jika >3 bulan (NDPH)

62. Anak 12 tahun, nyeri perut terutama di bagian tengah, muntah-muntah. BAK dan BAB normal.
Apakah terapi yang tepat  migrain abdominal (nyeri perut bagian tengah) domperidon 5 mg
sebelum diberikan NSAID. Beta blocker juga bisa dikasih untuk profilaksis: propanolol dan
metoklopramid. Gejala yang menyertai (nausea , vomit ,anoreksia, pucat). Lokasinya (midline
periumbilical , poorly localized)

63. Anak 8 tahun, nyeri perut terutama di bagian tengah, muntah-muntah. BAK dan BAB normal.
Apakah diagnosisnya  migrain abdominal (pokdi 2019 neuropediatri 297) sama dengan nomor
62

64. Pasien dengan nyeri kepala, tidak diperberat aktivitas. Apakah patomekanismenya  ?? Tension
type headache

74
TO UGM 17 FEBUARI 2022
65. Pasien dengan nyeri kepala, minum kopi 10 cangkir per hari. Apakah pernyataan yang benar 
Jawaban : (withdrawal caffeine) benar semua
Konsumsi kafein > 200 mg/hari selama 2 minggu
Nyeri menghilang setelah 7 hari penghentian total konsumsi kafein
Nyeri kepala mereda dalam 1 jam setelah konsumsi kafein 100 mg
Nyeri muncul 24 jam setelah berhenti konsumsi kafein

PEDIATRI

66. Seorang anak dengan bahu tersentak-sentak dan mengeluarkan suara tanpa arti.
Apakah diagnosis yang tepat ?
a. Tics fasial
b. Sindroma Tourette : koprolalia; kompleks vocal tics (berbicara kasar), Tic motorik
: terapi Haloperidol
c. Gerakan mioklonus
d. Khorea Sydenhams (demam rematik post infeksi streptokokkus beta hemolitikus
type A)
e. Gerak malingering

67. Pasien dengan tidak bisa diam, sering mengganggu, tidak bisa mengikuti perintah
gurunya. Terapi pasien adl? ADHD : metilphenidat dan amphetamine (ekstasi) (Lini 1) ;
60 mg per hari (anak anak) dewasa 100 mg/ hari
a.
B.
C. Metamfetamin (sabu sabu)
D. Atomoxetin ?? disertai dengan comorbid Tics , Tourette dan epilepsi
E.

68. Seorang ibu membawa anak usia 7 tahun dengan keluhan banyak bergerak tidak bisa duduk
diam. Pasien sering bermain dan sering mengambil mainan yang itu-itu saja. Anak dapat
bermain dengan teman sebaya. Saat ini belum lancar bicara mengemukakan keinginan dengan
merangkai 4 kata dan dapat mengerti perintah sederhana. Anak bersekolah di kelas 1 SD tapi
tidak mau duduk, banyak bergerak. Test pekembangan menunjukkan kemampuan anak usia 5
tahun. Apakah diagnosis yang paling mungkin pada penderita ini?
a. mental retardasi berat : hanya diajarkan untuk hygiene dan kemampuan bicara sederhana
b. mental retardasi sedang : mampu latih
c. mental retardasi ringan : mampu dididik tapi ketinggalan
d. ADHD : kesulitan menyesuaikan diri dan tidak dapat bersosialisasi (bermain bersama )
e. autisme : gangguan hubungan social , tingkah laku dan komunikasi

SLEEP

75
TO UGM 17 FEBUARI 2022
69. Seorang laki-laki 37 tahun datang dengan keluhan gangguan perilaku akibat mimpi. Dikatakan
oleh istri pasien terkadang memukul, menendang atau berteriak saat tidur. Pasien di diagnosis
Parkinson . Apakah diagnosis yang paling mungkin pada pasien tersebut? *RBD
a. Nocturnal cramps non rem (keram pada paha dan kaki) terapinya peregangan sebelum
tidur dan Quinine sulfat (lansia, ibu hamil)
b. Sleep bruxism non rem  tidak farmakologi; perbaiki penyakit gigi, dan pelindung gigi
(anak)
c. Parasomnia non REM
d. REM sleep behavior disorder  klona 0,5-1 mg; lorazepam
e. Central sleep apnea
N REM

Sleep terror

Sleep Talking

Sleep eating

Sleep Walking

REM

Nightmare

Sleep Paralysis

RBD

So

Halaman 77-78 buku

70. Anak-anak, mimpi buruk... dan saat bangun pasien mengingat mimpinya. Diagnosisnya? 
Nightmare
71. Pasien anak-anak mimpi buruk dan tidak dapat mengingat mimpinya. Diagnosisnya  sleep
terror
72. Pasien anak-anak di bawa oleh ibunya dengan keluhan sakit gigi. Anak sering menggertakkan
giginya saat tidur(bruxism). Apakah mekanismenya?  gangguan kontrol gerakan pada sirkuit
dopaminergik + faktor luar (gigi, alkohol, dsb)

73. Pasien dengan tidur jam 01.00 bangun jam 10.00. D/ delayed sleep fase syndrome. Apakah
terapi  memundurkan waktu tidur 3 jam. Light therapy

Kronototerapi (terapi cahaya) & Melatonin :


- Delayed : cahayanya diberi pagi hari (utk menjaga bangun) / melatonin diberi malam (1-3 mg)
- Advanced : cahayanya diberi malam hari (utk cegah tidur) / melatonin diberi pagi (saat bangun)
(1-3mg)

- Irregular SWR : disebabkan kerusakan otak, (demensia, retardarsi mental)  terapi :


kronoterapi, vitamin B12 dan obat2 hipnotik, MELATONIN MALAM HARI

76
TO UGM 17 FEBUARI 2022
- FREE RUNNING (non-24 hour/pada tuna netra (butaaaaaaaa)) : memberikan jadwal tidur
(mengatur irama sirkardian), kemudian dapat diberikan melatonin 10mg sebelum tidur

74. Pasien anak 2 tahun belum bisa berjalan, riw. lilitan tali pusat, tetraparesis hipertonus. Apakah
diagnosisnya?  CP spastik tetraparese
75. Anak-anak dengan lordosis. Pasien jika berdiri bertumpu pada pegangan lutut. Apakah
mekanismenya Diagnosa (muscular Dystrophy)  kelemahan pada paravertebra dan otot
abdominal?? Atau parese quadriceps femoris / kelemahan otot pelvis pola berjalan  waddling
gait

MOVDIS

76. Pada pasien dengan Parkinson’s Disease, mengalami disfungsi substansia nigra
termasuk neuron loss. Pemeriksaan postmortem pada substansia nigra pada pasien
Parkinson’s Disease ditemukan?
a. Intranuclear inclusion bodies
b. Intranuclear and intracytoplasmic inclusion bodies
c. Intracytoplasmic inclusion bodies
d. Neurofibrillary tangles (Alzheimer)
e. Amyloid plaques
77. Pasien wanita dengan keluhan gerakan tersentak-sentak pada jari (Chorea). Apakah terapi??
a. Pramipexole
b. Haloperidol ???
c. Olanzapine
d. ..
e. Clonazepam ??
Prinsip terapi chorea : menekan pergerakan (jadi ditekan dopaminnya)
- Neuroleptik tipikal dan atipkal
- Penekan dopamin : tetrabenazine dan reserpine
- Gabaergik : Amantadine

78. Pasien wanita dengan tremor terutama saat beraktivitas dan kakak perempuannya mengalami
hal yang sama. Apakah pernyataan yang benar  Tremor esensial
a. Autosomal dominan
b. Autosomal resesif
c. X-linked dominan
d. X-linked resesif
e. Y-linked
79. Pasien laki-laki dapat terapi injeksi, setelah di injeksi pasien kaku. Mekanisme obat yang
menyebabkan kaku?  blok reseptor dopamin (antagonis dopa)

TRAUMA

77
TO UGM 17 FEBUARI 2022
80. Pasien post kll, lebam pada belakang telinga. Dimanakah letak lesi posterior battle Sign? 
fossa posterior
Lesi Fossa Media : otorheea, parese N VII dan VIII
Fossa anterior Anosmia, Racoon eye, otorheea

81. Pasien wanita dengan nyeri pada bahu dan leher. Pasien mengalami kecelakaan dengan gerakan
leher kedepan secara cepat kemudian ke belakang. Apakah mekanisme tersebut?  whiplash
injury (flexi kedepan kemudian kebelakang)

NYERI

82. Pasien dengan nyeri-nyeri, tidak ada bengkak pada persendian. Kriteria pasien american...
rheumatology?? FIBROMYALGIA DIAGNOSTIC CRITERIA
A. Waktunya > 3 bln
B. Dia mengetahui letak nyeri
C. ...
D. WPI >7, Symptom Severity > 5
E. ..

Fibromyalgia : bukan nyeri neuropatik dan nosiseptif (tetapi namanya disfungsional)

83. Pasien dengan post trauma, keluhan tangan pucat, dingin, perubahan warna, kulit kering.
D/CRPS. Apakah faktor fisiologi?  simpatetik dengan vasospasme; Hiperaktifitas simpatis

84. Pasien riw. Stroke. Lesi pada Thalamus. Apakah terapi pasien tersebut?  antidepresan trisiklik
Diagnose : central post stroke pain
Lini 1 : antidepresan trisiklik : Amitriptilin  bisa ganti duloxetine kalau ada kontraindikasi
Lini 2 : anti epilepsi : gabapentin LTG,
Kombinasi lini 1 + 2

85. Pasien wanita dengan nyeri pada pipi terutama saat sikat gigi. Apakah terapi lini pertama? 
karbamazepin
Diagnose : Trigeminal Neuralgia
Lini 1 : Carbamazepin , Oxcarbazepin
Lini 2 : Baclofen dan LTG, gabapentin

78
TO UGM 17 FEBUARI 2022
86. Pasien dengan nyeri punggung bawah, tidak menjalar. Pemeriksaan Patrick (+). Pemeriksaan
yang dianjurkan? Sacroiliaca joint Ro. Pelvis

ONKOLOGI

87. Pasien dengan pituitari adenoma. Apakah gennya? Alel 11q13 (MEN1)
88. Pasien dengan riw. Operasi payudara. Pasien dengan metastasis. Pasien sebelumnya sudah
dilakukan operasi dan radioterapi. Apakah pilihan terapi selanjutnya?  KEMOTERAPI  tumor
payudara bersifat sensitif kemo (modul neuroonkologi). Terapi paliatif  WBRT (whole brain
radio teraphy)

Karsinoma radiosensitif : payudara dan paru (metas)


Karsinoma radioresisten : melanoma dan karsinoma renal (metas)

Efek samping akut : kulit kering, rambut rontok, sakit kepala, muntah, letargi, edema sereberi, dan otitis
media, "Somnolence syndrome" akibat kelelahan
Efek lanjutan [late effects) : radionekerosis, atrofi, leukoensefalopati, dan demensia (gangguan
neurokognitif yang lebih berat)

Dosis : Dosis radiasi 30Gy terbagi dalam 10 fraksi selama 2 minggu atau 37,5Gy terbagi dalam 15 fraksi
selama 3 minggu.

89. Pasien wanita, gemuk dengan nyeri kepala dengan benign Idiopatik hipertensia. Apakah terapi
 lumbal punksi
D/ Pseudotumor cerebry : gejala peningkatan TIK seperti gejala Tumor IK, pada Wanita gemuk
biasanya
90. Pasien wanita, dengan nyeri kepala dengan gangguan penglihatan. Foto polos?? Apakah
penyebabnya??
Copper beaten skull  penyebabnya peningkatan ICP akibat hidrosefalus obstruktif DD/ massa
intracranial, pada anak2 craniositosis.
Copper beaten skull : peningkatan TIK pada anak akibat , penebalan tabula Interna (lokasi
perderahan EDH biasanya)

79
TO UGM 17 FEBUARI 2022
91. Pasien dengan gambar CT scan kepala meningioma. Patofisiologi  degranulasi arachnoid
EGFR untuk astrositoma (modul neuroonkologi)

PENELITIAN

92. suatu penelitian bertujuan mencari hub Gula darah dengan kejadian stroke, penelitian
dilakukan 6 bulan, trus klo ga salah sampling dengan metode cosecutive. yg ditanya
metode pengambilan data ?
A. pencuplikan random
B. pencuplikan non random ??
C. random sistematis
D. pencuplikan diagnosis
E. lupa yg satunya

93. Pasien dengan PH 7,25 (7,35-7,45), PaCO2 60 (35-45), HCO3 24 (26-22). Diagnosis ?  asidosis
respiratorik
94. Pasien dengan kesadaran menurun, di konsul kebagian neurologi dengan SGOT 275, SGPT >200,
ureum 0,9, kreatinin normal. Apakah penyebab kesadaran menurun?  peningkatan amonia di
otak.
Encephalopathy Hepatikum  EEG triphasic wave
Peningkatan Ureum : ensefalopati Uremikum

95. Pasien dengan Ca paru dan ptosis yang memberat pada saat istirahat. Apakah mekanismenya?
 antibodi voltaged gated kanal kalsium. Dx. LEMS (paraneoplastic syndr)
96. Pemberian dexametason pada tumor otak. Sifat dari dexametason?  mineralokortikoid
rendah, waktu paruh panjang
Glikokortikoid??

80
TO UGM 17 FEBUARI 2022
97. Pasien wanita post di rawat di ICU. Pasien tidak dapat bergerak tetapi dapat melokalisir nyeri
dan menguap dalam waktu 2 bulan. Apakah diagnosis pasien?
a. Persisten Vegetative stage ( 4 minggu)
b. Permanen Vegetative stage (> 6 bulan pnyebab non traumatic injury) >12 bulan
penyebab traumatic Brain Injury
c. Minimal concious stage
d. LOCK in syndrom

e. Koma

98. Pasien dengan  aneurisma saccular


99. Pasien dengan pusing, dilakukan dix halpike. Apakah hasil dari manuver?  kanalis
semisirkularis posterior

KUK 1 edit kobe


Neurovaskular (Undip)

81
TO UGM 17 FEBUARI 2022
1. Laki-kaki 50 th datang dengan kelemahan sisi Kiri mendadak sejak 2 jam yang lalu.
Riwayat keluar dari ICCU 4 bulan yang lalu karena infark miokard. Riwayat operasi ORIF
2 bulan (2 minggu) yang lalu. Riw. BAB hitam 2 minggu sebelum masuk RS. Pemfis TD;
185/105 mmhg, GCS E3M6V5 dan lateralisasi sinistra. CT scan : infark di basal ganglia
dextra, GDS : 350 (>50-450)gr/dl. Pasien akan direncanakan rTPA.
Manakah faktor KI absolut rTPA ?
A. TD pasien
B. Infark miokard 2 bulan
C. OP mayor 2 bulan
D. Riw. Perdarahan GI 2 mgg yang lalu (3 minggu)
E. GDS 350 gg/dl

2. Laki2 52 th, ke poli saraf dengan keluhan mata kanan bengkak, kemerahan disertai nyeri
setelah KLL. Pada pemeriksaan dijumpai proptosis,, kemosis, bruit orbital dan
opthalmoplegia.. Tidak ada kelainan motorik.
Apakah pemeriksaan penunjang yang tepat untuk menentukan diagnosa tsb ?
F. VEP
G. TCD
H. CT Scan cerebral
I. DWI cerebral
J. DSA
BERULANG
3. Wanita, 67 th dengan keluhan kelemahan anggota gerak kiri mendadak 2 hari SMRS.
Riw. DM tipe 2 dan HT tidak terkontrol. EKG menunjukkan AF. TEE normal. CT
angiografi : stenosis 50% pada lumen A. Karotis eksterna kiri, stenosis 50% pada ICA
kanan.
Apakah terapi yang paling tepat :
A. Carotid endarterektomi
B. Antikoagulan
C. Trombektomi
D. Stenting A. Carotid eksterna kiri
E. Stenting a. Carotid Interna kanan KARENA HEMIPARESE SINISTRA

4. Wanita 47 th di IGD dengan kelemahan anggota gerak kiri mendadak onset 6 jam yll,
terdapat riwayat DM tipe 2, HT tidak terkontrol, EKG tidak ada AF. TEE normal, pemfis :
GCS 15. N. Cranialis parese N. VII dan N. XII kiri . Motorik 555/333, 555/333. Sensorik
dalam batas normal. CT scan tidak didapatkan infark.
Tatalaksana neurointervensi yang tepat?
A. Coiling
B. IV trombolisis
C. IA trombolisis
D. Trombektomi
E. Carotid angioplasty and stenting

82
TO UGM 17 FEBUARI 2022
5. Wanita, 67 th dengan keluhan kelemahan anggota gerak kiri mendadak 6 jam SMRS.
Riw. DM tipe 2 dan HT tidak terkontrol. EKG tidak menunjukkan AF. TEE normal. GCS
E4M6V5. N. Cranialis parese N. VII dan N. XII kiri . Motorik 555/333, 555/333. Sensorik
dalam batas normal. Tindakan Apakah yang tepat?
A. Coiling
B. IV trombolisis
C. IA trombolisis
D. Trombektomi
E. Carotid angioplasty and stenting

6. Perempuan 65 th, mengalami Multiple episode TIA, hemiparese kanan 3 jam, namun
masih bisa beraktivitas seperti biasa. Bicara pelo (+), mulut Perot (+), Riw. HT dan Riw.
DM (-).. Apakah tindakan yang tepat dilakukan ?
a. Coiling
b. IA trombektomi
c. IV trombolisis
d. Trombektomi
e. Carotid Angioplasty stenting

7. Laki2 28 th, hemiparese kiri mendadak, afasia global. GCS E4MrVx. CT scan infark
trombotik akut subkortikal frontoparietal kiri. Pemeriksaan. DSA didapatkan gambaran
oklusi pada arteri carotid Interna kanan dan kiri hingga seluruh MCA, ACA, PICA kanan
dan kiri. Disertai gambaran khas sesuai gambar :

Apakah diagnosa pada kasus tsb :


A. Stroke kardio emboli
B. Binswanger disease
C. Posterior reversible ensefalopati syndrome
D. Subarachnoid hemmoraghe dengan delayed ischemic

83
TO UGM 17 FEBUARI 2022
E. Trombosis serebri dengan moya-moya disease (smoked puff appearance)

8. Seorang laki-laki 34 tahun datang ke IGD karena mengalami penurunan kesadaran yang
disertai nyeri kepala hebat mendadak sejak 2 jam SMRS. Pada pemeriksaan GCS
E2M4V3, TD : 150/90 mmhg, HARI ; 110 x/i, RR 22 x/i. Temp 36,2. Pupil bulat, isokor
3mm/3mm. CT scan kepala menunjukkan gambaran : PSA pada regio frontalis. Pasien
direncanakan untuk CT angiography untuk mencari etiologi. Dimanakah Lokasi
aneurisma tersering pada kasus tsb?
a. Acom
b. Pcom
c. Bifurcatio ICA dan MCA
d. Pertemuan antara arteri communicans posterior dengan ICA
e. A. Basilaris distal

9. Laki-kaki 50 th datang dengan kelemahan sisi Kiri mendadak sejak 2 jam yang lalu.
Riwayat keluar dari ICCU 4 bulan yang lalu karena infark miokard. Riwayat operasi
ORIFLAME 2 bulan yang lalu. Riw. BAB hitam 2 minggu sebelum masuk RS. Pemfis TD;
185/105 mmhg, GCS E3M6V5 dan lateralisasi sinistra. CT scan : infark di basal ganglia
dextra, GDS : 350 gr/dl. Dimanakah letak tromboatherosklerosis tersering berdasarkan
hasil CT scan kepala?
A. Bifurcatio MCA
b. Arteri lentikulostriata
c. A. Thalamoperforata
d. A. Coroidalis anterior
e. Acom

Urutan tersering plak atheromatosa :


a. Bifurcation MCA (3)
b. ICA (1)
c. Proksimal PCA (4)
d. Proksimal ACA (5)
e. Pertemuan a. Vertebralis & a. Basilaris (2)

Lokasi tersering terjadinya plak atheromatosa:


1. ICA pada daerah percabangan dari a.carotis communis
2. Cabang cervical a. Vertebralis dan lokasi bertemunya menjadi a. Basilar
3. Bifurcasio MCA
4. Proximal PCA yang mengelilingi mesensefalon
5. Proximal ACA yang melalui anterior dan lengkung corpus callosum

10. Wanita 47 th dibawa ke poli saraf dengan keluhan kelemahan anggota gerak kiri, sejak 1
th yang lalu. Terdapat Riw. DM tipe 2 dan HT tidak terkontrol. EKG tidak menunjukkan
AF. TEE normal. Pada pemeriksaan fisik didapatkan GCS E4M6V5, N. Cranialis parese N.

84
TO UGM 17 FEBUARI 2022
VII dan N. XII kiri . Motorik 555/444, 555/444. Sensorik dalam batas normal. Meminum
obat CPG 75 mg/24jam. Apakah Efek samping CPG bila diperlukan dalam jangka lama?
A. Netropenia
B. Trombositopenia purpura
C. Vasodilitasi periperal
D. Leukopenia
E. Anemia

11. Pria 38 th ke UGD dengan keluhan kejang kelojotan , 1 jam yang lalu. Kejang terjadi
selama 5 menit, saat kejang pasien tidak sadarkan diri, setelah kejang pasien tampak
mengantuk dan bingung. OS mengatakan nyeri kepala hebat mendadak saat
beraktivitas, mual (-), muntah (-), pasien menyangkal adanya HT, DM. Dari pemeriksaan,
kaku kuduk ringan. NN cranialis dalam batas normal, kekuatan motorik 555/555,
555/555, sensorik dan otonom dalam batas normal. Apakah Penyebab terjadinya kejang
pada kasus tersebut ?
a. Stealing phenomena  pada AVM
b. Sentinel bleed:pada PSA
c. Vasospasme
d. Intraparenkimal extention
e. SIADH

12. Pria 38 th ke UGD dengan keluhan kejang kelojotan , 1 jam yang lalu. Kejang terjadi
selama 5 menit, saat kejang pasien tidak sadarkan diri, setelah kejang pasien tampak
mengantuk dan bingung. OS mengatakan nyeri kepala hebat mendadak saat
beraktivitas, mual (-), muntah (-), pasien menyangkal adanya HT, DM. Dari pemeriksaan,
GCS E3M5V3, kaku kuduk ringan. NN cranialis dalam batas normal, Kekuatan motorik
444/555, 444/555 sensorik dan otonom dalam batas normal.. Berapakah derajat klinis
berdasarkan hunt and Hess :
f. Grade I (asimptomatik, CM, nyeri kepala ringan, mati 11%)
g. Grade II (nyeri kepala berat, CM, mati 26%)
h. Grade III (somnolen, defisit ringan, mati 37%)
i. Grade IV (stupor, defisit berat, 71%)
j. Grade V (koma, mati 100%)

13. Pria 18 tahun ke IGD dengan keluhan sakit kepala mendadak saat sedang beraktivitas.
Tidak didapatkan mual, muntah, kelemahan anggota gerak, kesemutan, kejang maupun
penurunan kesadaran. Dari pemfis didapat kaku kuduk ringan, NN. Cranialis dalam batas
normal, Kekuatan motorik 555/555, 555/555, sensorik dan otonom dalam batas
normal.. Apakah usulan pemeriksaan penunjang pada pasien tersebut?
A. Lumbal pungsi
B. Transcranial doppler
C. Ct scan dengan non kontras

85
TO UGM 17 FEBUARI 2022
D. DSA
E. MRI cerebral dengan kontras

14. Wanita 47 th dibawa ke poli saraf dengan keluhan kelemahan anggota gerak kiri, sejak 1
th yang lalu. Terdapat Riw. DM tipe 2 dan HT tidak terkontrol. EKG tidak menunjukkan

AF. TEE normal. Pada pemeriksaan fisik didapatkan GCS E4M6V5, N. Cranialis parese N.
VII dan N. XII kiri . Motorik 555/444, 555/444. Sensorik dalam batas normal. DSA : AVM.
Apakah diagnosis yang tepat ?
A. Cerebral sinus venous trombosis
B. Carotid cavernous fistula
C. Dural arteri venous fistula
D. ARTERIVENOUS MALFORMATION
E. Aneurisma intrakranial

NEUROPEDIATRI
15. Seorang anak berusia 2 th dengan keluhan tidak dapat berjalan lebih lemah anggota
gerak bawahnya. Saat usia satu tahun pasien baru bisa miring dan kesulitan untuk
mengangkat kepala sendiri. Jika diberi minum sering tersedak. Pasien sering kejang sejak
3 bulan yang lalu. Ada kejang. Saat kejang kepala pasien menoleh ke kiri dan kaku
anggota geraknya. Anak lahir prematur 7 bulan, lahir dengan spontan, tidak langsung
menangis. Pada pemeriksaan fisik didapatkan refleks fisiologis meningkat dan refleks
patologis bilateral. Apakah terapi yang paling tepat untuk kasus kejang? Diagnosis : CP
a. Phenitoin (level C parsial)
b. Carbamazepin (level C parsial) Level D TPM,VPA
c. Asam valproat (level A utk absans, level C parsial)
d. Fenobarbital (level C parsial)
e. Clobazam (level C parsial

16. Seorang ibu membawa anak perempuannya berusia 3 tahun, dengan keluhan belum
dapat komunikasi. Saat usia menginjak 4 bulan anak mulai tengkurap dan saat usia 5
bulan anak sudah mulai duduk. Ibu pasien mengatakan bahwa mulai usia 6 bulan
anaknya mengalami gangguan perkembangan. Saat usia tersebut pasien mulai kesulitan
bergerak dan menggerakkan tangan yang tidak normal dan berulang-ulang. Apakah
diagnosis yang paling tepat pada kasus tersebut?

86
TO UGM 17 FEBUARI 2022
A. SINDROM RETT (wanita,ggn bahasa, pkmbgn sosial, motorik) mutase kromosom
Xq28, gen MECP2 : tidak ada terapi
B. Sindrom asperger (mirip autism: bahasa, sosial, perilaku) baik intelengensia
C. Sindrom disintegratif (laki2, penurunan 2-4thn, ggn bahasa, gerakan repetitif &
koordinasi)
D. Autisme (ggg3 komunikasiverbal/non verbal, g3 intraksi social, g3 tingkah laku)mk
E. Cerebral palsy

17. Seorang anak laki-laki 9 th dibawa ke poli saraf oleh ibunya. Anak dilaporkan sering usil
terhadap temannya, mudah marah, sering bermain didalam kelas, sering tidak
mendengarkan apa yang dikatakan dan tidak mau mengikuti pelajaran. Area otak
manakah yang berperan pada kondisi pasien tersebut? Diagnosa : ADHD, RM ringan
bolju
A. Gyrus frontalis
B. Gyrus parietal
C. Gyrus temporalis
D. Gyrus prefrontalis (eksekutif)
E. Gyrus angularis

18. Seorang anak perempuan berusia 7 tahun dibawa oleh orang tuanya karena anak sering
tampak bengong, berulang beberapa kali setiap hari. Keluhan dialami selama 4-20 detik
secara tiba-tiba saat terjadi kejang pasien tidak respon apabila di panggil. Pasien telah
diberi obat asam valproat, namun setelah minum obat pasien mengeluh badan terasa
gatal-gatal.. Obat pengganti yang paling tepat adalah?
A. Phenition
B. Carbamazepin
C. Levetiracetam
D. Lamotrigin (level C) DAN ETOSUCSIMUD
E. Gabapentin
Bangkitan absan : AS Valpro, ethosuximid/level A, LTG /level C (VEL)

19. Seorang anak laki-laki berusia 5 tahun dengan keluhan sulit berdiri sejak 6 bulan yang
lalu, bila berdiri anak memanjat diri sendiri, selain itu anak kesulitan untuk menyisir
rambut . pada pemeriksaan fisis otot betis membesar. Proses persalinan normal.
Dibidan. Perkembangan anak sebelumnya normal. Saat ini pasien telah minum obat
deflazacort jangka panjang rutin. Apakah diagnosis yang paling mungkin pada kasus ini?
A. DMD
B. SMA
C. Poliomielitis
D. Poliomiositis
E. Becker muscular distrofi

87
TO UGM 17 FEBUARI 2022
20. Seorang anak perempuan 13 th, dibawa ibunya ke poli dengan keluhan utama kejang
kejang terjadi terutama sewaktu bangun tidur pagi, sejak 1 tahun yang lalu. Kejang
berlangsung singkat, berupa gerakan seperti menghentak pada bahu dan kedua lengan.
Kejang berlangsung singkatdan terkadang kejang terjadi seluruh tubuh. Pemfis normal,
MRI dan CT Scan kepala juga normal. Terapi apa yang tepat untuk pasien ini? D/ JME :
LaLeVaTo : lamot, levetiracetam, valpro, topira
a. Phenobarbital
b. Lamotrigin
c. Clobazam
d. Phenitoin
e. Oxcarbamazepin

21. Seorang anak laki-laki usia 6 tahun datang ke poli dengan gejala kelemahan anggota
gerak bawah yang dirasakan sejak 2 tahun yang lalu. Dari pemeriksaan didapatkan anak
harus bertumpu saat berdiri, betis terlihat membesar. Kakak pasien mempunyai riwayat
gejala serupa dan meninggal pada usia 20 tahun. Apakah komplikasi yang mengancam
jiwa paling sering pada kasus ini adalah?
A. Rhabdomyolisis
B. Paralisis otot pernapasan
C. Kardiomiopati
D. Perdarahan otak
E. Gagal ginjal

22. Seorang anak laki-laki 8 tahun dengan keluhan kejang sering terjadi saat tidur. Saat
kejang mulut mencong, air liur diikuti gerakan menghentak-hentak seluruh tubuh.
Keluhan tersebut dialami tiga kali dalam sebulan. Terapi apa yang tepat pada pasien ini
adalah ? D/ BECTS/ rolandic : tdk perlu terapi karena remisi spontan
A. Asam valproat
B. Fenobarbital
C. Levetiracetam
D. Karbamazepin , valro, lamo, leveti (karba valing lale)
E. fenitoin

23. Seorang Laki-laki 65 th ke poli dengan sering lupa terutama apa saja yang baru dilakukan
dan sulit mengingat kegiatan yang sebelumnya telah direncanakan. Pasien masih bisa
menyetir dan mengurus diri dan kegiatan rumah tangga. Tidak ada riw. HT dan DM.
Apakah terapi yang paling tepat pada kasus tersebut? MMSE 25. Diagnose : MCI
A. Psikoterapi
B. Donepezil
C. Fisioterapi
D. Galantamin
E. stimulasi kognitif

88
TO UGM 17 FEBUARI 2022
24. Seorang pasien 54 th dengan keluhan kesulitan memahami pembicaraan secara tiba-tiba
dari pemeriksaan didapatkan pasien dapat berbicara lancar. Tidak dapat memahami isi
pembicaraan namun dapat mengulangi apa yang diperintahkan kepada pasien.
Gangguan bahasa yang terjadi pada pasien tsb?
F. Afasia Wernicke
G. Afasia transkortikal motorik
H. Afasia transkortikal sensorik (girus angularis, MCA post)
I. Afasia global
J. Afasia anomik

25. Laki-laki 55 tahun dengan keluhan anggota gerak kanan mendadak lemah sejak bangun
tidur pagi hari. Pasien sadar dapat berkomunikasi dan memahami pembicaraan orang
lain. Pemfis didapatkan pasien tidak memahami perkataan dan dapat mengulang kata-
kata. Dimanakah tropis kelainan letak lesi pada kasus tersebut? Dx : Afasia transkortikal
sensorik kah
A. Lobus frontal inferolateraL
f. Fasikulus arkuata
g. Gyrus cinguli
h. Girus angularis
i. Lobus temporal superior

26. Seorang laki-laki 65 tahun sering lupa sejak 2 tahun yang lalu. Pasien mulai sering lupa
arah tempat, sulit memakai baju sendiri dan sulit tidur di malam hari. Oleh dokter di
diagnosa Alzhemier berat dan telah diterapi. Apakah terapi yang paling tepat pada kasus
tersebut ?
F. Inhibitor kolinesterase + NMDA antagonis (donepezil dan memantine)
G. MAO inhibitor + AchEI
H. AchEI
I. NMDA antagonis
J. Gingkogiloba
Alzheimer ringan sedang (DRG) donepezil, rivastigmine, galantamine)
27. Seorang laki-laki 65 tahun mengeluh mendadak sulit membaca, bicara spontan lancar,
komprehensif bagus, penamaan dan pengulangan baik. Pasien dapat menulis tetapi
tidak dapat membaca apa yang ditulis. Pada kasus diatas arteri yang terlibat adalah
(aleksia tanpa agrafia) ?
a. A. Cerebri anterior
b. A. Choreidea anterior
c. A. Cerebri media
d. A. Cerebri posterior (PCA)KORTEKS KALKARINA, SPLENIUM
e. A. Serebelaris inferior
Aleksia dgn agrafia : gyrus angularis / MCA
28. Seorang laki-laki 40 tahun datang ke poliklinik saraf dengan keluhan kelopak mata turun
pada sisi kanan sejak 2 hari yang lalu. Hal ini timbul dirasakan semakin lama semakin

89
TO UGM 17 FEBUARI 2022
memberat disertai rasa tidak seimbang seperti mau jatuh (ataxia cerebellar), pasien juga
mengeluh pandangan sulit saat melihat ke bawah sisi medial. Pemeriksaan didapatkan
pasien terjatuh ke sisi kanan saat mata tertutup. Apakah diagnosis yang paling mungkin
pada kasus diatas?
A. Sindom Benedict
B. Sindrom Raymound
C. Sindrom Nothnagel : tectum of mesencepalon/midbrain : Parese N III ipsilateral ,
ataxia cerebellaripsilateral
D. Sindrom wallenberg
E. Sindrom Weber

29. Seorang pasien usia 54 tahun sulit bicara secara tiba-tiba. Dari pemeriksaaan didapatkan
pasien tidak dapat mengeluarkan suara dan dapat memahami isi pembicaraan dan tidak
dapat mengulangi apa yang diperintahkan kepada pasien. Area yang terjadi lesi pada
pasien ini adalah? Dx : afasia motorik
a. MCA anterior
b. MCA posterior
c. MCA superior
d. MCA inferior
e. ACA

30. Seorang wanita 70 th, pekerjaan IRT cekatan tangan kanan, dirujuk ke dokter spesialis
saraf karena mengalami penurunan kemampuan memori sejak 3 tahun yang lalu. Suami
pasien mencatat pasien sering lupa menaruh barang, membuat masakan. Pasien sering
lupa pertanyaan yang sudah ditanyakan beberapa menit sebelum pasien saling tersesat
jika pasien pergi sendiri, ia sering tersesat dan mudah tersinggung dan mudah marah.
Hasil MMSE adalah 21, pemeriksaan fisik umum dan neurologis tidak ditemukan
kelainan tetapi terdapat penurunan ADL. Sebutkan Diagnosa pada kasus ini?
A. Demensia Alzheimer
B. Demensia vaskuler
C. Demensia campuran
D. Demensia frontotemporal
E. Demensia lewy bodies

31. Seorang wanita menjalani pemeriksaan mata karena keluhan.... setelah ditetesi
mydriatil, pasien mengeluh rasa nyeri, pandangan kabur disertai mual muntah. Apakah
diagnosis pada pasien tersebut?
A. Amaurosis fugax
B. Glaukoma sudut terbuka
C. Glaukoma sudut tertutup
D. Iritasi pada lensa mata
E. Bjjerum vield deffect
Topiramate menyebabkn glaucoma sudut tertutup

90
TO UGM 17 FEBUARI 2022
32. Seorang pria 65 tahun mengeluh sering menabrak benda di sekitarnya saat berjalan.
Pasien mengatakan sisi kanan dan kiri terlihat gelap mulai dari sisi bagian ke atas. Pada
pemeriksaan didapatkan hemianopsia bitemporal (lesi chiasma optikum). Apakah
diagnosa yang paling tepat pada kasus tersebut?
a. Sindroma sinus cavernosus
b. Meningioma sphenoid wing
c. Hydrocephalus obstruktif
d. Adenoma hipofisis  jawaban CW
e. Kraniofaringioma  ??????

33. Seorang wanita 65 tahun, mengeluh pandangan kabur. Saat dilakukan pemeriksaan
refleks cahaya pada mata kanan, pupil mata kiri dan kanan tidak konstriksi. Saat cahaya
diarahkan pada mata kiri, pupil mata kanan konstriksi. apakah diagnosis pada pasien
diatas?
A. Afferent pupilary defect dengan tropis d traktus optikus kanan
B. Efferent pupillary defect dengan tropis di nervus optikus kanan
C. Afferent pupilary defect dengan topik nervus optikus kanan (RAPD) relative aferen
pupillary defek
D. Afferent pupillary defect dengan tropis di nervus optikus kiri
E. Afferent pupillary defect dengan tropis di FINS surga kalkarina

34. Seorang wanita mengeluh mendadak buta pada pemeriksaan.. dengan BMI 30 kg. TD
180/90. Riw. DM dan dislipidemia, hasil funduskopi adalah sbg berikut. Apakah kelainan
pada gambar funduskopi diatas dan apakah patogenesis yang sesuai?

A. Oklusi arteri retina sentral (fovea cherry)


B. Mikro aneurisma A. Retina
C. Oklusi vena retina sentral
D. Oklusi A. Ciliaris posterior
E. Roth spot yang disebabkan oleh kardioemboli retinopati.

91
TO UGM 17 FEBUARI 2022
35. Apakah kelainan yang didapatkan dari pemeriksaan funduskopi dan kemungkinan
penyebabnya?

A. Tortous vein, dot and blots hemmorrage, disebabkan oleh oklusi vena retina sentral
B. Hollenhorts plaque, platelet fibrin emboli, disebabkan oleh oklusi arteri retina
superior
C. Fovea cherry red spot yang disebabkan oleh oklusi arteri retina central
D. Contoh wool patches yang disebabkan oleh obstruksi precapillary arteriole
E. Maccular star, penumpukan lipid disekitar fovea yang disebabkan oleh hipertensi
kronis dan DM

Macular star  hard exudat


Cotton – wool spot  soft exudat
36. Seorang wanita 56 tahun dengan riw. HT dan DM mengeluhkan mata kanan kabur.
Beberapa hari kemudian pasien merasa gelap saat melihat ke atas dan ke bawah yang
makin memburuk (attitudinal inferior). Hasil pemeriksaan RC didapatkan papilarry
defect. Hasil funduskopi didapatkan “flame shaped hemorrage. Diagnosa pada kasus
diatas ? D/ AION
A. Anterior ischemic optic neuropathy
B. Idiopatik intrakranial Hipertension
C. PAPILEDEMA due tO increased intracranial pressure
D. Optic neuritis
E. Stroke hemmoraghe

37. Seorang perempuan datang dengan keluhan pusing berputar dengan durasi < 1 menit
ketika pasien melakukan perubahan posisi dari duduk kemudian telentang dan saat
pasien membungkuk ke depan, pasien didiagnosa mengalami BPPV (canalis posterior).
Manakah pernyataan yang benar dibawah ini?

A. Nistagmus upbeat torsional ke sisi lesi (torsional = rotational)


B. Nistagmus upbeat torsional kontralateral lesi
C. Nistagmus geotropik
D. Nistagmus apogeotropik

92
TO UGM 17 FEBUARI 2022
E. Nistagmus unidireksional kontralateral lesi
Manuver diagnostic BPPV : tes Dix hallpike (kanal post) normal tidak timbul
nystagmus
Abnormal : timbul nystagmus dan vertigo berkurang saat manuver diulang

38. Seorang perempuan usia 48 tahun datang dengan keluhan pusing berputar dengan
durasi < 1 menit, pusing bertambah jika pasien melakukan perubahan posisi dari duduk,
kemudian telentang dan saat pasien membungkuk ke depan , ketika dilakukan manuver
ditemukan nistagmus yang sesuai dengan nistagmus posisinal ke sisi kiri. Apakah
penatalaksanaan yang paling tepat pada kasus tersebut?
A. Manuver epley dari sisi kanan
B. Manuver epley dari sisi kiri
C. Manuver semont dari sisi kanan
D. Manuver brand Daroff dari sisi kanan
E. Manuver lampert dari sisi kiri

39. Seorang perempuan datang dengan keluhan pusing berputar dengan durasi < 1 menit
ketika pasien melakukan perubahan posisi dari duduk kemudian telentang dan saat
pasien membungkuk ke depan, pasien didiagnosa mengalami BPPV. Terapi
medikamentosa yang sesuai pada kasus diatas
a. Flunarizine 15-20 mg/hari
b. Betahistine12 mg/hari
c. Betahistine 24 mg/hari
d. Betahistine 48 mg/hari
e. Flunarizine 10-15 mg/hari
40. Seorang perempuan 37 tahun mengeluh pusing berputar yang hilang timbul, pasien
tidak memperhatikan secara jelas frekuensi dan lamanya pusing. OS mengaku badan
terasa panas, mual dan beberapa kali muntah. Tidak ada gangguan pendengaran. Tidak
ada defisit neurologi. Pasien dilakukan pemeriksaan neurootologi. Apakah hasil
pemeriksaan neurootologi yang paling mungkin muncul pada kasus tersebut ? D/
A. Head impulse test negatif (sentral)
B. Nistagmus unidireksional (perifer)
C. Skew deviation positif (sentral)
D. Romberg dipertajam buka mata terjatuh
E. Finger tO nose terdapat dismetria (sentral)

KUK 2 edit kobe


1. Laki-laki, 50 tahun datang dengan kelemahan sisi kiri mendadak sejak 2 jam yang lalu. Riwayat
keluar dar iICCU 4 bulan yang lalu karena infark miokard. Riwayat operasi ORIF 2 bulan yang lalu.
Riwayat BAB hitam sejak 2 minggu SMRS. Pemeriksaa fisik : tensi 185/105 mmHg, GCS E3M6V5

93
TO UGM 17 FEBUARI 2022
dan lateralisasi kkiri. Pemeriksaan CT scan kepala terdapat infark di bawah ganglia basalis
desktra. Hasil lab GDS 350g/dl. Pasien akan direncanakan pemberian rTPA tetapi tidak
memenuhi syarat. Manankah faktor yang menyebabkan pasien batal mendapat rTPA?
A. TD
B. Infark miokard 4 bulan yang lalu
C. Riwayat operaso mayor 2 bulan yang lalu
D. Riw perdarahan GIT 2 minggu yang lalu
E. GDS >350 g/dl

2. Laki-laki usia 52 tahun datang ke poli saraf dengan keluhan mata kanan bengkak, kemerahan
disertai nyeri setelah mengalami KLL. Pada pemeriksaan dijumpai proptosis, kemosis, bruit
orbital dan oftalmoplegia. Tidak didapatkan kelainan pada motorik maupun sensorik.
Apakah pemeriksaan penunjang yang tepat untuk penegakan diagnosis pasien tsb?
a. MRI
b. Transcranial carotid doppler
c. CT scan cerebral
d. Difuuse weight imaging serebral
e. Digital substraction angiography

3. Wanita 75 tahun dibawa ke UGD dengan kelemahan anggota gerak kiri mendadaka saejak 2 hari
yang lalu. Terdapat riw DM tipe 2 HT tak terkontrol. EKG tidak menunjukkan AF.
Transesofageechochardiografi normal. CT angigrafi menunjukkan stensosis 50% lumen a carotis
eksterna kiri dan stenosi 50% pada lumen a carotis interna kanan . apakah terapi yang paling
tepat ?
a. Carotid end arterectomy
b. Antikoagulan
c. Trombektomi
d. Stenting arteri carotis eksterna kiri
e. Stenting a carotis interna kanan

4. Wanita, 47 tahun dibawa ke IGD dengan kelemahan anggota gerak kiri mendadak 6 jam yang
lalu. Terdapat riwayar DM tipe 2 HT tidak terkontrol. EKG tidak menunjukkan AF.
Transesofageechochardiografi normal. Pada pemeriksaan fisik didapatkan GCS E4M6V5 , nn
cranialis , n VII, n XII sinistra sentral, motorik 555/333 555/333, sensorik dbn. Hasil ct scan
kepala tidak didapatkan infark.
Tatalaksana neurointercensi yang tepat pada pasien ini
a. Coiling
b. Iv trombolisi

94
TO UGM 17 FEBUARI 2022
c. Ia trombolisis
d. Trombektomi
e. Carotid angioplasty dan stenting

5. Perempuan 65 tahun mengalami kelemahan sisi kanan, hal ini berlangsung sejak 3 jam yang
lalu. pasien juga mengeluhkan Bicara pelo + mulut mencong +. Pasien memiliki riwayat multiple
episodik TIA. Serangan TIA terakhir 1 bulan yang lalu. Riwayat HT-, DM –
Bagaimana penanganan paling tepat pada kasus ini?
a. Coiling
b. Ia trombolisis
c. Iv trombolisis
d. Trombektomi
e. Carotid angioplasty dan stenting

6. Laki-laki 28 tahun ddatang ke IGD RS dengan kelemahan sisi tubuh kanan secara mendadak 1
hari yang lalu. Tanda vital GCS E4M6Vx (afasia global), TD 130/80, nadi 72x/m regular (bruit -).
Motorik lateralisasi kanan. Sensibilitas dbn. Ekg dbn. CT scan kepala : infark trombotik akut
subkortikal frontoparietal kiri. Pemeriksaan DSA sesuai gambar (moya-moya)  smoke puuf sign

Apakah diagnosis yang paling tepat :


a. Cavernous angioma
b. Trombosis vena serebri
c. Reversible cerebral vasocontriction syndrome
d. vasospasme
e. moyamoya disease

7. laki-laki 34 tahun datang ke IGD karena mengalami penurunan kesadaran yang disertai nyeri
kepala hebat mendadak sejak 2 jam sebelum MRS. Pada pemeriksaan didapatkan GCS
E2M4V3(9), TD 150/90, HR 110, RR 22, temp 36,2. Pupil bulat isokor 3mm/3mm. CT scan kepala

95
TO UGM 17 FEBUARI 2022
menunjukkan suatu gambaran PSA pada regio frontalis. Pasien direncanakan untuk dilakukan
pemeriksaan CT angiography untuk mencari etiologi.
Di manakah lokasi aneurisma intraserebral tersering pada kasus pasien tsb?

a. a komunikans anterior (abulia + weakness leg)


b. a. Komunikans post
c. bifurcatio dari arteri karotis interna dan MCA
d. pertemuan antara arteri komunikans post dengan a kartois interna
e. a basilaris distal

PCA : parese N III + ataxia

MCA : hemiparese lemah tangan dari pada kaki

ACA : sindrom lobus frontal

8. laki laki 50 tahun datang dengan kelemahan sisi kiri mendadak sejak 2 jam yang lalu. Pemeriksaa
fisik : tensi 185/105 mmHg, GCS E3M6V5 dan lateralisasi kkiri. Pemeriksaan CT scan kepala
terdapat infark di bawah ganglia basalis desktra.
Di manakah letak lesi vaskular berdasarkan hasil pemeriksaan CT scan kepala pasien
a. bifurcatio a cerebri media
b. arteri lenticulostriata
c. arteri thalamoperforata
d. arteri choroidal anterior
e. arteri comunicans anterior
9. wanita 47 tahun dibawa ke poli saraf dengan kelemahan anggota gerak kiri sejak 1 th yang lalu.
Pada pemeriksaan fisik didapatkan GCS E4M6V5 nn cranialis parese n vii dan n xii sinistra
sentral, motorik 555/444 555/444, sensorik dbn. Pasien rutin meminum obat CPG 75 mg/24
jam.
Apakah efek samping pemberian obat jangka lama?

A. Neutropenia
B. Trombositopenia purpura : sifat antiagregasi CPG disebabkan o/penghambatan pengikatan
ADP ke reseptor Trombositnya (cuit 157)
C. Vasodiltasai perifer
D. Leukopenia
E. Anemia

10. Pria 38 tahun datang ke uGD dengan keluhan kejang menghentak-hentak sejak 1 yang lalu.
Kejang terjadi sejak 5 menit, saat kejang pasien tidak sadarkan diri. Setelah kejang pasien
tampak ,mengantuk dan bingung, pasien mengatakan sakit kepala hebat mendadak saat sedang
beraktivitas. Mual – muntah -, pasien menyangkal adanya HT, DM. Dari pemeriksaan didapat

96
TO UGM 17 FEBUARI 2022
kaku kuduk ringa, nn cranialis dbn. Kekuatan 555/555 555/555, sensorik otonom dbn. Ct scan
kepala menunjukkan PSA.
Apa penyebab terjadinya kejang?
a. Stealing phenomen
b. Sentinel bleed
c. Vasopasme
d. Intrapranchymal extention
e. SIADH

11. Pria 38 tahun datang ke IGD dengan keluhan kejang kelojotan 1 jam yang lalu. Kejang terjadi
selama 5 menit, saat kejang pasien tidak sadarkan diri. Setelah keajng pasien tampak mengantuk
dan bingung. Pasien mengatakan sakit kepala hebat mendadak saat sedang beraktivitas. Mual –
muntah -, pasien menyangkal adanya HT DM. Dari pemeriksaan didapatkan GCS E3M5V3, kaku
kuduk (+), nn cranialis dbn, kekuatan motorik 444/555 444/555, sensorik dan vegetatif dbn.
Berapakah derajat klinis pada pasien berdasarkan Hunt and Hess
a. Grade I
b. Grade II
c. Grade III : kaku kuduk +, mild fokal deficit, drownies,
d. Grade IV :stupor
e. Grade V : koma dalam

12. Pria 38 tahun datang ke IGD dengan keluhan sakit kepala hebat mendadak saat sedang
beraktivitas. Tidak didapatkan mual, muntah, kelemahan anggota gerak, kesemutan, kejang
maupun penurunan kesadaran. Dari pemeriksaan fisik didapatkan TD 180/100, pemeriksaan
neurologis didapatkan kaku kuduk, nn cranialis dbn, kekuatan motorik 555/555 555/555,
sensorik normal (PSA)
Apakah penyebab keluhan nyeri kepala pada pasien?
A. Stealing phenomen
B. Sentinele bleed
C. Sirkulasi kolateral
D. Intraparenchymal extention
E. Peningkatan glutamat pada korteks cerebral

97
TO UGM 17 FEBUARI 2022
13. Pria 40 tahun datang ke IGD dengan keluhan sakit kepala hebat mendadak saat sedang
beraktivitas. Tidak didapatkan mual, muntah, kelemahan anggota gerak, kesemutan, kejang
maupun penurunan kesadaran. Dari pemeriksaan fisik didapatkan TD 180/100, pemeriksaan
neurologis didapatkan GCS E4M6V5, kaku kuduk (+), nn cranialis dbn, kekuatan motorik 555/555
555/555, sensorik normal
Berapakah derajat klinis berdasarkan Hunt and Hess?
A. Grade I
B. Grade II
C. Grade III
D. Grade IV
E. Grade V

14. Pria 40 tahun datang ke IGD dengan keluhan sakit kepala hebat mendadak saat sedang
beraktivitas. Tidak didapatkan mual, muntah, kelemahan anggota gerak, kesemutan, kejang
maupun penurunan kesadaran. Dari pemeriksaan fisik didapatkan TD 180/100, pemeriksaan
neurologis didapatkan GCS E4M6V5, kaku kuduk (+), nn cranialis dbn, kekuatan motorik 555/555
555/555, sensorik normal
Apakah usulan pemeriksaan penunjang pada kasus tsb?
A. Pungsi lumbal
B. Tcd
C. Ct scan kepala nonkontras :
D. Mri kepala
E. dsa
15. perempuan 47 tahun dibawa ke poli saraf dengan kelemahan anggota gerak kanan sejak 1
tahun yang lalu. Terdapat riwayat DM yang tidak terkontrol. EKG tidak AF. Pada pemeriksaan
fisik didapatkan GCS 15, parese n VII XII dextra, motorik 444/555 444/555 sensorik normal. Hasil
dsa didapatkan gambaran sbb.

98
TO UGM 17 FEBUARI 2022
Apakah diagnosis yang paling mungkin
a. Cerebral sinus venous trombosi
b. CCF
c. Dural AF fistula
d. Aneurisma intrakranial
e. AVM

16. Perempuan 28 tahun telah didiagnosis epilepsi sejak usia 17 th. Selama ini pasien mendapat as
valproat 2x250 mg dan fenitoin 3x100 mg dari dokter sebelumnya. Saat ini pasien berobat ke
anda pertama kali dan dalam kehamilan 12 minggu. Bentuk kejang bengong selama 1 menit.
Dengan 2 obat tsb, pasien masih kejang 1-2 x per bulan.
Apa tatalaksana yang akan anda rencanakan pada pasien ini?
a. Menghentikan as valproat dan meneruskan fenitoin, serta memberikan as folat 1-5 mg/hari
b. Tidak mengganti obat selama kehamilan dan memberikan as folat 1-5 mg/hari
c. Mengganti as valproat dengan lamotrigine dan menuruskan fenitoin, serta memberikan
asam folat 1-5 mg/hari
d. Mengganti asam valrpaot dengan lamotrigine dan mengganti fenitoin dengan
carbamazepine, serta memberikan asam folat 1-5mg/hari
e. Menghentikan fenitoin dan meneruskan asam valproat dengan menaikkan dosis menjadi
2x500mg

17. Laki-laki 18 tahun datang ke IGD dengan keadaan kejang berulang. Pasien telah didiagnosis
epilepsi dengan obat fenitoin 3x100 mg. Pasien telah bebas kejang selama 1 tahun namun tiba-
tiba pasien mengalami kejang 3x dalam waktu 1 hari.
Tiga hari terakhir pasien terlihat murung dan sedih karena tidak terima di PTN. Pasien hanya
mengurung diri di kamar.
Pada saat datang di UGD pasien terlihat kejang kemnali.

99
TO UGM 17 FEBUARI 2022
Pada pemeriksasan fisik didapatkan tanda vital normal. Suhu afebris. GCS E4M6V4 (disorientasi),
nn cranialis dan motorik kesan dbn.
Apakah pemeriksaan penunjang awal yang harus dilakukan
a. Pemeriksaan kadar obat dalam darah
b. Ct scan kepala dengan kontras
c. EEG
d. MRI kepala tanpa kontras
e. Pemeriksaan elektrolit, darah lengkap

18. Laki-laki 18 tahun datang ke IGD dengan keadaan kejang berulang. Pasien telah didiagnosis
epilepsi dengan obat carbamazepine 3x200 mg. Pasien telah bebas kejang selama 1 tahun
namun tiba-tiba pasien mengalami kejang 3x dalam waktu 1 hari.
Pada saat datang di UGD pasien terlihat kejang kembali. Semiologi : tiba-tiba terdiam, mata dan
wajah tertarik ke sisi kiri, tangan kiri kaku menekuk, kemudian menjadi kaku seluruh tuibuh,
diikuti kejang klonik seluruh tubuh, tidak sadar, durasi <5 menit. Setelah kejang pasien terlihat
bingung. Menurut keluarga, pasien tidak pernah sadar penuh seperti semula. GCS E4M6V4
(disorientasi)
Apa diagnosis kerja pada pasien tsb?
a. Status epilepstikus
b. TLE
c. FLE
d. Acute symptomatic seizure
e. Partial continua

19. Anak laki-laki 10 tahun dibawa ke IGD dengan keluhan kelemahan seluruh esktremitas kesan
memberat sejak 5 hari lalu. TB 120 cm, BB 30 kg. Tiga minggu yang lalu pasien dikabarkan
mengalami demam tinggi 39 derajat, diare cair 5x dalam sehari, demam disertai nyeri kepala
hebat. Pasien dibawah ke PKM kemudian diberi obat namun tidak ada perbaikan. Setelah itu
menurut orang tua pasien, pasien tampak lemah terkulai keempat ekstremitas kesan bersamaan
dan tampak spasme otot. Pada pemeriksaan fisik GCS E4M6V5, tetraparese flaccid LMN, tidak
ada gangguan sensorik dan otonom. Pasien berasal dari daerah kumuh padat penduduk,
pekerjaan orang tua buruh serabutan, imunisasi tidak lengkap.
Apkah etiologi penyakit tsb?
a. Infeksi virus RNA enterovirus family picornavirus (A)
b. Infeksi bakteri Corynobacterium diphteria
c. Infeksi M tuberculosis
d. Post infeksi campylobacterium jenjuni dilanjutkan dengan proses autoimun
e. Infeksi jamur mucormycosis

100
TO UGM 17 FEBUARI 2022
20. Anak laki-laki 10 tahun dibawa ke IGD dengan keluhan kelemahan seluruh esktremitas kesan
memberat sejak 5 hari lalu. TB 120 cm, BB 30 kg. Tiga minggu yang lalu pasien dikabarkan
mengalami demam tinggi 39 derajat, diare cair 5x dalam sehari, demam disertai nyeri kepala
hebat. Pasien dibawah ke PKM kemudian diberi obat namun tidak ada perbaikan. Setelah itu
menurut orang tua pasien, pasien tampak lemah terkulai keempat ekstremitas kesan bersamaan
dan tampak spasme otot. Pada pemeriksaan fisik GCS E4M6V5, tetraparese flaccid LMN, tidak
ada gangguan sensorik dan otonom. Pasien berasal dari daerah kumuh padat penduduk,
pekerjaan orang tua buruh serabutan, imunisasi tidak lengkap.
Apakah tahapan gejala penyakit yang dialami oleh pasien tsb?
a. Polineuropati sensorik dan motorik tipe demilienisasi
b. Polimielitis paralitik (B)
c. Polineuritis diphteria
d. Polimielitis ensefalitis
e. Werdnigg-hoffmann disease

21. Laki-laki 60 tahun datang ke IGD dengan keluhan kehilangan penglihatan tiba-tiba mata kanan 6
jam yang lalu. Kelihan terjadi selama 1 jam lalu pulih total. Riwayat pasien pernah mengalami
keluhan serupa 1 tahun yang lalu, dengan durasi 30 menit dan pulih sempurna. Pasien memiliki
HT Dan DM, terkontrol dengan obat HT dan obat hipoglikemik oral. Pasien mengkomsumsi ASA
1x100 mg secara rutin. (Kanan BMT - sudah dapat)
Dari hasil pemeriksaan pencitraan pembuluh darah didapatkan adanya oklusi pada a karotis
interna kanan pada segmen ekstrakranial dan stenosis 72% pada a karotis interna kiri segmen
C1.
Tatalaksana yang paling tepat :
a. Carotid artery stenting ICA kanan
b. Carotid stenting ICA kiri
c. Carotid end arterectomi ICA kanan
d. Terapi konservastif dengan double antiplatelet, statin, dan faktor resiko lain
e. Terpai konservatif dengan antikoagulan, statin dan kontrol

22. Laki-laki 76 tahun, pensiunan PNS, datang dengan gangguan daya ingat secara perlahan-lahan
sejak 4 tahun yang lalu. Istri pasien mencatat sejak awal perubahan, pasien kadang lupa
menaruh barang. Pasien sering menanyakan sesuati yang telah ditanyakan beberapa menit
sebelumnya. Pasien juga pernah menggunakan pakaian secara terbalik (pakaian dalam di luar).
Akhir-akhir ini, pasien sering pergi dan lupa jalan pulang fisik umum dan fisik neurologis tidak
ditemukan kelainan. Pada pemeriksaan neurobehaviour menunjukkan tidak ada gangguan pada
domain atensi dan bahasa. Terdapat gangguan apada domain memori. Penurunan ADL/IADL.
Di manakah lokasi lesi utama pada kondisi gangguan memori pasien ini? Alzheimer Disease
a. Lobus frontalis basalis, lobus tempral media, hipokampus
b. Lobus temporal medial, hipokampus, perisylvian
c. Lobus parietalis hemisfer dominan, perisilvian, hipokampus
d. Lous parietalis hemisfer nondominan, gangliua basalis, hipokampus

101
TO UGM 17 FEBUARI 2022
e. Lobus tempral medial, perisilvian, ganglian basalis

23. Laki-laki 76 tahun, pensiunan PNS, datang dengan gangguan daya ingat secara perlahan-lahan
sejak 4 tahun yang lalu. Istri pasien mencatat sejak awal perubahan, pasien kadang lupa
menaruh barang. Pasien sering menanyakan sesuati yang telah ditanyakan beberapa menit
sebelumnya. Pasien juga pernah menggunakan pakaian secara terbalik (pakaian dalam di luar).
Akhir-akhir ini, pasien sering pergi dan lupa jalan pulang fisik umum dan fisik neurologis tidak
ditemukan kelainan. Pada pemeriksaan neurobehaviour menunjukkan tidak ada gangguan pada
domain atensi dan bahasa. Terdapat gangguan apada domain memori. Penurunan ADL/IADL.
Manakah yang merupakan gejala dini pada jenis demensia ini?
a. Memori
b. Apraxia agnosia
c. Bahasa
d. Abstraksi
e. paratonia

24. perempuan 80 tahun, IRT diantar ke poli oleh keluarganya dengan keluhan cenderung lebih
sering menyendiri di kamar dibandingkan berkumpul dan bercakap bersama keluarga. Sering
terlihat murung, sering lupa menaruh barang-barang di rumah, dan inisitiatif untuk
mengerjakan sesuatu berkurang. Hal ini dialami sejak 1 tahun yang lalu, sebelumnya pasien
telah berobat karena mengalami keluhan sering gemetar, langkah kecil-kecil jika melangkah
dan jalan lamban sejak 10 tahun lalu. Pasien meminum obat secara teratur.
Apakah diagnosis pada pasien ini?
a. Dmensia alzherimer
b. Parkinson disease dementia
c. Mild cognitive impairement
d. Depresi
e. Demensa kortikal
25. Wanita 45 tahun dibawa ke poli dengan perubahan perilaku yang diamati keluarganya sejak
pasien mengalami kecelakaan 3 bulan yang lalu. Saat ini pasien
Tentukan pemeriksaan neurobehaviour yang tepat pada diangosis ini
a. Helstead-reitan tactual performance test
b. Visual memory
c. Lhermitte signoret amnesic syndrome
d. Reitan trail making test
e. Color naming

26. Seorang perempuan 57 tahun iRT datang ke klinik memori diantar oleh anaknya dengan keluhan
mudah lupa, sulit mengenal kembali anggota keluarganya. Sehari-hari pasien tampak tidak
semangat, tidak ada gairah hidup, dan selalu diam. Riwayat ia dirawat di RS sekitar 8 bulan yang
lalu dengan stroke perdarahan. Pasien saat ini tidak dapat menggerakan anggota gerak
kanannnya sama sekali. Sejak saat itu ia tidak dapat lagi melakukan aktivitas keseharian dan

102
TO UGM 17 FEBUARI 2022
pekerjaan rutin sebelumnya sekalipun masih dapat berjalan. Ia mempunyai riwayat hipertensi
sejak 2 tahun yang lalu, berobat tidak teraktur. Dx. Demensia Vaskular
Bagaiman patofis pada kasus ini
a. Terjadi stimulasi glutamat yang berlebihan pada respetor NMDA
b. Terjadi stimulasi GABA yang berlebihan pada resptor GABA
c. Terjadi kematian sel, degradasi DNA, fagositosis
d. Terjjadi penumpukan protein tau
e. Terjjadi penumpukan Beta amyloid

27. Seorang perempuan 57 tahun iRT datang ke klinik memori diantar oleh anaknya dengan keluhan
mudah lupa, sulit mengenal kembali anggota keluarganya. Sehari-hari pasien tampak tidak
semangat, tidak ada gairah hidup, dan selalu diam. Riwayat ia dirawat di RS sekitar 8 bulan yang
lalu dengan stroke perdarahan. Pasien saat ini tidak dapat menggerakan anggota gerak kanannnya
sama sekali. Sejak saat itu ia tidak dapat lagi melakukan aktivitas keseharian dan pekerjaan rutin
sebelumnya sekalipun masih dapat berjalan. Ia mempunyai riwayat hipertensi sejak 2 tahun yang
lalu, berobat tidak teratur. Demensia vaskular
Apakah terapi farmakologis untuk kasus di atas?
a. Donepezil rivastigmin galantamine
b. Donepezil rivastigmin memantine
c. Ginko biloba galamtamin donepezil
d. Citicholine rivastigmin donapezil
e. levodopa donepezil galantamin

28. laki-laki 60 tahun dibawa keluarganya ke klinik memori dengan keluhan utama gangguan
perilaku, mood sering berubah-ubah, namun paling sering tampak cemas dan depresi tanpa
penyebab jelas. Setelah dilakukan beberapa pemeriksaan neuropsikiatri, pasien didiagnosis
demensia.
Manajemen non farmakologis yang dapat dianjurkan pada pasien ini?
a. Terapi reminiscence
b. Akupuntur
c. Terapi visual reality
d. Biofeedback
e. TENS

29. Laki-laki 74 tahun pensiunan pegawai. Dilakukan pemeriksaan di klinik memori karena
keluhan mudah lupa akan cerita atau perisitwa yang baru-baru saja terjadi, termasuk sering
mengulang-ulang cerita dan mudah lupa letak barang yang baru saja disimpan seperti jam tangan
dan kunci. Keluhan diala,i sejak 6 bulan yang lalu secara tiba-tiba. Pasien masih dapat melakukan
aktiivtas sehari=haro seperti bpergian dan berbelanja. Ada riwayat HT dan DM sejak 2 0

103
TO UGM 17 FEBUARI 2022
tahuyn yang allu, namun tidak berobat teratur. Didapatkan hasil pemeriksaan MMSE 27/30,
CDR clinical demensia rating 0,5, geriatric depression scale (GDS) 0/15
Kemungkinan diagnosis pasien di atas?
a. Demensia alzherimer
b. Demensia vaskular
c. Mixed dementia
d. Vascular mild cognitive impairement
e. Dementia frontotemproal

104
TO UGM 17 FEBUARI 2022
CDR
30. Laki-laki 79 tahun dirawat di bangsal saraf, dirawat dengan keluhan nyeri kepala disertai gelisah
dan mengamuk. Dalam perawatan, pasien tampak aneh, sering melihat bunga dan roti berada di
sekitar tempat tidur pasien. Pasien mengalami gangguan fluktuasi kognitif. Tidak ada riwayat
diagnosis gangguan psikiatri sebelumnya. Salah satu jenis obat yang dikonsumsi secara rutin

105
TO UGM 17 FEBUARI 2022
beberapa waktu terakhir adalah carbidopa 50 mg. Pada pemeriksa neuropsikologi, didapatkan
gangguan fungsi eksekutif dan visuospasial. Dx. DLB
Obat apa yang perlu diberikan untuk mengatasi gangguan fungsi kognitif pad apasien tsb?
a. Donepezil
b. Rivastigmin
c. Ginkgo biloba
d. Memantine
e. Rivastigmine : krn ada halusinasi

31. seorang wanita 20 tahun datang dibawa orang tuanya ke IGD RS dengan keluhan demam, nyeri
kepala, muntah, penglihatan mata kanan menjadi kabur. Dalam 1 minggu visus pasien 1/300.
Hasil lab leukositosis 14.000. gambaran EEG menunjukkan voltase tinggi dengan aktivitas
lambat di daerah temporal kanan. Hasil MRI otak diperlihatkan di gambar di bawah ini : (a) Axial
flair MRI, (b) axial T1 weigthed post kontras MRI (c) axial difussion weighet MRI.

apakah diagnosis yang paling tepat pada pasien ini?


a. Cerebritis
b. Lymne diasese
c. Abses cerebri
d. Metastasis lesion
e. Toxoplasmosis

32. Laki-laki 19 tahun datang ke IGD dengan kejang. Kejang ini sudah dialami sejak pasien 2 bulan
ini. Kejang bersifat general menghentak-hentak. Riwayat HIV positif serta minum obat ARV
tidak teratur. Pemeriksaan neurologis GCS E3V4M5, tensi 110/70 suhu 36,3. Gambaran CT scan
kepala dijumpai multiple lesi dengan ring enchancement.
Manakah obat yang paling tepat?
a. Pirimethamine 50 mg lanjut 25 mg/hari
b. Pirimethamine 100 mg lanjut 50 mg/hari
c. Pirimethamine 150 mg lanjut 75 mg/hari
d. Pirimethamine 200 mg lanjut 100 mg/hari
e. Pirimethamine 250 mg lanjut 125 mg/hari

106
TO UGM 17 FEBUARI 2022
33. Laki-laki 42 tahun pekerjaan membersihkan kandang sapi. Dibawa ke poliklinik neurologi
dengan keluhan kaki kiri terluka akibat tersabit parang. Luka ini tidak dibersihkan dan dibiarkan
sejak 2 hari ini. Setelah seminggu dijumpai demam tinggi serta mulut yang terkunci serta kejang
rangsang. Manakah patofisiologi yang mendasari penyakit
a. Disfungsi otonom komplikasi dari tetanus
b. The heavy chain toxin tetanus memblok exositosis
c. Timbulnya spasme disebabkan blokade sistem simpatis
d. Tetanospasmin menginhibisi pelepasan glutamat medula spinalis
e. Toxin tetanus menginhibisi pelepasan GABA medula spinalis

34. Perempuan 25 tahun dengan keluhan penurunan kesadaran sejak sekitar 5 hari lalu perlahan-
lahan. Keluhan disertai riwayat demam 1 minggu hilang timbul dan kelemahan anggota gerak
kanan. Pada pemeriksaan ditmeukan GCS E3M5V3=11, tidak ada kaku kuduk, tes HIV +, CD4
150 sel/mm3. Pada pemeriksaan funduskopi didapatkan papil edema bilateral. Foto thorax
bronkitis. CT scan kepala kontras : massa menyangat kontras multipel dengan edema
perifokal di lobus frontal kiri dan ganglia basal kiri.
Apakah diagnosis kerja yang paling mungkin
a. Meningitis
b. Tuberkuloma (4S Small, subcotical, solid enhance, surrounding edema)
c. Meningitis kriptiokokus  tdk ada gbr khusus, biasanya atrofi,
d. Ensefalitis herpes
e. Ensefalitis toxoplasma

35. Wanita 25 tahun datang ke poli dengan vertigo akut yang telah berlangsung 3 hari. Disertai
dengan mual dan muntah sampai beberapa jam. Dijumpai disequilobrum, instabilitas gait, dan
gangguan visus selama 2 hari. Dijumpai imbalance di saat gelap dan disorientasi ruang.
Kehilangan pendengaran/tinitus dan rasa penuh pada telinga disangkal. Penderita sebelumnya
mengalami penyakit flu yang berlansung selama 2 minggu sebelum vertigo. Riwayat penggunaan
obat disangkal.
Pemeriksaan neurologis tidak didapatkan defisit neurologis, dijumpai nistagmus torsional
horizontal ke kiri pada saat melirik ke kiri. tes head thrust dijumpai sakadik setelah rotasi kepala
dari kiri ke kanan. Tes audiometri normal.
Apakah diangosis yang paling tepat pada kasus ini? AVS ec Neuritis
a. Bppv kanalis posterior
b. Bppv kanalis horizontal
c. Meniere disease
d. Neuritis vestibularis (right ear)
e. Labirinitis : harus ada gangguan pendengaran

36. Wanita 25 tahun datang ke p;oli dengan vertigo akut yang telah berlangsung 3 hari. Disertai
dengan mual dan muntah sampai beberapa jam. Dijumpai disequilobrum, instabilitas gait, dan
gangguan visus selama 2 hari. Dijumpai disequilobrum, instabilitas gait, dan gangguan visus
selama 2 hari. Dijumpai imbalance di saat gelap dan disorientasi ruang. Kehilangan
pendengaran/tinitus dan rasa penuh pada telinga disangkal. Penderita sebelumnya mengalami

107
TO UGM 17 FEBUARI 2022
penyakit flu yang berlansung selama 2 minggu sebelum vertigo. Riwayat penggunaan obat
disangkal.
Pemeriksaan neurologis tidak didapatkan defisit neurologis, dijumpai nistagmus torsional
horizontal ke kiri pada saat melirik ke kiri. Pada saat melirik ke kanan didapatkan nistagmus
torsional horizontal ke kiri. tes head thrust dijumpai sakadik setelah rotasi kepala dari kiri ke
kanan. Audiometri normal.
Tipe nistagmus yang muncul pada kasus ini
a. Tipe geotropik
b. tipe apogeotripik
c. tipe bidirektional
d. tipe undirektional
e. tipe rotasional

37. Wanita 25 tahun datang ke p;oli dengan vertigo akut yang telah berlangsung 3 hari. Disertai
dengan mual dan muntah sampai beberapa jam. Dijumpai disequilobrum, instabilitas gait, dan
gangguan visus selama 2 hari. Dijumpai disequilobrum, instabilitas gait, dan gangguan visus
selama 2 hari. Dijumpai imbalance di saat gelap dan disorientasi ruang. Kehilangan
pendengaran/tinitus dan rasa penuh pada telinga disangkal. Penderita sebelumnya mengalami
penyakit flu yang berlansung selama 2 minggu sebelum vertigo. Riwayat penggunaan obat
disangkal.
Pemeriksaan neurologis tidak didapatkan defisit neurologis, dijumpai nistagmus torsional
horizontal ke kiri pada saat melirik ke kiri. Pada saat melirik ke kanan didapatkan nistagmus
torsional horizontal ke kiri. tes head thrust dijumpai sakadik setelah rotasi kepala dari kiri ke
kanan. Audiometri normal.
Klasifikasi nistagmus
a. Nistagmus kongenital
b. Spontaneuos
c. Gaze-evoked
d. Posisional
e. Vestibular

38. Seorang wanita 50 tahun datang ke poliklinik saraf dengan keluhan vertigo akut sejak 1 bulan
disertai rasa penuh pada telinga kanan disertai perasaan bunyi, namun menghilang dalam
beberapa hari. Durasi vertigo 30 menit dan keluhan telah berulang 5x dalam 1 bulan ini. Keluhan
muncul tanpa pencetus yang jelas.
Pemeriksaan neurologis dan otoskopik normal. Tes weber dijumpai lateralisasi ke kiri dan tes
Rinne kanan kiri positif. Pemeriksaan neurootogik tidak dijumpai nistagmus spontan, test head
thrust normal. Bagaimana mekanisme pada kasus ini? EVS-S ec meniere disease
a. Adanya free-flating debris pada kanalis semisirkularis
b. Atrofi nervus vestibularis
c. Ruptur membranous labirin
d. Infark arteri serebelar anterior inferior

108
TO UGM 17 FEBUARI 2022
e. Neuroma akustik

39. Pria 20 tahun dengan keluhan sakit kepala setelah kepala terbentur aspal akibat KLL 5 hari
SMRS. Saat kejadian, ia pingsan sekitar 15 menit. Pada pemeriksaan fisik neurologis dijumpai
kesadaran menurun GCS 13 dan amnesia paska trauma selama lebih kurang 3 jam. Segera
setelah trauma kepala pasien bingung dan tidak ingat saesaat sebelum dan sesudah kejadian
nyeri kepala. Pemeriksaan imejing tidak dijumpai ICH maupun kontusio cerebri.
Diagnosis yang paling mungkin
a. Nyeri kepala akut paska trauma terkait trauma kapitis ringan (dalam 7 hari)
b. Nyeri kepala akut paska trauma terkait trauma kapitis sedang
c. Nyeri kepala persisten paska trauma terkait trauma kapitis ringan (>3bulan)
d. Nyeri kepala persisten paska trauma terkait trauma kapitis sedang
e. trauma kapitis ringan

40. wanita 24 tahun keluhan sakit kepala berdenyut sebelah kanan dengan insentias berat. Nyeri
kepala seperti ini sudah dialami sejak 7 bulan terakhir dengan frekuensi sekitar 16 hari/bulan.
Durasi sekitar 24 jam setiap kali serangan. Nyeri kepala bertambah berat dengan aktivitas rutin.
Kadang nyeri kepala disertai mual dan muntah, pemeriksaan neurologis tidak dijumpai kelainan
Obat abortif spesifik yang paling sesuai
a. Naproxen sodium 275-550mg/2-6jam/hari
b. Sumatriptan 50-100 mg
c. Ibuprofen 400-800 mg/6 jam
d. Asetaminofen 1000 mg
e. Metilprednisolon 4 mg/8 jam

41. Pria 63 tahun datang dengan keluhan sakit kepala. Sakit kepala biasanya mengalami perbaikan
bila pasien minum obat anti HT. Saat diperiksa TD 175/90 mmHg. Pasien diketahui penderita HT
selama ini. Pada pemeriksaan fisik tidak dijumpai kelainan.
Diagnosis yang paling sesuai
a. Nyeri kepala berkaitan dengan HT arterial (TD >180/120)
b. Nyeri kepala berkaitan krisis HT
c. Nyeri kepala berkaitan ensefalopati hipertesnif
d. Nyeri kepala berkaitan dengan HT ringan
e. Nyeri kepala berkaitan dengan kelainan homeostatis

42. Laki-laki 79 tahun datangdengan keluhan tremor. Didapatkan bradikinesia, tremor, rigiditas
bilateral, namun lebih berat pada tangan kanan. Istrinya mengatakan pasien mengantuk
sepanjang hari. Dopamin agoonist bikin ngantuk
Terapi yang paling tepat
a. Ropinirole

109
TO UGM 17 FEBUARI 2022
b. Carbidopa-levodopa
c. Pramipexole : dibawah 60 thn
d. THP
e. Bromokriptine

43. Wanita 34 tahun dibawa suaminya ke poli setelah diketahui mengalami kesulitan membaca
sejak 3hari lalu. Pemeriksaan neurologis lengkap tidak ditemukan defisit neurologis selain
gangguan tsb. Pasien mengalami kesulitan saat disuruh menuliskan atau menirukan kata-kata
yang ditunjuk kepaldanya. Namun masih mampu menulis spontan.
Berdasarkan gejala klinis, kemungkinan letak lesi
a. Korpus callosum posterior cocok (alexia without agrafia)
b. Girus cinguli kiri
c. Girus angularis kiri
d. Lobus oksipital posterior kiri
e. Lobus oksipital bilateral

44. Seorang anak membawa orang tua laki-laki ke poli karena mengalami sulit bicara. Saat
diwawancara, pasien tampak kesulitan menjawab pertanyaan,bahkan terkadang pasien hanya
mengangguk atau menggeleng saja ketika ditanya. Sesekali pasien mengulang kata-kata yang
ditanyakan oleh pemeriksa dengan kata-kata yang lancar (repetisi baik). Saat diperintah,
menyebutkan nama benda pena yang ditunjukkan, pasien terlihat sulit untuk menyebutkan.
Tetapi jika ditanya apakah tau nama benda tsb, pasien mengangguk. Kemampuan menulis
terganggu. Pasien direncanakan untuk pemeriksaan MRI kepala.
Gangguan apakah yang dialami pada pasien di atas
a. Motorik
b. Transkortikal motorik : ACA-MCA watershed
c. Transkortikal campuran
d. Afasia konduksi
e. Afasia anomik

45. Laki-laki 55 tahun ke poli karena mengalami kesulitan berbicara sejak 1 minggu. Saat
diwawancara, pasien tampak kesulitan untuk menjawab pertanyaan, bahkan terkadang pasien
hanya mengangguk/menggeleng saja jika ditanya. Sesekali pasien mengulang kata-kata yang
ditanyakan oleh pemeriksa dengan kata-kata yang lancar(repetisi baik). Saat diperintah,
menyebutkan nama benda pena yang ditunjukkan, pasien terlihat sulit untuk menyebutkan.
Tetapi jika ditanya apakah tau nama benda tsb, pasien mengangguk. Kemampuan menulis
terganggu. Pasien direncanakan untuk pemeriksaan MRI kepala. Transkortikal motoric : area
motor tambahan , gyrus cingularis
Berdasarkan gejala di atas, perkiraan letak lesi yang ditemukan
a. daerah kortikal yang luas dan menyisakan korteks perisilvian kiri
b. fasikulus arkuata pada operkulum parietal kiri - konduksi
c. Area motor tambahan dan girus singuli yang berdekatan hemisfer kiri(dominan)

110
TO UGM 17 FEBUARI 2022
d. Temporalis posterior kiri
e. Girus frontal inferior dan daerah di dekat operkulum serta insula kiri – afasia motorik

46. Wanita 56 tahun dibawa ke poliklinik neurologi paska perawatan stroke 2 minggu yang lalu. Saat
diwawancara, pasien masih kesulitan berbicara. Pasien dinilai mampu berkomunikasi dua arah,
termasuk mengeluarkan kata-kata bahkan kalimat, namun sesekali pasien salah menyebutkan
beberapa kata dengan kata yang bunyinya sama, namun tidak ada arti. Pasien terkadang
menyadari kesalahannya dan mengeluhkan hal tersebut, seperti menyebut “rumah” dengan
“dunah” (Parafasia fonemik). Repetisi dinilai terganggu, saat disuruh membaca, pasien
mengatakan mengerti apa yang dibaca, namun sulit membacakan dengan suara keras.
Berdasarkan gejala di atas : Afasia konduksi
a. Afasia motorik : MCA SUP, lubus frontal inferolateral
b. Transkortikal motorik : area motoric tambahan & girus cyngularis
c. Transkortikal sensorik : MCA post , gyrus angularis dominan, temporal
d. Konduksi : MCA post, fasikulus arkuatus, pada operculum parietal kiri
e. Anomik : MCA post, gyrus angularis kiri, temporalis anterior/posterior

47. Laki-laki 62 tahun ke IGD karena kelemahan sesisi kanan tubuh mendadak 1 jam yang lalu. Saat
diwawancara, pasien dinilai mampu berkomunikasi 2 arah termasuk mengeluarkan kata-kata
bahkan kalimat. Namun sesekali pasien salah menyebutkan beberapa kata dengan kata yang
bunyinya sama namun tidak ada arti, seperti menyebut “mobil” dengan “dotil”. Ketika disuruh
mengulang kata-kata yang disebutkan, pasien tampak kesulitan. Saat disuruh membaca, pasien
tidak mampu membaca dengan suara keras. D :/ afasia konduksi
Gambaran CT scan yang kemungkinan ditemukan adalah
a. Iskemik pada girus forntal interior dan daerah di dekat operkulum serta insula kiri
b. Iskemik pada fasikulus arkuata pada operkulum temporal kiri
c. Iskemik pada fasikulus arkuata pada operkulum parietal kanan
d. Iskemik pada area motor tambahan dan girus singuli yang berdekatan hemisger kiri
e. Iskemik pada fasikulus arkuata pada operkulum parietal kiri

48. Wanita 34 tahun dibawa suaminya ke poliklinik neurologi setelah diketahui mengalami kesulitan
membaca sejak 3 hari yang lalu. Pada pemeriksaan neurologis secara lengkap, tidak ditemukan
defisit neuorologis lainnya selain gangguan tsb. Pasien mengalami kesulitan saat disuruh
menuliskan atau menirukan kata-kata yang ditunjukkan kepadanya namun masih mampu
menulis spontan.
Berdasarkan gejala klinis tersebut, di manakah kemungkinan letak lesi? D:/ Aleksia without
agrafia
a. Corpus callosum posterior
b. Girus singuli kiri
c. Girus angularis kiri
d. Lobus oksipital posterior kiri
e. Lobus oksipital bilateral

111
TO UGM 17 FEBUARI 2022
49. Seorang laki-laki dibawa keluarganya ke poli dikatakan kesulitan menggerakkan lengan dan
tungkai kiri. Pada pemeriksaan pasien mampu berdiri tegak namun mengalami kesulitan
melangkahkan tungkai kiri. Tangan kiri ketika diangkat tidak terlihat adanya kelemahan, namun
pasien sulit menggerakan lengan kirinya sesuai yang dikehendaki. Pemeriksaan refleks dalam
batas normal.
Berdasarkan gejala tersebut, apa nama kelainan yang dialami pasien.
a. Psikogenic stroke
b. Apraxia callosal (disconnexion syndr)lesi korpus kallosum
c. Hemineglect
d. Apraxia idemotoror : mengganti alat dengan benda
e. Apraxia psikogenik

50. Seorang laki-laki dibawa keluarganya ke poli dikatakan kesulitan menggerakkan lengan dan
tungkai kiri. Pada pemeriksaan pasien mampu berdiri tegak namun mengalami kesulitan
melangkahkan tungkai kiri. Tangan kiri ketika diangkat tidak terlihat adanya kelemahan, namun
pasien sulit menggerakan lengan kirinya sesuai yang dikehendaki. Pemeriksaan refleks dalam
batas normal.
Berdasarkan gejala tersebut, pembuluh darah apa yang diduga terkena?
a. MCA M4 superior kanan
b. MCA M4 inferior kanan
c. ACA cabang pericalosal
d. ACA cabang callosomarginal
e. MCA Proksimal M1

51. Wanita 65 tahun di klinik memori, pasien disimpulkan tidak mengalami gejala gangguan memori.
Pasien hanya tidak mampu mengenal wajah anak maupun keluarga yang lain, namun mampu
mengenal mereka dari suaranya. Tidak ada gangguan pada jaras visual penderita.
Berdasarkan gejala di atas, pasien dikatakan mengalami
a. Agnosia visual asosiatif : mengenal benda, tau kegunaannya , tidak mampu menyebut
namanya
b. Agnosia visual aperseptif/ taktil/kinestetik ( tidak tau kegunaannya , tidak mampu menyebut
benda yang dilihatnya, namun tau setelah diletakkan di tangan)
c. Prosopagnosia (ketidak mampuan mengenali wajah) lesi PCA dextra/bilateral
d. Agnosia visuospasial
e. Agnosia simultan

52. Wanita 65 tahun di klinik memori, pasien disimpulkan tidak mengalami gejala gangguan memori.
Pasien hanya tidak mampu mengenal wajah anak maupun keluarga yang lain, namun mampu
mengenal mereka dari suaranya. Tidak ada gangguan pada jaras visual penderita.
Berdasarkan gejala di atas, di manakah kemungkinan letas lesi
a. Girus temporal anterior kanan

112
TO UGM 17 FEBUARI 2022
b. Girus oksipitotemporal kanan
c. Girus oksipitoparietal bilateral
d. Girus oksipitoparietal kiri
e. Girus fusiform bilateral : PCA D/bilateral

53. Wanita 65 tahun di klinik memori, pasien disimpulkan tidak mengalami gejala gangguan memori.
Pasien hanya tidak mampu mengenal wajah anak maupun keluarga yang lain, namun mampu
mengenal mereka dari suaranya. Tidak ada gangguan pada jaras visual penderita.
Kemungkinan pembuluh darah yang terkena :
a. MCA M4 inferior kanan
b. PCA kanan
c. MCA M4 superior bilateral
d. PCA bilateral
e. PCA Kiri

54. Laki-laki 57 tahun dibawa ke IGD karena kelemahan tubuh kanan tiba-tiba onset 2 jam SMRS.
Tidak ada sakit kepala, tidak ada mual muntah. Dari pemeriksaan fisik didapatkan kesadaran CM,
TD 160/90, nadi 110x/m, RR 24x/m. penderita tidak bisa menyebutkan nama benda tapi bisa
mengungkapkan isi pikiran orang lain. CT scan kepala didapatkan infark lakunar pada
periventrikular hemisfer kiri. D/afasia anomik
Apa jenis gangguan berbahasa yang dialami pasien tersebut
a. Afasia transkortikal motorik
b. Afasia transkortikal sensorik
c. Afasia anomik=nominal : MCA post , gyrus angularis kiri, temporalis ant/post
d. Afasia broka
e. Afasia wernike

55. Laki-laki 57 tahun dibawa ke IGD karena kelemahan tubuh kanan tiba-tiba onset 2 jam SMRS.
Tidak ada sakit kepala, tidak ada mual muntah. Dari pemeriksaan fisik didapatkan kesadaran CM,
TD 160/90, nadi 110x/m, RR 24x/m. penderita tidak bisa menyebutkan nama benda tap ibisa
mengungkapkan isi pikiran orang lain. CT scan kepala didapatkan infark lakunar pada
periventrikular hemisfer kiri.
Pemeriksaan penunjang apa yang dibutuhkan untuk mengetahui adanya sumber emboli di
jantung
a. Transcranial doppler monitoring
b. Elektrokardiogram
c. treadmil
d. Holter monitor
e. Echocardiogram transesofagus

56. Perempuan 65 tahun dibawa ke IGD karena penuruna kesadaran tiba-tiba 12 jam SMRS.
Sebelum penurunan kesadaran pasien mengalami kejang kelojotan seluruh tubuh. Sebelum

113
TO UGM 17 FEBUARI 2022
kejang pasien asadar, saat kejang tidak sadar, pasca kejang tidak sadar. Riwayat HT tapi tidak
rutin minum obat. Ada riwayat DM tidak terkontrol. Pada pemeriksaan fisik didapatkan GCS
E3M5V3, lateralisasi ke kiri, refleks patologis positif bilateral. TD 200/110, HR 98x/m, RR 22x/m,
suhu 37. CT scan kepala : infark lakunar basal ganglia kanan.
Bagaimana tatalaksana untuk HT
a. Diltiazem iv
b. Nicardipine iv
c. Nifedipine sublingual
d. Furosemide iv
e. Manitol

57. Perempuan 55 tahun datang ke poli dengan nyeri kepala sejak 1 tahun yang lalu dan memberat
1 minggu terakhir. pasien juga mulai sering lupa dengan pekerjaannya sebagai IRT dan
terkadang marah-marah tanpa alasan yang jelas. Riwayat HT (-), DM (-), KB suntik (+). Pada
pemeriksaan fisik didapatkan TD 130/80, nadi 80x, suhu 36,6. MMSE 22. CT scan kepala kontras :
lesi hiperdens yang homogen dan berbentuk bulat ukuran 2 cm melekat pada batas duramater.
Di manakah lokasi : D:/ Meningioma
a. Ventrikel lateral
b. Parasagital anterior
c. Foramen magnum
d. Falk posterior
e. Cerebelopontin angel

58. Laki-laki 59 tahun datang ke poli dengan keluhan pusing , merasa bergoyang pada saat jalan dan
duduk. Keluhan ini dirasakan sejak 6 bulan yang lalu dan memberat 3 hari terakhir. pada
pemeriksaan fisik didapatkan TD 120/80, nadi 76x/m, suhu 36,5, tonus otot ekstremitas atas dan
bawah kanan menurun. Hasil CT scan kepala kontras menunjukkan lesi bulat hiperdens dengan
batas tegas dan berbentuk kistik dan kalsifikasi pada area periventrikel IV.
Apa diagnosis yang tepat
a. Meningioma
b. Ependimoma
c. Tumor pineal
d. Tumor metastase
e. Primary CNS Lymphoma

59. Laki-laki 76 tahun dikonsul ke bagian neurologi dengan ptosis mata kanan disertai miosis. Pasien
juga merasakan hanya sisi wajah kanannya yang tidak berkeringat. Tidak ditemukan defisit
neurologis lainnya.
Apakah penyakit yang mungkin
a. Tumor pada apeks paru kanan
b. Tumor medula spinalis level C8 sd Th2 (ordo 1)
c. Lesi yang berada di antara hipotalamus dan ciliospinal center og Budge

114
TO UGM 17 FEBUARI 2022
d. Diseksi arteri carotis interna yang melibatkan regio mid cervical di atas bifurcatio karotis
(ordo 2 preganglion)
e. Hematom besar pada bagian bawah arteri subclavia akibat dari pemasangan akses vena
sentral

60. Laki-laki 56 tahun dengan kesadaran menurun dirawat di ruang perawatan intensif neurologi.
Pada pemeriksaan fisik ditemukan adanya gerakan konjugasi cepat kedua mata bergerak ke
arah inferior, diikuti dengan gerakan lambat kedua mata ke arah superior (occular bobbing),
tanpa adanya gerakan horizontal kedua mata.
Apa penyebab dari kasus tsb?
a. Pons (ataxic pontomedullary), (apneusis pontine bilateral)
b. Mesensefalon (hyperventilasi)
c. Korteks cerebri (cheyne stokes)
d. Thalamus
e. Cervicomedulary junction (apneu – respiratory arrest)

61. Perempuan 32 tahun datang ke poli saraf dengan penglihatan kabur mata kanan disertai
fotofobia. Pupil mata kanan berukuran 5 mm dan pupil mata kiri berdiameter 3 cm. Pada
pemeriksaan ditemukan arefleksia pada tendon achiles bilateral, tajam penglihatan mata kanan
menurun, pemeriksaan neurologis dalam batas normal.
Penyebab kasus di atas
a. Sindrom foville 6 + 7; MLF, PPRF, horner
b. Sindrom weber, nervus III + hemiparesis kontralateral
c. Pupil marcus gunn  RAPD (nama lain)
d. Sindrom Holmes-Adie
e. Sindrom argyl robertson : refeks pupil negatif near (+), Dissosiasi, bilateral

Adie’s pupil
a. Hilangnya refleks tendon dalam pada ekstremitas bawah. Umumnya pada wanita.
b. Kelainan saraf parasimpatis
c. Berespon saat disinari sangat terang

115
TO UGM 17 FEBUARI 2022
116
TO UGM 17 FEBUARI 2022
62. Laki-laki 35 tahun datang ke poli dengan keluhan penglihatan ganda binokuler. Pada
pemeriksaan monokuler, fungsi otot rektus medularis kedua mata dbn, namun pada
pemeriksaan binokular menunjukkan kegagalan otot rektus medial mata kanan untuk
berkontraksi sinkron dengan otot rektus lateral kontralateral. Nistagmus juga ditmeukan pada
mata yang berabduksi. INO kanan
Apakah diagnosis pada kasus di atas?
a. Deviasi konjugat
b. Internuklear oftalmoplegia
c. Sindrom webino (INO tipe 1)
d. Sindrom one-and a half (1 paresis horizontal + 1 INO)
e. Top of basillar syndrome

63. Pasien laki-laki usia 7 tahun dengan keluhan kesulitan bangkit dari posisi duduk ke berdiri sejak
usia 5 tahun, sulit naik tangga dan berjalan dengan berjinjit. Riwayat prenatal dan natal normal.
Riwayat tumbuh kembang aspek sosial normal. Berjalan sejak usia 16 bulan. Kakak laki-laki
pasien juga mengalami keluhan yang hampir sama. PF wadding gait (kelainan pada gluteus
medius), hipotonus, otot wajah normal, kelemahan otot predominan proksimal,refleks +2/+2,
hipertrofi calf. Lab CK 10200 IU/L. MLPA DMD : delesi axon 55. Dx. DMD
Apakah terapi yang dapat diberikan untuk memperpanjang periode ambulasi pada pasien?
a. Latihan fisik aerobik
b. Latihan fisik anerobik
c. Glukokortikoid : Prednisone 0.75 mg/kkgBB/24j dan Deflazacort 0,9 mg/kgBB/24j
d. Interferon
e. IVIG

64. Laki-laki 35 tahun dikonsulkan sejawat IPD dengan penurunan kesadaran sejak 12 jam yang lalu.
Pada pemeriksaan kesadaran diperoleh : pasien tidak dapat membuka mata spontan, namun
dapat kelopak mata terbuka dengan rangsang nyeri. Pada rangsang nyeri, hanya diperoleh
gerakan fleksi, refleks pupil dan kornea negatif, dan pernafasan iregular.
Berapakah skor FOUR pada pasien ini?
a. E2M2B1R2 (E1)
b. E2M2B0R1
c. E3M2B1R2
d. E3M2B0R1
e. E2M1B0R1
Soal UNAS 19 maret 2022

117
TO UGM 17 FEBUARI 2022
65. Laki-laki 52 tahun dirawat di ruang rawat insentif dengan penurunan kesadaran sejak 3 hari
SMRS. Penurunan kesadaran terjadi secara mendadak saat sedang rapat. Pasien telah dilakukan
pemeriksaan MRI kepala dengan hasil sbb/ glioma optik

118
TO UGM 17 FEBUARI 2022
Temuan klinis apakah yang terlihat pada pasien ini?
a. Pola pernafasan Kusmaull
b. Pupil miosis dengan refleks cahaya positif
c. Roving eye movement negative
d. Akinetik mutism
e. Ocular bobbing

Soal UNAS 19 maret 2022


66. Laki-laki 45 tahun dirawat di bangsal neurologis selama 1 bulan. Riwayat pasca cardiac arrest 3
minggu yang lalu. Pada saat di ruang intensif, pasien diketahui dalam kondisi koma. Namun,
berangsur-angsur mengalami perbaikan. Saat ini pasien belum dapat berespon dengan baik
dengan stimulasi visual ataupun auditorik, belum dapat berinteraksi dengan sekitar, keterjagaan
fluktuatif namun siklus bangun-tidur masih terpelihara. Pada pemeriksaan fisik didapatkan
refleks pupul, kornea, okulosefalik, dan oculovestibular normal.
Berdasarkan temuan klinis tersebut, apakah kondisi kesdadaran yang sesuai dengan kasus ini
a. Locked in syndrome
b. Catatonia
c. Minimaly consciousness state
d. Persistent vegetative state (4 mingg)
e. Akinetik mutism

67. Pasien perempuan 30 tahun dengan covid-19 derajat berat, syok sepsis, dan gagal ginjal akut
dikonsulkan oleh teman sejawat di ICU dengan penurunan kesadaran bertahap sejak 12 jam lalu.
Pemeriksaan neurologis GCS E3M5V3, pola nafas regular, pupil 3mm/3mm reaktif bilateral,
kekuatan motorik ekstremitas kesan sama kuat sisi kanan dan kiri. CT scan otak nonkontras
menunjukkan infark multipel di area antara teritori arteri serebri media dan anterior kedua
hemisfer serebri.
Apakah proses patofisiologi utama yang mendasari kelainan pada pasien ini?
a. Aterosklerosis
b. Hipoksia

119
TO UGM 17 FEBUARI 2022
c. Hiperkoagulasi
d. Emboli
e. Cortical spreading depression

68. Pasien laki-laki 43 tahun berobat di IGD RSUD dengan keluhan sakit kepala hebat sejak 2 jam
lalu. Survei primer compos mentis, jalan nafas dan pernafasan dbn. Frekuensi nadi 120x/menit,
TD 200/100 mmHg, VAS 9. Pemeriksaan neurologis GC S15, pupil 3mm/3mm reaktif bilateral,
kaku kuduk positif, kekuatan motorik baik. CT scan otak nonkontras dbn.
Apakah langkah diagnostik selanjutnya yang tepat pada pasien ini ?
a. Pungsi lumbal
b. EEG
c. MRI
d. TCD
e. USG optic nerve sheath diameter

120
TO UGM 17 FEBUARI 2022
LATIHAN SOAL edit

1. Seorang wanita, 58 tahun dibawa suaminya ke poli Saraf karena mengalami kejang sejak
2 tahun terakhir yang makin lama semakin memberat. Awalnya pasien sering kehilangan
barang karena lupa dimana meletakkannya dan sering bicara berulang ulang. Akhir akhir
ini pasien tidak bisa lagi mengerjakan pekerjaan rumah tangga seperti memasak dan
pernah tersesat ketika pulang dari pasar. Selama ini pasien tidak memiliki riwayat
hipertensi dan DM, pasien rutin medical check up karena difasilitasi perusahaan
suaminya. Paman pasien memiliki riwayat pikun. Dx. Alzheimer Disease
Gen penyebab tersering pada pasien ini adalah…
a. APO-E4 (late) : k19
b. Amyloid- beta precursor protein (21)
c. Presenilin-1 (14)  tersering
d. Presenilin-2 (1)
e. UBQLNI (tidak terkait kromosom)

2. Seorang laki-laki 71 tahun, mengalami sering lupa dan perubahan perilaku. Menurut
istrinya, pasien sering lupa akan pembicaraan/peristiwa yang belum lama terjadi. Pasien
juga kadang tampak bingung, kesulitan mengenali orang dan tempat dia berada saat itu.
Pasien juga pernah berhalusinasi seperti melihat makhluk halus. Pasien memang telah
menderita gangguan pergerakan berupa jalan yang lambat dan kaku seperti robot, serta
tangan yang sering gemetar sejak sekitar 8 tahun lalu namun keluhan lupa dan
perubahan perilaku ini baru terjadi dalam 1 tahun terakhir.
Tatalaksana farmakologi yang paling tepat untuk pasien ini adalah. PDD Parkinson lalu
demensia (10-15 thn)
a. Rivastigmin (buat yg ada halusinansi)
b. Donepezil (tidak ada efek pada halusinasi)
c. Amantadine (mirip memantine, memicu halusinasi)
d. Galatamine
e. Gingko biloba

3. Seorang wanita 73 tahun, dibawa keluarganya ke IGD karena gaduh gelisah. Sejak 1
bulan terakhir pasien sering tidak tidur, justru mengoceh sepanjang malam, bicaranya
melantur. Pasien mudah tersinggung, merasa keluarganya hendak melukainya(delusi).
Puncaknya pasien mengamuk karena merasa menantunya mengambil barangnya. Pasien
telah mengalami sering lupa dan tidak dan tidak bisa lagi mengerjakan aktivitas sehari-
hari tanpa bantuan sejak 3 tahun terakhir tapi keluarganya tidak membawanya ke dokter
karena dianggap pikun biasa. Ketika diperiksa, skor NPI ( Nuropsychiatric inventory)
pasien 3/sedang (keparahan) dan 4 (distress) (NPI : Mild <20; Moderate 20-50; Severe
>50)
Tatalaksana farmakologis yang paling tepat untuk pasien ini adalah…..
a. Haloperidol (tipikal)
b. Chlorpromazine (tipikal)

121
TO UGM 17 FEBUARI 2022
c. Fluphenazine (tipikal)
d. Pimozide (tipikal)
e. Quetiapine (untuk ggg bipolar dan skizo) (LINI I Risperidone)

4. Seorang laki-laki, 56 tahun, seorang pegawai swasta, berobat ke poliklinik saraf Bersama
istrinya. Pasien tidak bisa lagi mengerjakan beberapa tugas dalam satu waktu, kesulitan
melakukan penghitungan sederhana, sulit memfokuskan perhatian sejak 1 tahun
terakhir hingga akhirnya dipecat dari pekerjaan. Belakangan ini pasien juga mudah
lupa, tetapi basic ADL pasien belum terganggu. Pasien tidak pernah mengalami stroke
tetapi sudah menderita hipertensi sejak 3 tahun lalu. Ketika dilakukan pemeriksaan
imaging tampak hasil sebagai berikut : demensia vaskular

Tatalaksana yang paling tepat untuk pasien ini adalah…..


a. Kontrol factor risiko
b. Kombinasi donepezil dan memantine
c. Kombinasi rivastigmine dan memantine
d. Memantine dan control factor risiko
e. Donepezile dan control factor risiko

5. Laki-laki 45 tahun dengan kelemahan sesisi tubuh kanan tiba-tiba onset 1 jam yang lalu.
Kekuatan tangan kanan 3 dan tungkai kanan 3 dengan nilai NIHSS 8. Tekanan darah
160/90 mmHg, dari anamnesa tidak ada kontraindikasi untuk dilakukan tindakan IV
trombolisis. Hasil Ct scan kepala tidak tampak tanda perdarahan.
Tindakan yang sebaiknya dilakukan selanjutnya adalah..
a. Pemberian Aspilet 160-300 mg per oral
b. Pemberian IV trombolisis RtpA
c. Pemberian antikoagulan oral
d. Pemberian dual antiplatelet dan statin
e. Pemberian aspilet 80 mg

122
TO UGM 17 FEBUARI 2022
6. Perempuan 32 tahun dengan kelemahan sesisi tubuh kanan tiba-tiba onset 1 jam yang lalu
disertai bicara pelo yang ringan. Kekuatan tangan kanan 4+ dan tungkai kanan 4+ dengan
NIHSS 3 (harus 5-25). Tekanan darah 180/90mmhg, dari anamnesa tidak ada
kontraindikasi untuk dilakukan IV trombolisis. Hasil Ct scan kepala tidak ada tanda
perdarahan. (Minor Stroke)
Tindakan yang sebaiknya dilakukan selanjutnya adalah.
a. Pemberian ASA 160-300 mg per oral
b. Pemberian IV Trombolisis
c. Pemberian dual antiplatelet dan statin
d. Pemberian antikoagulan oral
e. Pemberian antikoagulan injeksi

7. Laki-laki 56 tahun dengan keluhan kelemahan sesisi tubuh kiri (kapsula interna) disertai
gangguan berjalan. Pasien sering menabrak benda yang ada disisi kirinya saat berjalan.
Terkadang pasien juga mengeluh rasa baal pada lengan dan tungkai kiri (thalamus),
pasien memiliki riwayat diabetes melitus dan tidak rutin control.
Berdasarkan gejala yang dialami pasien, bagian dari otak mana yang mengalami
kerusakan.
a. Kapsula interna-thalamus-traktus optikus (KTT)
b. Traktus kortikospinalis - lemnicus medial – pedincukus cerebellum superior
c. Cerebral cortex baik pre centralis maupun post centralis
d. Tractus motoric dan sensorik di mesensepalon, serta mengenai korteks occipital
e. Tractus kortikospnalis – serebellum – lemnikus medial

8. Wanita 65 tahun berpendidikan S1 datang Bersama suami ke poli dengan keluhan lupa
meletakkan barang, PIN ATM, mengulang pertanyaan yang sama sejak 3 tahun yang lalu.
Pasien tidak bisa merencanakan perjalanan dan tidak berminat pada hobi sebelumnya.
Suami mengambil alih semua pekerjaan rumah dan perpajakan. Saat ini pasien
mengkonsumsi Amlodipine 1x5mg. pemeriksaan fisik tidak ditemukan kelainan. MMSE
23/30(mild) ( gangguan di memori dan atensi). Dx. Alzheimer
Apakah diagnosis klinis yang sesuai dengan gambaran klinis diatas adalah…..
a. Demensia vascular
b. Pseudodemensia
c. Demensia Lewy Bodes
d. Mild kognitif Impairment
e. Demensia Alzheimer

123
TO UGM 17 FEBUARI 2022
9. Pria berusia 68 tahun dibawa oleh istri dengan keluhan kencing tidak dapat ditahan sejak
6 bulan yang lalu yang disertai dengan perubahan mood, mudah menangis
(pseudobulbar palsy) bila bertemu teman. Pemeriksaan fisik ditemukan parkinsonian
gait. CT Scan ditemukan Fazekas Skor 3. Pasien memiliki riwayat hipertensi dan
dyslipidemia sejak 5 tahun yang lalu.
Apakah diagnosis klinis yang sesuai dengan gambaran klinis diatas…..
a. Demensia vascular : depresi + faktor risiko
b. Parkinson disease demensia
c. Demensia Lewy Bodies
d. Demensia Alzheimer
e. Subcortical Vascular Demensia (ada perubahan mood dan pseudobulbar palsy) 
BINSWANGER DISEASE

10. Wanita, seorang eksekutif berusia 55 tahun dibawa oleh suaminya, datang dengan
keluhan istrinya mulai mengalami perubahan perilaku seperti mengkritik orang secara
terbuka, berpakaian yang tidak sesuai dengan kondisi, menjadi suka makanan yang
manis-manis secara berlebihan. Pemeriksaan neurologis dalam batas normal. Hasil MRI
kepala menunjukkan atrofi singulata anterior dan atrofi frontotemporal. MMSE
27/30.
Apakah diagnosis klinis yang sesuai dengan gambaran klinis diatas adalah.
a. Behaviour variant frontotemporal dementia (FTD karena perubahan perilaku)
b. Behaviour variant Alzheimer dementia
c. Sematic dementia
d. Depresi bipolar
e. Posterior variant Alzheimer dementia
3 varian pick disease :
- Behavioral variant (frontal)
- Progressive nonfluent aphasia
- Semantic demensia

11. Pasien laki-laki, usia 69 tahun 3 bulan pasca perawatan stroke, keluarga mulai kesulitan
berbicara. Pada pemeriksaan ditemukan hemiparesis kanan ringan. Dalam tes Bahasa
ditemukan ekspresi Bahasa yang tidak lancar, dengan komprehensi yang cukup baik,
namun pasien masih bisa mengulang kalimat yang terdiri dari 5 kata.
Apakah diagnosis klinis pasien tersebut..
a. Afasia Broca
b. Afasia Transkortikal motoric (kortek angular - supplementary motor area)
c. Afasia Wernicke
d. Afasia transkortikal sensorik

124
TO UGM 17 FEBUARI 2022
e. Afasia Anomia

12. Pasien wanita 53 tahun dengan riwayat Hipertensi dan dyslipidemia. Pasien mengalami
stroke 3 bulan yang lalu. Pasien mengalami kesulitan menulis dan kesulitan berhitung
uang saat melakukan pembayaran di kasir. Pada pemeriksaan neurobehaviour ditemukan
kesulitan dalam mengindentifikasi sisi kanan dan kiri pada dirinya sendiri, disamping itu
pasien mengalami kesulitan dalam penamaan jati dirinya sendiri.( syndrome gerstman)
Kemungkinan lesi pada kasus ini berada pada…
a. Lobus oksipital korteks visual primer dan asosiasi lobus oksipital kiri
b. Lobus oksipial korteks visual sekunder dan asosiasi lobus oksipital kanan
c. Lobus parietal inferior kiri (gyrus angularis)  MCA post
d. Lobus parietal superior kiri
e. Lobus parietal superior kanan

13. Pasien wanita usia 50 tahun dengan riwayat hipertensi mengkonsumsi amlodipine 1x5
mg. pasien kinan dan berpendidikan S1. Pasien 3 bulan yang lalu mengalami stroke
dengan gejala hemihipestesi sebelah kanan. Pada pemeriksaan fungsi luhur MMSE 30/30.
Pasien mengalami kesulitan memperagakan gesture memutar koin dan menjentikkan jari
dengan tangan kiri. Apraksia ideamotor : gangguan urutan
Apakah diagnosis klinis kasus diatas….
a. Agnosis visual
b. Lesi di subkortikal lobus parietal kiri
c. Pasien mengalami apraksia ideasional : salah urutan (lobus dominan)
d. Pasien mengalami apraksia ideomotor : salah alat (lobus dominan)
e. Pasien mengalami prospanogsia

Apraksia
• apraksia Ideasional: kegiatan multikomponen > melipat sebuah surat, menyisipkannya ke dalam
amplop, dan merekatkan amplop (lesi bilateral)
• apraksia Ideomotor: gerakan spontan: melambaikan tangan, minta tumpangan, menjahit, menyedot
dari sebatang sedotan, atau bersiul, memalu malah ambil gunting
• Apraksia Simpatetik (lesi frontal kiri)
• Apraksia Kallosal/ disconection syndrome, apraksia ideomotor KIRI, lesi korpus kalosum
• Apraksia Konstruksional, penderita tidak mampu membuat konstruksi sederhana atau menyalin
sebuah gambar, hemisfer nondominan (kanan)
• Apraksia dressing, kesulitan berpakaian, lesi parietal nondominan

125
TO UGM 17 FEBUARI 2022
14. Pasien 36 tahun, mengalami trauma kepala 10 tahun yang lalu. Dikatakan keluarga pasien
menjadi tidak acuh terhadap masalah disekitarnya, tidak dapat menahan emosi dan
kesulitan mengerjakan 2 hal secara bersamaan.
Pemeriksaan dibawah ini yang perlu dilakukan pada pasien adalah..
a. Tes MMSE
b. Luria Hand sequence
c. Beck depression scale
d. Trail making test A
e. Tes Serial 7

15. Seorang wanita usia 35 tahun mengalami stroke 3 bulan yang lalu dengan kelemahan
ringan pada ekstremitas kanan namun pasien mengalami kesulitan dalam berhitung, salah
mengucapkan kata dan berusaha mengkoreksinya. Pada pemeriksaan neurobehaviour
didapatkan gangguan mengulang kalimat dan parafasia, namun dapat berbicara lancer
dan mengerti perintah.
Gangguan Bahasa ini adalah.
a. Afasia konduksi (fasikulus arkuatus pd operculum parietal kiri)MCA post
b. Afasia transkortikal motoric
c. Afasia Wernicke
d. Afasia transkortikal sensorik
e. Afasia anomia

16. Pasien perempuan 16 tahun datang dibawa ibu dengan keluhan keempat ekstremitas
bergoyang-goyang diluar kendali. Pada pemeriksaan fisik ditemukan Tics fasialis,
anxietas, iritabilitas, dan korea pada keempat ekstremitas. Sydenham chorea
Pernyataan dibawah ini yang sesuai dengan klinis diatas adalah kecuali.
a. Pada pemeriksaan swab tenggorokan biasanya ditemukan streptokokus beta
hemolitikus : streptokokus hemolitikus type A
b. Pemerikaan kualitatif anti streptolisin 0 positif  ASTO grup A streptokokus

126
TO UGM 17 FEBUARI 2022
c. Pasien membutuhkan antibiotic penisilin jangka pendek  harus panjang
d. Pada pasien dapat ditemukan miokarditis
e. Sebagian pasien mengalami gejala korea yang berulang atau menetap

Korea primer : Huntington disease (genetik)


Korea sekunder : Sydenham korea, toksik obat, dll

17. Wanita berusia 75 tahun datang ke klinik dengan keluhan utama gemetar pada kedua
tangan sejak 2 tahun yang lalu. Pemeriksaan neurologis ditemukan resting tremor,
rigiditas, bradikinensia kanan lebih berat dari kiri. Pull tes (retropulsi) mundur 4
langkah (normal <2 langkah  menilai instabilitas postural). Pasien masih mampu
melakukan aktivitas sehari-hari tanpa bantuan dan tidak pernah jatuh
Berapakah stadium Hoehn and Yahr pada kasus diatas..
a. Stadium 1 (tremor 1 sisi
b. Stadium 2 (tremor 2 sisi
c. Stadium 3 (ADA GANGGUAN KESEIMBANGAN, disfungsi sedang)
d. Stadium 4 (tidak bisa berdiri sendiri)
e. Stadium 5 (cacat total)

18. Wanita berusia 75 tahun datang ke klinik dengan keluhan utama gemetar pada kedua
tangan sejak 2 tahun yang lalu. Pemeriksaan neurologis ditemukan resting tremor,
rigiditas, bradikinensia kanan lebih berat dari kiri. Pull tes mundur 4 langkah. Pasien
masih mampu melakukan aktivitas sehari-hari tanpa bantuan dan tidak pernah jatuh
Mengamati kasus di atas, terapi yang sesuai diberikan adalah..
a. Tryhexiphendil 3x2 mg
b. Levodopa 3 x 62,5 mg titrasi naik

127
TO UGM 17 FEBUARI 2022
c. Ropinrole 1x4 mg titrasi naik
d. Pramipexole 1,5mg – 4,5 mg
e. Propranolol 2 x 10mg titrasi naik

19. Pria 48 tahun mengalami gemetar kedua tangan dan jalan lamban serta memberat
sejak 1 tahun yang lalu. Pada pemeriksaan fisik ditemukan tremor, rigiditas dan
bradykinesia kiri lebih berat dari kanan. Pasien masih mandiri dalam mengerjakan
pekerjaan sehari hari.
Terapi yang dapat diberikan pada pasien..
a. Levodopa 3x125mg titrasi naik
b. Ropinirole 1x4mg titrasi naik (dosis terendah 0,25 -6 mg tertinggi))
c. Propranolol 2x10mg titrasi naik (2x40mg)
d. Pramipexole 1x1 mg (maksimal 0,375)
e. Entacarpone 2 x 200 mg titrasi naik (200 – 8x200 / hari)  bikin urin berubah warna

128
TO UGM 17 FEBUARI 2022
129
TO UGM 17 FEBUARI 2022
20. Pasien pria usia 72 tahun sudah didiagnosis parkinson disease datang dengan keluhan
gerakan badan dan ekstremitas yang tidak dapat dikontrol (diskinesia). Pada pemeriksaan
neurologis ditemukan tremor, rigidity, akinesia, dan postural instability. Pasien sudah
mendapat terapi levodopa 3x250mg. Dyskinesia (cuit 59) (levo 3x100 Max. 1500)
Manakah yang dapat diberikan pada pasien adalah….
a. Kombinasi levodopa dan COM inhibitor
b. Dosis levodopa tetap dan menambahkan dopamine agonis
c. Meningkatkan dosis levodopa dan menambahkan dopamine agonis
d. Menurunkan dosis levodopa dan menambahkan dopamine agonis
e. Mengganti dengan dopamine agonis tunggal

21. Seorang wanita 69 tahun datang ke poli neuro dengan keluhan kelemahan anggota gerak
kiri sejak 2 hari yang lalu saat pasien bangun tidur. Keluhan dirasakan memberat, mula-
mula tungkai masih dapat berjalan walaupun diseret dan lengan dapat menggenggam
benda, tetapi sejak kemarin tungkai hanya bisa digeser dan lengan tidak dapat digerakkan
sama sekali. Keluhan disertai bicara pelo dan mulut mencong ke kanan (tipika). Pasien
memiliki riwayat hipertensi dan DM.
Pemeriksaan penunjang yang menjadi gold standar pada pasien ini adalah..
a. Lumbal pungsi
b. CT scan kepala
c. MRI kepala
d. Angiografi
e. USG Karotis

22. Seorang wanita 69 tahun datang ke poli neuro dengan keluhan kelemahan anggota gerak
kiri sejak 2 hari yang lalu saat pasien bangun tidur. Keluhan dirasakan memberat, mula-
mula tungkai masih dapat berjalan walaupun diseret dan lengan dapat menggenggam
benda, tetapi sejak kemarin tungkai hanya bisa digeser dan lengan tidak dapat digerakkan
sama sekali. Keluhan disertai bicara pelo dan mulut mencong ke kanan. Pasien memiliki
riwayat hipertensi dan DM.
Bila pada pasien ditemukan bruit pada pemeriksaan A. karotis, pemeriksaan yang paling
baik dilakukan…..
a. Transcranial doppler
b. CT scan kepala
c. MRI kepala
d. Angiografi
e. Carotid duplex

130
TO UGM 17 FEBUARI 2022
23. Pasien laki-laki usia 60 tahun datang dengan keluhan penglihatan ganda, gangguan
pengecapan, dan lemah tubuh sebelah kiri mendadak. Pada pemeriksaan fisik ditemukan
parese NVI kanan dan parese N VII perifer kanan disertai dengan adanya gangguan
pengecapan 2/3 anterior lidah dan hemiparese sinistra.
Dimanakah lokasi lesi pasien di atas.. Foville (jawaban KUK)
a. Pons kiri
b. Pons kanan hemiparese kontralateral
c. Mesensefalon kiri
d. Mesensefalon kanan
e. Medulla oblongata kiri

24. Pasien laki-laki usia 60 tahun datang dengan keluhan penglihatan ganda, gangguan
pengecapan, dan lemah tubuh sebelah kiri mendadak. Pada pemeriksaan fisik ditemukan
parese NVI kanan dan parese N VII perifer kanan disertai dengan adanya gangguan
pengecapan 2/3 anterior lidah dan hemiparese sinistra.
Dimanakah lokasi lesi pasien di atas.
a. Foville syndrome
b. Benedict syndrome
c. Milliard Gubler Syndrome (6-7 SAJA)
d. Weber Syndrome
e. Cerebellar Syndrome

25. Seorang wanita, 69 tahun datang ke poliklinik saraf dengan keluhan kelemahan anggota
gerak kiri sejak 2 hari yang lalu saat pasien bangun tidur. Keluhan dirasakan memberat,
mula-mula tungkai masih dapat berjalan walaupun diseret dan lengan dapat
menggenggam benda digerakkan sama sekali. Keluhan disertai bicara pelo dan mulut
mencong ke kanan. Pasien memiliki riwayat hipertensi dan DM.
Manakah obat antiagregasi di bawah ini yang dapat bekerja menghambat ADP..
a. Aspirin : siklooksigenase 1
b. Clopidogrel : menghambat ADP
c. Abeiximab : gpIIbIIIa inhibitor
d. Cilostazol : fosfodiastrase
e. Triflusal : trombosigenesis

26. Seorang laki-laki, usia 65 tahun, pasca perawatan stroke penyumbatan 4 minggu yang
lalu. Pasien mengalami gangguan bicara tetapi masih dapat mengerti pembicaraan dan
hemiparesis kanan (afasia motoric). Pasien bedridden. Pasien diketahui menderita
hipertensi dan diabetes mellitus sejak 15 tahun yang lalu tetapi control tidak teratur,
obat antihipertensi hanya diminum jika nyeri kepala saja.
Apakah masalah utama pada pasien di atas..
a. Pasca stroke infark

131
TO UGM 17 FEBUARI 2022
b. Disabilitas dan kepatuhan yang buruk
c. Hipertensi dan diabetes melitus
d. Tirah baring dan gangguan komunikasi
e. Independensi

27. Seorang laki-laki, usia 65 tahun, pasca perawatan stroke penyumbatan 4 minggu yang
lalu. Pasien mengalami gangguan bicara tetapi masih dapat mengerti pembicaraan dan
hemiparesis kanan afasia motorik. Pasien debridden. Pasien diketahui menderita
hipertensi dan diabetes mellitus sejak 15 tahun yang lalu tetapi control tidak teratur, obat
antihipertensi hanya diminum jika nyeri kepala saja.
Dimanakah lokasi lesi infark pasien di atas..
a. Lobus temporal kiri
b. Lobus frontalis kiri (frontal infero lateral) MCA sup
c. Lobus frontalis kanan
d. Lobus parietal kiri
e. Lobus temporal kanan

28. Seorang laki-laki, usia 65 tahun, pasca perawatan stroke penyumbatan 4 minggu yang
lalu. Pasien mengalami gangguan bicara tetapi masih dapat mengerti pembicaraan dan
hemiparesis kanan. Bila lengan kanan digerakkan pasien menyeringai karena nyeri.
Pasien diketahui menderita hipertensi dan diabetes mellitus sejak 15 tahun yang lalu
tetapi control tidak teratur, obat antihipertensi hanya diminum jika nyeri kepala saja.
Apakah komplikasi yang terjadi pada pasien ini…
a. Deep vein thrombosis
b. Disuse atrofi
c. Contracture
d. Vegetative state
e. Central post stroke pain  amitriptilin (lini I) / lamotrigin

29. Seorang laki-laki, usia 45 tahun, 3 jam yang lalu mendadak mengalami lemah tubuh
sebelah kanan disertai sulit berbicara tetapi masih dapat mengerti pembicaraan. Bicara
menjadi pelo dan terdapat kesemutan pada tubuh sebelah kanan. Pasien memiliki riwayat
serangan jantung 3 bulan yang lalu dan menggunakan antikoagulan. Pada pemeriksaan
CT scan kepala didapatkan adanya infark ganglia basalis kiri. INR 1,8 (batas 1,7). Skor
NIHSS 12.(5-25)
Manakah tatalaksana yang paling tepat untuk pasien ini.
a. Aspirin 80-325 mg
b. Clopidogrel 300mg
c. Recombinant Tissue Plasminogen Activator Intravena (karna INR dan ada infark)
tidak boleh ada infark di CT scan
d. Trombolektomi intraarterial
e. Cilostazol 200mg : jarang diberi fase akut

132
TO UGM 17 FEBUARI 2022
30. Seorang laki-laki, usia 23 tahun datang dengan kelemahan pada anggota gerak kanan
mendadak saat sedang aktivitas sejak 6 jam yang lalu. Pasien tetap sadar. Bicara pelo dan
mulut mencong ke kiri, dan sulit untuk berbicara. Pasien diketahui memiliki kelainan
jantung sejak 3 tahun yang lalu, berobat tidak teratur. Pada pemeriksaan fisik didapatkan
denyut jantung irregular. Denyut nadi 88, denyut jantung 118 x/i.
Apakah diagnosis yang paling mungkin pada kasus diatas.
a. Stroke
b. Meningitis
c. Tumor otak
d. Ensefalitis
e. Abses cerebri
Soal koas

31. Seorang laki-laki, usia 23 tahun datang dengan kelemahan pada anggota gerak kanan
mendadak saat sedang aktivitas sejak 6 jam yang lalu. Pasien tetap sadar. Bicara pelo dan
mulut mencong ke kiri, dan sulit untuk berbicara. Pasien diketahui memiliki kelainan
jantung sejak 3 tahun yang lalu, berobat tidak teratur. Pada pemeriksaan fisik didapatkan
denyut jantung irregular. Denyut nadi 88, denyut jantung 118 x/i.
Apakah kemungkinan penyebab kelainan pada kasus diatas…
a. Dilatasi kardiomiopati
b. Decompensation cordis
c. Atrial fibrilasi
d. Mitral regurgitasi
e. Tetralogy falot

32. Seorang laki-laki, usia 23 tahun datang dengan kelemahan pada anggota gerak kanan
mendadak saat sedang aktivitas sejak 6 jam yang lalu. Pasien tetap sadar. Bicara pelo dan
mulut mencong ke kiri, dan sulit untuk berbicara. Pasien diketahui memiliki kelainan
jantung sejak 3 tahun yang lalu, berobat tidak teratur. Pada pemeriksaan fisik didapatkan
denyut jantung irregular. Denyut nadi 88, denyut jantung 118 x/i.
Apakah pengobatan yang paling sesuai untuk kasus diatas..
a. Antiagregasi
b. Antikoagulan : karena penyebab valvular langusg warvarin
c. Neuroprotektan
d. Vitamin neurotropik
e. Trombolisis

33. Seorang laki-laki usia 69 tahun datang dengan keluhan sesak napas dan jantung berdebar
disertai dengan nyeri menjalar ke tungkai kiri. Tungkai kiri juga menjadi sulit untuk
melangkah dan ditemukan rasa kesemutan disertai adanya nyeri. Pasien diketahui
mengalami gangguan jantung sejak lama, dan sedang berobat ke dokter jantung. Pada

133
TO UGM 17 FEBUARI 2022
pemeriksaan fisik ditemukan adanya hipestesi setinggi lumbal 1 kiri. Pulsasi A.
femoralis dan dorsalis pedis kiri tidak teraba.
Apakah komplikasi neurologi yang terjadi pada kasus di atas..
a. Stroke emboli
b. Peripheral Artery Disease
c. Stroke trombotik
d. Deep vein thrombosis (ex burger disease)
e. Neuropathy

34. Seorang laki-laki usia 69 tahun datang dengan keluhan sesak napas dan jantung berdebar
disertai dengan nyeri menjalar ke tungkai kiri. Tungkai kiri juga menjadi sulit untuk
melangkah dan ditemukan rasa kesemutan disertai adanya nyeri. Pasien diketahui
mengalami gangguan jantung sejak lama, dan sedang berobat ke dokter jantung. Pada
pemeriksaan fisik ditemukan adanya hipestesi setinggi lumbal 1 kiri. Pulsasi A. femoralis
dan dorsalis pedis kiri tidak teraba.
Pembuluh darah manakah yang tersumbat pada kasus diatas…..
a. Aorta abdominalis
b. A. carotis interna kanan
c. A. serebri media kanan
d. A. serebri anterior kanan
e. A. Iliaca kiri : peripheral artery disease

35. Seorang perempuan berusia 55 tahun diantar ke IGD RS dengan kelemahan separuh
tubuh sebelah kanan mendadak sejak 5 jam yang lalu. Didapatkan riwayat hipertensi dan
DM. pada pemeriksaan fisik didapatkan TTV Dbn. Pada pemeriksaan neurologis
didapatkan GCS 456, motoric hemiparese kanan, sensorik Dbn. Pada pemeriksaan Ct
scan kepala didapatkan (CT scan diganti dengan)

Apakah etiologic yang paling mungkin menurut klasifikasi TOAST untuk kasus diatas.
a. Small vessel disease
b. Embolik kardioembolik
c. Cerebral venous thrombosis

134
TO UGM 17 FEBUARI 2022
d. Arterosklerotik pembuluh darah besar
e. Diseksi karotis
TOAST :
- LVA – LVO
- SVD
- Cardioembolic
- Other determinded cause
- Undeterminded cause

36. Perempuan 35 tahun datang ke poli saraf dengan keluhan kejang pada ekstremitas sebelah
kiri. Beberapa saat setelah kejang, pasien mengalami kelemahan ekstremitas kiri sampai
beberapa jam. Pasien pernah dilakukan pemeriksaan angiography DSA ditemukan nidus
AVM highflow dilobus temporoparietal kiri dari MCA kiri sesuai gambar dibawah.

Apakah penyebab paling mungkin terjadinya hemiparesis kiri pada pasien tersebut.
a. Adanya steal phenomenon di area hemisfer kanan (harusnya kiri)
b. Adanya kompresi nidus AVM
c. Adanya perdarahan akibat rupture pada nidus AVM
d. Adanya hypoplasia vascular hemisfer kanan
e. Adanya thrombosis di MCA kanan

37. Seorang perempuan berusia 55 tahun diantar ke IGD RS dengan kelemahan separuh
tubuh sebelah kanan mendadak sejak 5 jam yang lalu. Didapatkan riwayat hipertensi dan
DM. pada pemeriksaan fisik didapatkan TTV TD 170/100 mmhg, N 100 x/I, RR 12 x/I, T
37,5 C. Pada pemeriksaan neurologis didapatkan GCS 456, motoric hemiplegia kanan,
sensorik hemihipestesi kanan. Pada pemeriksaan Ct scan kepala didapatkan

135
TO UGM 17 FEBUARI 2022
Pada pemeriksaan karotis doppler didapatkan plak pada bulbus karotis kiri dan
kanan.
Apakah etiologi yang paling mungkin menurut klasifikasi TOAST untuk kasus
tersebut…..
a. Small vessel disease
b. Embolik kardioembolik
c. Perdarahan subarachnoid
d. Diseksi karotis
e. Arterosklerotik pembuluh darah besar

38. Perempuan berusia 35 tahun datang ke IGD dengan keluhan nyeri leher akut, nyeri
kepala, dan hilangnya visus pada lapang penglihatan kanan sejak 4 hari yang lalu.
Pemeriksaan fisik juga didapatkan hemiparesis kanan dan afasia non fluent. Pasien
memiliki pijat urat saraf dengan manipulasi leher. Tidak ada riwayat hipertensi dan
gangguan jantung. Pemeriksaan GD dan profil lipid Dbn. MRI kepala ditemukan
gambaran hiperintens di daerah karotis kiri seperti berikut ini

Apakah etiologi yang paling memungkinkan tersebut…..


a. Cerebral Venous Trombosis
b. Metastasis Keganasan
c. Fibrilasi atrial
d. Stenosis Karotis
e. Diseksi arteri  antiplatelet 3 bulan

136
TO UGM 17 FEBUARI 2022
39. Perempuan 45 tahun datang ke IGD RS dengan keluhan nyeri kepala hebat mendadak
sejak 2 jam yang lalu. Nyeri terutama di kepala bagian belakang. Pemeriksaan TS
180/100 mmHg, numeric scale 8-9, dan kaku kuduk positif. Hasil CT scan kepala non
kontras dalam batas normal
Apakah langkah selanjutnya yang tepat dilakukan untuk pasien ini.
a. Pungsi lumbal
b. MRI kepala
c. EEG
d. USG karotis
e. Transkranial Doppler
Premesencephali : PSA tanpa ada gambaran perdarahan

40. Laki-laki berusia 65 datang ke IGD RS dengan keluhan kelemahan lengan dan tungkai
kanan sejak 2 jam yang lalu. Keluhan disertai bicara pelo saat berkomunikasi. Pasien
baru operasi prostat 1 minggu lalu (harus >2 mgg) . Pemeriksaan fisik TD 200/85
mmHg, Nadi 102 x/I, suhu 37,3 C dan saturasi 98%. Hasil GDS 190 mmhg. Skor
NIHSS 14. CT scan kepala non kontras tidak ditemukan perdarahan intracranial.
Apakah langkah selanjutnya dalam penatalaksanaan kasus ini.
a. Aspirin 80-325 mmhg
b. Nikardipin mulai 5 mg/jam
c. Alteplase 0,9 mg/kgBB
d. Insulin drip 1 unit/jam
e. Manitol loading 0,5 g/kgbb
KI : oprasi minor 10 hari

41. Pasien laki laki 60 tahun datang ke IGD dengan keluhan kelemahan tubuh sisi kanan
sejak 8 jam yang lalu. Keluhan disertai rasa baal sesisi tubuh kanan dan gangguan lapang
pandang sisi kanan (tipika). Pemeriksaan fisik menunjukkan hemiparesis kanan,
hemihipestesi kanan, dan hemianopsia homonin kanan. Pemeriksaan CT scan kepala non
kontras menunjukkan lesi hipodens di krus posterior kapsula interna kiri, thalamus kiri,
dan sisi lateral mesensefalon kiri.
Apakah nama pembuluh darah yang menjadi penyebab keluhan pasien di atas…..
a. Arteri serebri media
b. Arteri lentrikulostriata medial
c. Arteri karotis interna
d. Arteri komunikans posterior
e. Arteri koroidalis anterior

42. Perempuan berusia 62 tahun datang ke IGD RS dengan keluhan kelemahan sisi kanan
sejak 4 jam yang lalu. Keluhan disertai gangguan sensibilitas lengan sisi kanan.
Pemeriksaan fisik didapatkan kelemahan wajah sisi kanan(tipika), cara berbicara
pasien menjadi sulit dan terfragmentasi, dan tidak mampu menamai beberapa

137
TO UGM 17 FEBUARI 2022
objek (anomik). Pemahaman Bahasa masih intak, akan tetapi terdapat kesulitan dalam
repetisi. D/afasia motorik
Apakah arteri yang paling mungkin menjadi penyebab keluhan pasien diatas…..
a. Arteri serebri anterior
b. Arteri serebri media cabang inferior (afasia sensorik)
c. Arteri lentrikulostriata
d. Arteri hubner
e. Arteri serebri media cabang superior

43. Laki laki 70 tahun menderita stroke sejak 1 tahun lalu dengan kelemahan anggota gerak
kanan yang masih menetap.
Manakah nilai CBF berikut ini yang berhubungan dengan kerusakan irreversible fungsi
saraf….
a. 65 mL/100g/menit
b. 45 mL/100g/menit
c. 35 mL/100g/menit
d. 15 mL/100g/menit
e. 5 mL/100g/menit

44. Seorang laki-laki usia 48 tahun datang dengan keluhan kelemahan anggota gerak
kanan mendadak saat aktifitas 5 jam yll. Lengan masih disertai dengan kesulitan
berbicara tetapi pasien masih dapat mengerti pembicaraan. Tampak mulut mencong ke
kiri. Pasien memiliki riwayat hipertensi dan control tidak teratur. Pada pemeriksaan CT
scan didapatkan lesi hipodens kortikal subkortikal frontotemporal kiri. USG karotis
didapatkan plak unstable di A.carotis komunis kiri dan stenosis 75%. Echocardiografi
tidak ditemukan kelainan katup, EF 70%.
Klasifikasi stroke manakah yang paling sesuai berdasarkan Bamford.
a. Partial anterior circulation infarct
b. Total anterior circulation infarct
c. Posterior circulation infarct
d. Lacunar anterior circulation infarct
e. Middle cerebral circulation infarct

138
TO UGM 17 FEBUARI 2022
45. Seorang laki-laki usia 48 tahun datang dengan keluhan kelemahan anggota gerak kanan
mendadak saat aktifitas 5 jam yll. Lengan masih disertai dengan kesulitan berbicara tetapi
pasien masih dapat mengerti pembicaraan. Tampak mulut mencong ke kiri. Pasien
memiliki riwayat hipertensi dan control tidak teratur. Pada pemeriksaan CT scan
didapatkan lesi hipodens kortikal subkortikal frontotemporal kiri. USG karotis didapatkan
plak unstable di A.carotis komunis kiri dan stenosis 75%(plak >50). Echocardiografi
tidak ditemukan kelainan katup, EF 70%.
Klasifikasi stroke manakah yang paling sesuai untuk kasus diatas berdasarkanTOAST..
a. Small Vessel Disease (lacunar : jadi cari gejala LACI)
b. Large vessel aterosklerosis
c. Cardioembolic stroke
d. Stroke undertermined
e. Stroke unspesific

46. Seorang laki-laki usia 48 tahun datang dengan keluhan kelemahan anggota gerak kanan
mendadak saat aktifitas 5 jam yll. Lengan masih disertai dengan kesulitan berbicara
tetapi pasien masih dapat mengerti pembicaraan. Tampak mulut mencong ke kiri. Pasien
memiliki riwayat hipertensi dan control tidak teratur. Pada pemeriksaan CT scan
didapatkan lesi hipodens kortikal subkortikal frontotemporal kiri. USG karotis didapatkan
plak unstable di A.carotis komunis kiri dan stenosis 75%. Echocardiografi tidak
ditemukan kelainan katup, EF 70%.
Apakah terapi yang paling sesuai untuk kasus diatas..
a. Trombolisis intravena
b. Aspirin 80-325 mg
c. Atorvastatin dosis tinggi
d. Antikoagulan
e. CPG 300mg

47. Seorang laki-laki usia 48 tahun datang dengan keluhan kelemahan anggota gerak kanan
mendadak saat aktifitas 5 jam yll. Lengan masih disertai dengan kesulitan berbicara tetapi
pasien masih dapat mengerti pembicaraan. Tampak mulut mencong ke kiri. Pasien
memiliki riwayat hipertensi dan control tidak teratur. Pada pemeriksaan CT scan
didapatkan lesi hipodens kortikal subkortikal frontotemporal kiri. USG karotis didapatkan
plak unstable di A.carotis komunis kiri dan stenosis 75%. Echocardiografi tidak
ditemukan kelainan katup, EF 70%.
Bila terdapat gangguan katup jantung dari echocardiografi dan USG karotis dbn,
manakah obat yang paling tepat sebagai prevensi sekunder untuk kasus tersebut….
a. Aspirin
b. Dabigatran
c. Rivaroxaban
d. Warfarin
e. CPG

139
TO UGM 17 FEBUARI 2022
48. Seorang laki-laki usia 48 tahun datang dengan keluhan kelemahan anggota gerak kanan
mendadak saat aktifitas 5 jam yll. Lengan masih disertai dengan kesulitan berbicara tetapi
pasien masih dapat mengerti pembicaraan. Tampak mulut mencong ke kiri. Pasien
memiliki riwayat hipertensi dan control tidak teratur. Pada pemeriksaan CT scan
didapatkan lesi hipodens kortikal subkortikal frontotemporal kiri (favor delay
OAC). USG karotis didapatkan plak unstable di A.carotis komunis kiri dan stenosis
75%. Echocardiografi tidak ditemukan kelainan katup, EF 70%.
Bila pada pemeriksaan echocardiografi didapatkan adanya thrombus dan USG karotis
ditemukan plak stabil 40%, kapankah waktu yang paling tepat pemberian terapi stroke
sekunder…. (harus diatas 7 hari)
a. Hari ke 1
b. Hari ke 2
c. Hari ke 5
d. Hari ke 8
e. Hari ke 13
Faktor untuk menunda OAC pada stroke :
- Transformasi
- NIHSS >8
- Infark luas
- Perlu operasi
- Operasi karotis
- Usia tua
- HT tidak terkontrol
- Neurologic unstable

49. Laki-laki 50 tahun, datang dengan kelemahan sisi kiri mendadak sejak 2 jam yang lalu.
Riwayat keluar dari ICCU 4 bulan yang lalu karena infark miokard. Riwayat operasi
ORIF 4 bulan lalu. Riwayat BAB hitam 2 minggu SMRS. Pemeriksaan fisik: TD
185/105 mmhg. GCS E3M6V5 dan lateralisasi sinistra. Pemeriksaan CT scan kepala
terdapat infark di basal ganglia dextra. Hasil laboratorium GDS 350 g/dl. Pasien akan
direncanakan pemberian rTPA tetapi tidak memenuhi syarat.
Manakah factor yang menyebabkan pasien batal mendapat rTPA….
a. Tekanan darah pasien (185/110)
b. Infark miokard 4 bulan yang lalu (3 bulan)
c. Riwayat operasi mayor 4 bulan yang lalu (2 minggu)
d. Riwayat perdarahan GI tract 2 minggu yang lalu (minimal 21 hari)
e. Gula darah dari 350 g/dl (50-400)

50. Laki-laki 50 tahun, datang dengan kelemahan sisi kiri mendadak sejak 2 jam yang lalu.
Riwayat keluar dari ICCU 4 bulan yang lalu karena infark miokard. Riwayat operasi
ORIF 4 bulan lalu. Riwayat BAB hitam 2 minggu SMRS. Pemeriksaan fisik: TD 185/105
mmhg. GCS E3M6V5 dan lateralisasi sinistra. Pemeriksaan CT scan kepala terdapat
infark di basal ganglia dextra. Hasil laboratorium GDS 350 g/dl.
140
TO UGM 17 FEBUARI 2022
Letak trombus tersering..
a. Bifucartio arteri cerebral media
b. Arteri carotis interna
c. Proksimal arteri cerebral posterior
d. Proksimal arteri cerebral anterior
e. Pertemuan antara arteri vertebralis dan arteri basilaris

Lokasi tersering terjadinya plak atheromatosa:


1. ICA pada daerah percabangan dari a.carotis communis
2. Cabang cervical a. Vertebralis dan lokasi bertemunya menjadi a. Basilar
3. Bifurcasio MCA
4. Proximal PCA yang mengelilingi mesensefalon
5. Proximal ACA yang melalui anterior dan lengkung corpus callosum

51. Laki-laki usia 52 tahun datang ke poli saraf dengan keluhan mata kanan bengkak,
kemerahan disertai nyeri setelah mengalami KLL. Pada pemeriksaan dijumpai proptosis,
chemosis, bruit orbital dan ophtalmoplegia. Tidak didapatkan kelainan pada motoric
maupun sensorik. (CCF)
Apakah tindakan yang tepat penegakkan diagnose pasien tsb…..
a. VEP
b. TCD
c. CT scan
d. DWI
e. DSA

52. Wanita 67 tahun dibawa ke IGD RS dengan kelemahan anggota gerak kiri mendadak 2
hari SMRS. Terdapat riwayat DM tipe II dan HT tidak terkontrol. EKG tidak
menunjukkan AF.TEE normal. CTA menunjukkan stenosis 50% pada lumen a. carotis
eksterna kiri dan stenosis 50% pada lumen a. carotis interna kanan.
Terapi pada pasien ini…..
a. Antiplatelet
b. Antikoagulan
c. Trombektomi
d. Stenting a carotis eksterna kiri
e. Stenting a carotis interna kanan (karena hemiparese kiri)

53. Wanita 47 tahun dibawa ke IGD dengan kelemahan anggota gerak kiri mendadak 6 jam
yll. Terdapat riw DM tipe II dan HT tidak terkontrol. EKG tidak menunjukkan AF. TEE
normal. Pada pemeriksaan fisik didapatkan GCS E4 M6 V5. Nn cranialis paresis N VII,
N XII sinistra sentral, motoric 555/333, 555/333, sensorik DBN.
Tatalaksana neurointervensi yang tepat pada pasien ini…
a. Aspirin 80-325 mg

141
TO UGM 17 FEBUARI 2022
b. IV trombolisis
c. IA trombolisis (trombektomi)
d. Carotis stensing
e. Carotid angioplasty

54. Perempuan 65 tahun mengalami multiple episode TIA berupa kelemahan sisi kanan, hal
ini berlangsung sejak 5 jam SMRS. Saat serangan os mengalami kelemahan anggota
gerak, namun masih bisa beraktivitas seperti biasa. Bicara pelo (+), mulut perot (+),
riwayat HT dan DM disangkal. Hasil carotid duplex stenosis 80% pada arteri carotis
interna sinistra.
Apakah neurointervensi yang tepat pada kasus ini…..
a. Aspirin 80-325 mg
b. IA trombolisis
c. IV trombolisis
d. Trombektomi
e. Carotid angioplasty and stensting (CAS karena dibawah 70 thn)
KETENTUAN STENTING :
- Stenosis minimal 50% yang simtomatik (kelas I, peringkat bukti A). : EKSTRAKRANIAL
- Stenosis minimal 60% yang asimtomatik (kelas I, peringkat bukti A). :
EKSTRAKRANIAL
- Utk tindakan  noninvasif : >70% (USG doppler); invasif >50% (MRA,CTA,DSA)
KETENTUAN STENTING EKSTRAKRANIAL :
- Stenosis 50-99% simptomatik  CEA
- Stenosis <50% simptomatik  tidak rekomendasi CEA & CAS
- Jika CEA di indikasikan HARUS lakukan dalam 2 minggu
- Usia tua >70 tahun  CEA lebih baik dari CAS,
- usia <70 thn  CEA = CAS
- stenosis <70% simptomatik  kontraindikasi CEA (anatomi sulit)  CAS
Vertebrobasilar ekstrakranial :
- Kalau BMT sudah diberikan tapi masih bergejala setelah pengobatan  tindakan
endovaskular bisa dilakukan
Aterosklerosis intrakranial :
- Stenosis 50-99% simptomatik : aspirin 325 mg (DI KASIH TERUS)
- Stenosis 70-99% simptimatik (TIA/Stroke dalam 30 hari) : Aspirin 325 + Clopidogrel
75% (90 HARI)
- STATIN DIBERIKAN PADA PASIEN INI
- BMT : antitrombotik, statin, modifikasi faktor risiko, tekanan darah <140

142
TO UGM 17 FEBUARI 2022
55. Wanita 41 tahun nyeri kepala sejak 1 tahun yll. Didapatkan kelemahan anggota gerak 5
jam SMRS. Mulut perot (+), bicara pelo (+), DSA didapatkan dural AV fistula dengan
feeding arteri a cerebri media kanan, segmen M4, cognard tipe IIa.
Apakah terapi neurointervensi yang tepat pada kasus ini…
a. Clipping
b. Trombektomi
c. Embolisasi fistula
d. Reseksi microsurgery
e. Radioterapi

56. Laki-laki 28 tahun datang ke IGD RS dengan kelemahan sesisi tubuh kanan secara
mendadak 1 hari yll. TTV, TD 130/80 mmhg, HR 72 x/I, bruit (-), GCS E4 Mx Vx
afasia global. Motoric 444/555, 444/555. Sensibilitas DBN. EKG DBN. CT scan kepala:
infark trombotik akut subcortical frontoparietal kiri. Pemeriksaan DSA didapatkan
gambaran oklusi pada arteri carotis interna kanan dan kiri hingga seluruh MCA, ACA,
PICA kanan dan kiri. Disertai gambaran khas sesuai gambar.

Diagnosis yang tepat pada pasien diatas…..


a. Stroke kardioemboli
b. Binswanger disease
c. Posterior reversible ensepalophaty syndrome
d. Subarachnoid hemorrhage dengan delayed ischemic
e. Trombosis serebri dengan moyamoya diseases (puff smoke appearance)

57. Seorang laki-laki 34 tahun datang ke IGD karena mengalami penurunan kesadaran yang
disertai nyeri kepala hebat mendadak sejak 2 jam SMRS. Pada pemeriksaan didapatkan
GCS E2 M4 V3, TD 150/90 mmHg, HR 110 x/I, RR 22x/I, T 36,2 C. pupil bulat, isokor
3 mm/3mm. ct scan kepala menunjukkan suatu gambaran perdarahan subarachnoid.
Pasien direncanakan untuk dilakukan pemeriksaan CTA untuk mencari etiologi.
Lokasi aneurisma intraserebral kedua tersering pada kasus pasien diatas adalah….
a. A. komunikans anterior (1)

143
TO UGM 17 FEBUARI 2022
b. A. komunikanas posterior
c. Bifucartio dari arteri karotisinterna dan arteri serebri media (ke 3)
d. Pertemuan antara arteri communikans posterior dengan arteri karotis interna
(ke 2)
e. A. basilaris distal

58. Wanita 47 tahun dibawa ke poli saraf dengan kelemahan anggota gerak kiri sejak 1 tahun
yll. Terdapat riwayat DM tipe II dan HT tidak terkontrol. EKG tidak menunjukkan AF.
TEE normak. Pada pemeriksaan fisik didapatkan GCS 555/444, 555/444, sensorik
normal. Pasien rutin meminum obat CPG 75mg/24 jam.
Efek samping CPG dalam jangka waktu yang lama adalah :
a. Neurotropenia
b. Thrombotic thrombositopenia purpura
c. Vasodilatasi peripheral  cilos
d. Dizziness
e. Anemia

59. Pria 38 tahun ke IGD dengan keluhan kejang kelonjotan 1 jam yll. Kejang selama 5
menit, saat kejang pasien tidak sadarkan diri, setelah kejang pasien tampak mengantuk
dan bingung. Os mengatakan sakit kepala hebat mendadak saat sedang beraktivitas. Mual
muntah, HT, DM disangkal. Dari pemeriksaan didapatkan kaku kuduk ringan, nn
cranialis dbn, kekuatan motoric 555/555, sensorik dan vegetative dbn
Penyebab terjadinya kejang….
a. Stealing phenomen
b. Seninel bleed
c. Sirkulasi kolateral
d. Intraparenchymal extention
e. Peningkatan glutamate pada korteks serebral

60. Pria 38 tahun ke IGD dengan keluhan kejang kelonjotan 1 jam yll. Kejang selama 5
menit, saat kejang pasien tidak sadarkan diri, setelah kejang pasien tampak mengantuk
dan bingung. Os mengatakan sakit kepala hebat mendadak saat sedang beraktivitas.
Mual muntah, HT, DM disangkal. Dari pemeriksaan didapatkan kaku kuduk ringan, nn
cranialis dbn, kekuatan motoric 555/555, sensorik dan vegetative dbn (SKALA KLINIS
1-5)
Skala klinis pasien tsb adalah…
a. Grade 1
b. Grade II
c. Grade III
d. Grade IV
e. Grade V
Skala klinis I-IV:

144
TO UGM 17 FEBUARI 2022
Grade I : Sadar, tanpa gejala atau dengan sakit kepala ringan dan/atau ada kaku
kuduk
Grade II :Sadar, dengan sakit kepala sedang sampai berat dan ada kaku Kuduk
Grade III :Mengantuk atau Bingung, dengan atau tanpa defisit fokal neurologi
Grade IV : Stupor dengan hemiparesis sedang sampai berat dan ada tanda dr
peningkatan tekanan intra kranial
Grade V : Koma dengan tanda peningkatan tekanan intrakranial berat

61. Pria 38 tahun ke IGD dengan keluhan kejang kelonjotan 1 jam yll. Kejang selama 5
menit, saat kejang pasien tidak sadarkan diri, setelah kejang pasien tampak mengantuk
dan bingung. Os mengatakan sakit kepala hebat mendadak saat sedang beraktivitas. Mual
muntah, HT, DM disangkal. Dari pemeriksaan didapatkan kaku kuduk ringan, nn
cranialis dbn, kekuatan motoric 555/555, sensorik dan vegetative dbn
Penyebab terjadinya klinis nyeri kepala
a. Stealing phenomen
b. Sentinel bleed
c. Sirkulasi kolateral
d. Intraparenchymal extention
e. Peningkatan glutamate pada korteks serebral

62. Pria 18 tahun, datang ke poli saraf dengan keluhan sakit kepala hebat mendadak saat
sedang beraktivitas. Mual muantah (-), kelemahan anggota gerak, kesemutan, kejang
maupun penurunan kesadaran disangkal. Dari pemeriksaan fisik didapatkan kakukuduk
ringan, nn cranialis DBN, kekuatan motoric 555/555, 555/555. Sensorik dan vegetative
dbn
Tindakan yang tepat pada kasus diatas ini… SAH (grade II)
a. Pemeriksaan factor koagulasi
b. TCD
c. CT scan kepala non kontras
d. DSA
e. MRI dengan kontras

63. Wanita 47 tahun dibawa ke poliklinik dengan kelemahan anggota gerak kiri. Sejak 1
tahun yang lalu. Terdapat riw DM tipe II dan HT tidak terkontrol. EKG tidak
menunjukkan AF. TEE normal, GCS E4 M6 V5. Parese NVII dan XII sinistra sentral.
Motoric 555/444, 555/444, sensorik normal. Dari hasil DSA didapatkan gambaran sbb:

145
TO UGM 17 FEBUARI 2022
Diagnosa yg tepat pada kasus diatas adalah…..
a. Cerebral sinus venous thrombosis
b. Carotid cavernous fistula
c. Dural arteri venous fistula
d. Aneurisma intracranial
e. Artervenous malformation

64. Seorang perempuan usia 48 tahun datang dengan keluhan pusing berputar dengan durasi
< 1 menit. Pusing bertambah jika pasien melakukan perubahan posisi duduk kemudian
terlentang dan saat pasien membungkuk kedepan, setelah mendapatkan terapi, pasien
dminta untuk melakukan latihan metode bandaroff di rumah.
Manakah dibawah ini yang mendukung pernyataan diatas..
a. Latihan dilakukan 5 siklus pagi, siang sore selama 3 minggu
b. Latihan dilakukan 5 siklus pagi,dan sore selama 3 minggu
c. Latihan dilakukan 3 siklus pagi, siang sore selama 2 minggu
d. Latihan dilakukan 3 siklus pagi, dan sore selama 2 minggu
e. Latihan dilakukan 3 siklus pagi, siang sore selama 3 minggu

Brandt daroff : 3 minggu  pagi & sore; 2 minggu : pagi siang sore

65. Seorang perempuan 48 tahun datang dengan keluhan pusing berputar dengan durasi < 1
menit, pusing bertambah jika pasien melakukan perubahan posisi kepala, pasien
didiagnosa mengalami BPPV
Manakah pernyataan yang benar dibawah ini…..
a. Nistagmus bersifat bidireksional, rotatoar, vertical
b. Nystagmus bersifat bidireksional, horizontal, vertical
c. Nystagmus bersifat bidireksional, horizontal, rotatoar
d. Nystagmus bersifat unidireksional, horizontal, vertical
e. Nystagmus bersifat unidereksional, horizontal, rotatoar

66. Seorang perempuan 48 tahun datang dengan keluhan pusing berputar dengan durasi < 1
menit, pusing bertambah jika pasien melakukan perubahan posisi dari duduk kemudian

146
TO UGM 17 FEBUARI 2022
terlentang dan saat pasien membungkuk kedepan, pasien didiagnosa mengalami
BPPV
Manuver terapi manakah dibawah ini yang tidak dapat dilakukan pada pasien diatas…..
a. Maneuver barbeque terapi
b. Maneuver epley terapi
c. Maneuver lampert Roll terapi
d. Maneuver Brandt daroff(terapi)
e. Maneuver Dix-Hallpike (diagnose)

67. Seorang perempuan 48 tahun datang dengan keluhan pusing berputar dengan durasi < 1
menit, pusing bertambah jika pasien melakukan perubahan posisi dari duduk kemudian
terlentang dan saat pasien membungkuk kedepan, pasien didiagnosa mengalami BPPV.
Terapi medikamentosa yang sesuai untuk kasus diatas….
a. Flunarizine 15-20 mg/hari
b. Betahistin 12mg/hari
c. Betahistine 24 mg/ hari
d. Betahistin 48 mg/hari
e. Flunarizine 10-15mg/hari

68. Seorang perempuan usia 37 tahun merupakan mahasiswa paska sarjana. Saat ini pasien
sedang menghadapi minggu ujian. Selama seminggu ini os mengeluh pusing berputar
yang hilang timbul. Os tidak memperhatikan secara jelas frekuensi dan lamanya pusing.
Os mengaku, badan terasa panas, mual dan beberapa kali muntah, pendengaran menurun
terutama sisi kanan. Kemudian dilakukan pemeriksaan fungsi labirin dengan irigasi air
hangat 44c pada kedua saluran telinga secara bersamaan.
Hasil yang diharapkan dari pemeriksaan diatas…..
a. Didapatkan nystagmus yang mendekati aliran ke posisi kanan
b. Adanya deviasi tonik kearah bawah dengan nystagmus komponen cepat ke atas
c. Nystagmus komponen cepat menjauhi aliran posisi kiri
d. Tidak didapatkan nystagmus maupun perubahan posisi bola mata
e. Terdapat nystagmus komponen cepat kebawah dengan deviasi tonik ke arah
atas

Kalau unilateral kalori  nistagmus horizontal  COWS


Kalau bilateral kalori  nistagmus vertikal  Cold Slow thing down / kalau panas cepat
buka bawah

69. Seorang laki-laki 46 tahun datang ke poliklinik saraf dengan keluhan kelopak mata turun
pada sisi kanan sejak 2 hari yll. Hal ini timbul dirasakan semakin lama semakin
memberat disertai rasa tidak seimbang seperti mau jatuh (ataxia), os juga mengeluh

147
TO UGM 17 FEBUARI 2022
pandangan dobel saat melihat ke bawah sisi medial. Pemeriksaan tandem gait didapatkan
os terjatuh kesisi kanan saat mata terbuka.
Diagnosis yg tepat untuk kasus diatas…
a. Sindrom benedict ( hemiparese kontralateral, parese N III ipsilateral, tremor
cerebellar, hemiataxia kontralateral))
b. Sindrom Raymond (parese VI ipsilateral, hemiparese kontralateral)
c. Sindrom nothnegel (parese III ipsilateral, ataxia cerebellar ipsilateral
d. Syndrome Wallenberg/lateral medullary syndrome(horner syndrome,ataxia, disfasia,
ptosis parsial ipsilateral)mioklonik palatal)
e. Sindrom cerebellum

70. Seorang laki-laki 46 tahun datang ke poliklinik saraf dengan keluhan kelopak mata turun
pada sisi kanan sejak 2 hari yll. Hal ini timbul dirasakan semakin lama semakin
memberat disertai rasa tidak seimbang seperti mau jatuh, os juga mengeluh pandangan
dobel saat melihat ke bawah sisi medial. Pemeriksaan tandem gait didapatkan os terjatuh
kesisi kanan saat mata terbuka. ( syndrome nothnagel)
Dimanakah diagnose topis dari kasus di atas…
a. Dorsal midbrain  parinaud
b. Base of pons
c. Tectum of midbrain
d. Base of midbrain
e. Tegmentum of medulla

71. Seorang laki-laki 46 tahun datang ke poliklinik saraf dengan keluhan kelopak mata turun
pada sisi kanan sejak 2 hari yll. Hal ini timbul dirasakan semakin lama semakin
memberat disertai rasa tidak seimbang seperti mau jatuh, os juga mengeluh pandangan
dobel saat melihat ke bawah sisi medial. Pemeriksaan tandem gait didapatkan os terjatuh
kesisi kanan saat mata terbuka.
Apakah penyebab tersering dari kasus diatas…
a. Infeksi
b. Aneurisma
c. Tumor
d. Infark
e. Perdarahan

72. Seorang wanita 35 tahun, dirujuk dari RS didaerah dengan keluhan mendengar suara
gemuruh dikepala, disertai dengan nyeri kepala yang menetap sejak 2 hari ini dan
memberat saat batuk dan mengejan. Pasien juga mengeluh seperti melihat lorong.
Pada saat diperiksa tekanan darah pasien 180/100 mmhg, pasien memiliki riwayat HT
namun tidak minum obat. BMI 30 kg/m. pasien membawa hasil CT scan kepala yang
tampak normal. Setelah dilakukan pemeriksaan funduskopi didapatkan papil edema
bilateral.

148
TO UGM 17 FEBUARI 2022
Apakah tatalaksana pada pasien tersebut… IIH idiopatik inrakranial Hipertensi
/pseudotumor
a. Injeksi ketorolac 30mg/iv
b. Lakukan CT scan kepala ulang dengan kontras
c. Lumbal pungsi untuk menurunkan tekanan LCS 30mmH20 
d. Mannitol 125mg/8jam/iv
e. Menurunkan berat badan dengan agresif

73. Seorang wanita 65 tahun dirawat inap karena menolak makan dan minum sehingga badan
lemas. Riwayat cabut gigi graham kiri 7 minggu yg lalu. Pasien mengeluh nyeri pada
area wajah sisi kiri bila terkena angin dan bila kesakitan pasien menangis. Pasien
tampak melindungi wajah sisi kirinya dengan tangan.
Apakah diagnosis pasien diatas.
a. Tics Douloureux (neuralgia trigeminal)
b. Tics facialis
c. Hemifacial spasme
d. Neck tongue syndrome
e. Nyeri kepala yg berkaitan dengan keluhan lain

74. Seorang laki laki 26 tahun berkacamata tebal, datang ke poli saraf mengeluhkan nyeri
berdenyut dibelakang kedua mata dan mata tampak kemerahan. Pasien dengan
riwayat kosumsi obat epilepsy selama 2 tahun. Pada pemeriksaan didapatkan injeksi
konjungtiva bilateral. TD 160/90 mmHg. D/Glaucoma
Obat epilepsy manakah yg dapat menimbulkan nyeri kepala tipe diatas…
a. Levetiracetam
b. Asam valproate
c. Topiramate & Zonizamid
d. Phenytoin
e. Clobazam

75. Seorang pria sedang mendaki gunung merbabu dan mengeluh nyeri diseluruh kepala yg
memberat saat bergerak dan membungkuk. Semakin naik semakin terasa nyeri. Pasien
seringkali mengalami hal ini saat mendaki gunung dan naik pesawat.
Yg tidak termasuk karakteristik nyeri kepala tipe ini adalah….
a. Nyeri kepala bilateral
b. Intensitas ringan sedang
c. Membaik dalam waktu 24 jam
d. Membaik dengan penurunan ketinggian dibawah 2000m (2500m)
e. Diperburuk dengan batuk (bungkuk, ketegangan, aktivitas fisik)

76. Seorang wanita mengeluh nyeri kepala. Pasien terbiasa minum kopi lebih dari 10 cangkir
(>200mg) perhari karena nyeri ulu hati pasien memutuskan tidak minum kopi.
Manakah yg seusai dengan diagnose pasien ini….

149
TO UGM 17 FEBUARI 2022
a. Nyeri kepala muncul 12 jam setelah asupan kafein berakhir (setelah 24 jam)
b. Nyeri kepala membaik dalam waktu 1 jam setelah konsumsi kafein 200mg (perbaikan
cukup 100 mg)
c. Nyeri kepala diakibatkan konsumsi kafein lebih dari 200mg perhari selama lebih
seminggu, yg terputus (2 mingguu konsumsi)
d. Nyeri kepala sembuh dalam waktu seminggu setelah withdrawal kafein
e. Nyeri kepala memburuk setelah 2 minggu withdrawal kafein (tidak ada guys)

77. Seorang anak lakilaki usia 4 tahun mengeluh nyeri tumpul pada perut bagian tengah
dengan intensitas sedang-berat, disertai muntah dan nafsu makan menurun. Pasien sudah
mengeluh mual sejak 2 hari, total serangan 10 kali, dengan bebas gejala diantara
serangan.
Diagnose pada kasus diatas adalah…..
a. Cyclic vomiting syndrome
b. Recurrent gastrointestinal disturbances
c. Functional abdominal pain syndrome
d. Irritable bowel syndrome
e. Migrain abdominal

78. Seorang anak lakilaki usia 4 tahun mengeluh nyeri tumpul pada perut bagian tengah
dengan intensitas sedang-berat, disertai muntah dan nafsu makan menurun. Pasien sudah
mengeluh mual sejak 2 hari, total serangan 10 kali, dengan bebas gejala diantara
serangan.
Terapi terbaik yg dapat diberikan kpd pasien ini..
a. Naproxen sodium 275 mg/6 jam/ hari
b. Domperidone 10mg po 20 menit sebelum pemberian analgesic (karena dibawah
12 thn) migrain pd anak dan remaja anti emetiknya dompe, analgetiknya Cuma PCT
dan ibuprofen dan triptan spray : selain itu kontra indikasi
c. Metoclopramide 10mg iv 30 menit setelah pemberian ergotamine derivate  di pokdi
metoklopramid , profilaksis Beta bloker????
d. Diclofenac potassium powder 200mg terbagi 2 dosis
e. Narariptan dikombinasi dengan dihidroergotamin

79. Seorang wanita 45 tahun mengeluhkan nyeri kepala sisi kiri sejak 4 hari yll dan tidak
membaik dengan minum obat Pereda nyeri kepala yang dibeli bebas di apotek. Keluhan
disertai mual muntah. Pasien tidak dapat beraktivitas. Tidak ada demam sebelumnya.
Terapi farmakologis yang paling tepat untuk diberikan pada pasien ini adalah.
a. Rawat inap, injeksi triptan dan injeksi metoclopramide
b. Rawat jalan setelah injeksi metoclopramide dan diresepkan dihidroergotamin oral
c. Rawat jalan setelah injeksi sumatriptan 6 mg subkutan, dan diresepkan bisoprolol
1x5mg oral
d. Rawat jalan setelah injeksi metoclopramide dan diresepkan amitriptilin 1x25mg

150
TO UGM 17 FEBUARI 2022
e. Rawat inap, CT scan kepala cito, dan aspirin 300mg oral

80. Seorang mahasiswi 25 tahun mengeluh nyeri seluruh kepala seperti diikat, nyeri kepala
sebenernya ringan namun sering terjadi sehingga mengganggu konsentrasi pasien dalam
belajar. Dalam sebulan bisa kambuh lebih dari 20x dan keluhan ini sudah berlangsung
mungkin 2 bulan ini. Teman kos pasien menyarankan untuk dipijat dikepala namun terasa
nyeri ketika dipijat. TTH dengan pericranial tenderness - CUIT 126
Terapi apa yang paling tepat untuk pasien ini.
a. 325 aspirin, acetaminophen + 40mg kaffein
b. Acetaminophen 4x100mg
c. Amitriptilin 3x10mg
d. Oksigen 100% 7 lpm selama 15 menit
e. Sumatriptan injeksi subkutan 6 mg

TTH infrekuen <1 hari/bulan


TTH Frekuen 1-14 hari/bulan
TTH kronis >15 hari/bulan

81. Seorang perempuan usia 70 tahun, dalam satu tahun ini mengalami kesuilitan untuk
berjalan, tremor pada tangan kanan dan rigiditas. Pasien didiagnosa sebagai penyakit
Parkinson dan keluhan membaik dengan pengobatan rutin.
Bila penyakitnya berkembang, berkurangnya kemampuan bicara manakah yg sesuai
untuk pasien tersebut.
a. Suara yg tidak terdengar progresif
b. Afasia reseptif  wernicke
c. Afasia ekspresif Broca
d. Ekolalia
e. Neologisme

82. Seorang laki-laki 64 tahun dengan tremor dan kaku pada tangan kanan. Pasien terlihat
menyeret kaki kanannya ketika berjalan. Pada pemeriksaan terdapat tremor pada tangan
kanan yang menghilang saat pasien mengambil pensil. Pergerakan lebih lambat pada sisi
kanan dibandingkan kiri. (stadium 1) Terdapat kekauan pada lengan kanan.
Diagnosis yg tepat pada pasien tersebut…
a. Dyskinesia tardif
b. Dystonia responsive dopa
c. Penyakit Parkinson
d. Tortikolis spasmodic
e. Spasme hemifasial

151
TO UGM 17 FEBUARI 2022
83. Seorang perempuan 65 tahun mengeluh sering lupa sejak 8 bulan terakhir. Penderita
mudah menangis, sering melihat objek dan mendengar suara suara tertentu. Riwayat
penyakit dahulu disangkal pemeriksaan fisik umum dbn. Pemeriksaan skrinning MMSE
adalah 17 (sedang). ADL sehari hari pasien dibantu oleh keluarga.
Pada fase akhir penyakit ini gangguan memori yg terganggu.
a. Remote memory (fase akhir)
b. Immediate memory (langsung ulang saat disuruh)
c. Recent memory (fase awal) (memory autobiografinya pasien)
d. New learning ability (recent memory)
e. Kalkulasi
Working memory (memory jangka pendek)

84. Seorang wanita 45 tahun, menjalani pemeriksaan mata karena keluhan pandangan
kabur 4 jam setelah ditetesi dengan midriatil. Pasien mengeluh mata kanan terasa
nyeri, pandangan kabur disertai mual dan muntah.
Apakah diagnosis pada pasien tersebut…
a. Amourusis fugax
b. Glaucoma sudut terbuka
c. Glaucoma sudut tertutup
d. Iritasi pada lensa mata
e. Bjjerum tangent field defect (20-30 derajat)

85. Seorang pria 20 tahun mengeluh sering menabrak benda disekitarnya saat berjalan.
Pasien mengatakan sisi kanan dan kiri terlihat gelap. Keluhan dimulai dari sisi atas.
Pasien juga mengeluh sering nyeri kepala sejak 1 tahun terakhir. Pada pemeriksaan
didapatkan hemianopsia bitemporal. (lesi chiasma optikus)
Apakah diagnosis yg paling mungkin pada kasus diatas…..
a. Sindroma sinus cavernosus
b. Meningioma para selae
c. Hydrocephalus obstruktif
d. Tumor ptituary
e. Kraniofaringioma

86. Seorang wanita 65 tahun mengeluh pandangan kabur. Saat dilakukan pemeriksaan reflek
cahaya pada mata kanan, pupil mata kiri tidak konstriksi. Saat cahaya diarahkan pada
mata kiri, pupil mata kiri dan kanan berkonstriksi.
Apakah diagnosis klinis dan topis pada pasien diatas…
a. Afferent pupillary defect dengan topis di traktus optikus kanan

152
TO UGM 17 FEBUARI 2022
b. Efferent pupillary defect dengan topis di nervus opticus kanan
c. Afferent pupillary defect dengan topis di nervus optikus kanan (RAPD)
d. Afferent pupillary defect dengan topis di nervus optikus kiri
e. Afferent pupillary defect dengan topis di visura calcarina

87. Seorang wanita 55 tahun, mengeluh mendadak buta. Pada pemeriksaan fisik didapatkan
dengan BMI 30 kg/m, TD 180/90 mmHg, riwayat DM dan dyslipidemia, hasil
funduskopi adalah sebagai berikut: CRAO

Apakah kelainan dari gambaran funduskopi diatas dan apakah pathogenesis yg sesuai.
a. Fovea cherry red spot yg disebabkan oleh oklusi arteri retina central
b. Mikroaneurisma arteri retina yg disebabkan oleh diabetes melitus
c. Retinal drusen yg disebabkan oleh penumpukan lipofuhsin dan debris retina
d. Cotton wool patches yg disebabkan oleh obstruksi precapillary arteriola
e. Roth spot yg disebabkan oleh cardioembolic retinopati

88. Seorang pria 65 tahun, mengeluh mendadak buta, tidak disertai nyeri pada mata. Pada
pemeriksaan fisik didapatkan TD 180/90 mmHg. GDS 350 mg/dl, cholesterol 250mg/dl.
Hasil funduskopi adalah sbb:

Apakah kelainan yg didapatkan dari pemeriksaan funduskopi dan kemungkinan


penyebabnya. CRVO

153
TO UGM 17 FEBUARI 2022
a. Turtous vein, dot and blots hemorrhage, disebabkan oleh oklusi vena retina
sentral
b. Hollenhorts plagues, platelet fibrin emboli, disebabkan oleh oklusi arteri retina
superior
c. Fovea cherry redspot yg disebabkan oleh oklusi arteri retina sentral
d. Cotton wool patches yg disebabkan oleh obstruksi precapillary arteriole
e. Macular star, penumpukan lipid disekitar fovea yg disebabkan oleh hipertensi kronis
dan DM

89. Seorang laki laki usia 63 tahun mengeluh nyeri kepala yg makin memberat disertai
penciuman berkurang dan gangguan penglihatan sejak 9 bulan yg lalu. Pada pemeriksaan
didapatkan anosmia pada lubang hidung sisi kanan. D/ foster kenedy lesi Lobus frontal
(meningioma olfactory groove)
Manakah temuan klinis yg sesuai dengan diagnosis pasien ini.
a. Pada funduskopi didapatkan papil edema sisi kiri dan disc atrophy sisi kanan
b. Pada funduskopi didapatkan papil edema sisi kanan dan disc atropi sisi kiri
c. Hemianopsia bitemporal
d. RAPD mata kiri positif
e. Pupil anisokor mata kanan
90. Seorang wanita 56 tahun dengan riwayat hipertensi dan DM, mengeluhkan mata kanan
kabur. Beberapa hari kemudian pasien merasa gelap saat melihat keatas dan kebawah yg
makin memburuk. Pasien tidak mengeluhkan adanya nyeri pada mata kanan. Hasil
pemeriksaan reflek cahaya didapatkan afferent pupillary defect. Hasil funduskopi
didapatkan edema disertai flame shaped hemorrhage. (NII mata kanan)  NA-AION
Apakah diagnosis yg paling mungkin pada kasus tersebut.
a. Anterior ischemic optic neuropathy
b. Idiopatic intracranial hypertention
c. Papilledema due to increased intracranial pressure
d. Optic neuritis
e. Central retinal arterial occlusion

91. Seorang wanita 23 tahun datang dengan keluhan beberapa hari ini. Penglihatan kabur yg
semakin memburuk hingga pandangan sisi samping kanan gelap. Pada pemeriksaan
didapatkan hemianopsia homonym kanan. Pada pemeriksaan MRI T2 dan T1 kontras
didapatkan hasil lesi traktus optikus kiri seperti dibawah ini  optic neuritis

154
TO UGM 17 FEBUARI 2022
Terapi yg dapat diberikan kepada pasien ini adalah.
a. Metilprednisolon
b. Antibiotik broad spektrum
c. Antikoagulan dan antiplatelet
d. Neurotropik
e. Neuroprotektan

92. Seorang laki laki 26 tahun keluhan demam naik turun sejak 5 hari yll disertai keluhan
menggigil, pasien terlihat mengantuk dan sesekali bicara meracau. Pada pemeriksaan
funduskopi didapatkan gambaran sbb malaria cerebral

Apakah temuan funduskopi yg paling tepat pada kasus tersebut….


a. Perdarahan retina
b. Bercak Roth  bacterial endocarditis atau multiple systemic illness
c. Pemutihan macula  bercak roth
d. Fenomena tramlining  penampakan alur putih di sisi dalam pembuluh darah
besar yang mengikuti lekukan pembuluh darah besar tersebut.
e. Pemudaran warna pembuluh darah

155
TO UGM 17 FEBUARI 2022
93. Seorang laki laki 43 tahun dengan tumor Pancoast, mengeluh kelopak mata kanan tampak
menyempit, namun bola mata tampak sedikit masuk ke dalam. Pasien juga mengeluh
wajah sebelah kiri tidak berkeringat meskipun cuaca panas. Dx Horner Syndrome Ordo 2
Apakah gambaran klinis lain yg dapat ditemukan pada pasien tersebut..
a. Ptosis akibat disfungsi m. levator palpebra
b. Ptosis akibat disfungsi m. tarsalis superior
c. Miosis akibat disfungsi dilatasi spingter pupil
d. Midriasis akibat hilangnya hiperaktif dilator pupil
e. Midriasis akibat hilangnya fungsi m. sfingter pupil

94. Seorang laki laki 59 tahun dikonsulkan dengan penurunan kesadaran. Pasien
mempunyai riwayat cedera kepala 7 hari. Pada pemeriksaan neurologis didapatkan mata
pasien melirik ke sisi kiri dan hemiparese kanan.
Dimanakah letak lesi pada kasus tersebut..
a. Korteks kanan
b. Korteks kiri
c. Pons kanan
d. Pons kiri
e. Korteks bilateral

95. Seorang perempuan usia 75 tahun dirujuk klinik memori dgn stroke iskemik. Keluarga
pasien mengeluh saat pasien berbicara walaupun akan tetapi sering keluar kata kata
aneh. Pemeriksaan neurobehaviour menunjukkan pemahaman baik, penamaan dan
repetisi terganggu.
Kemungkinan lesi pada pasien ini terletak pada…..
a. Area broca
b. Area Wernicke
c. Girus kalkarina
d. Area prefrontalis
e. Fasikulus arkuata : pada operculum parietal kiri ke frntal kiri

96. Seorang pria 70 tahun dengan keluhan gangguan berjalan sejak 1 tahun terakhir,
mengeluh sulit untuk memulai melangkah dan ayunan tangan semakin lambat. Beberapa
bulan kemudian pasien juga mengeluh sering lupa kejadian yg baru terjadi (demensia),
lupa meletakkan jam dan kunci, dan mengalami gangguan dalam berhitung sehingga
pasien mengalami kesulitan dalam melakukan aktivitas sehari hari. Sejak lima bulan
terakhir pasien mengalami gangguan BAK (inkontinensia). Tidak dijumpai riwayat
stroke, HT dan DM. pemeriksaan neurologis MMSE 24/30, dengan gangguan pada
proses recall dan kalkulasi. Pemeriksaan motoric DBN. Pemeriksaan gait menunjukkan
wide based stance, pemeriksaan imajinh dengan MRI menunjukkan dilatasi simetris
dari seluruh ventrikel dan tidak dijumpai abnormalitas lain.
Apakah diagnosis yg paling tepat pada pasien tersebut..
a. Demensia vascular

156
TO UGM 17 FEBUARI 2022
b. Normal Pressure Hydrocephalus (ADI)  LP
c. Demensia Alzheimer
d. Demensia Lewy Bodies
e. Demensia Parkinson

97. Seorang pasien perempuan berusia 68 tahun datang berobat dengan keluhan sulit
berjalan sejak 1 tahun terakhir, dimana pasien mengeluh sulit untuk memulai langkah
dan ayunan tangan semakin lambat. Beberapa bulan kemudian, pasien juga mengeluh
sering lupa kejadian yg baru terjadi, lupa meletakkan jam dan kunci, lupa berhitung
sehingga pasien mengalami kesulitan dalam melakukan aktivitas sehari-hari. Sejak lima
bulan terakhir pasien juga mengalami gangguan dalam BAK (beser). Tidak dijumpai
riwayat stroke, HT, dan DM. pemeriksaan fisik umum DBN. Pemeriksaan neurologis
menunjukkan skor MMSE 24/30, dengan gangguan pada proses recall dan kalkulasi.
Pemeriksaan motoric menunjukkan kekuatan motoric dan tonus normal di keempat
ekstremitas, reflek tenson normal, tidak ada reflek patologis. Pemeriksaan gait
menunjukkan wide based stance, langkah melambat, pendek, gemetar, dan tungkai tidak
terangkat tinggi dari lantai. Pemeriksaan laboratorium DBN. Pemeriksaan imajing MRI
menunjukkan dilatasi simetris dari seluruh ventrikel dan tidak dijumpai abnormalitas lain.
Diagnosis pasien ini adalah…
a. Multiple System Atrophy
b. Parkinsons disease
c. Idiopathic Normal Pressure Hydrocephalus
d. Shy-Drager Syndrome
e. Parkinson’s Disease Dementia

98. Seorang pria 70 tahun dengan keluhan gangguan berjalan sejak 1 tahun terakhir,
mengeluh sulit untuk memulai melangkah dan ayunan tangan semakin lambat. Beberapa
bulan kemudian pasien juga mengeluh sering lupa kejadian yg baru terjadi, lupa
meletakkan jam dan kunci, dan mengalami gangguan dalam berhitung sehingga pasien
mengalami kesulitan dalam melakukan aktivitas sehari hari. Sejak lima bulan terakhir
pasien mengalami gangguan BAK. Tidak dijumpai riwayat stroke, HT dan DM.
pemeriksaan neurologis MMSE 24/30, dengan gangguan pada proses recall dan kalkulasi.
Pemeriksaan motoric DBN. Pemeriksaan gait menunjukkan wide based stance,
pemeriksaan imajinh dengan MRI menunjukkan dilatasi simetris dari seluruh ventrikel
dan tidak dijumpai abnormalitas lain.
Trias gejala / tanda yang didapatkan pada pasien ini adalah..
a. Demensia, akalkulia, gangguan gait
b. Demensia, gangguan gait, inkontinensia
c. Gangguan gait, anomia, inkontinensia
d. Akalkulia, gangguan gait, inkontinensia
e. Demensia, inkontinensia, akalkulia

157
TO UGM 17 FEBUARI 2022
99. Laki laki 72 tahun, keluhan kesulitan mengingat 2 tahun terakhir disertai bicara pelan
dan tidak lancer, penderita tampak apatis. Riwayat hipertensi sejak lama minum obat
amlodipine 1x5mg. hasil MMSE adalah 20. Pemeriksaan neurologis lain dalam batas
normal.
Kemungkinan diagnosis pada pasien ini adalah..
a. Definite penyakit Alzheimer (biopsy otak)
b. Probable penyakit Alzheimer
c. Progressive supranuclear palsy
d. Normal pressure hydrocephalus
e. Keadaan polifarmasi

100. Laki-laki usia 63 tahun berobat ke klinik memori dengan keluhan sejak 1 tahun
yll sering lupa kejadian yg baru, lupa meletakkan barang. Pasien masih dapat melakukan
kegiatan sehari-hari, berbelanja, transaksi ke bank. Riwayat hipertensi terkontrol dengan
obat. Hasil pemeriksaan neuropsikologis: MMSE: 28, CDT 4, CDR 0,5, ADL 0.
Pemeriksaan fisik umum dan laboratorium DBN. MRI menunjukkan: infark watershed di
parieto-occipital kiri, leucoaraiosis periventricular, volume hippocampus normal.
Apakah diagnosis yg paling tepat…..
a. Pseudodemensia
b. Demensia Vaskular
c. Demensia Alzheimer
d. Demensia Lewy Body
e. Vascular Cognitive Impairment

101. Laki-laki 74 tahun datang dengan keluhan nyeri kepala berdenyut. Keluhan ini
dirasakan pasien 7 hari setelah terjatuh di kamar mandi, tidak didapatkan keluhan tidak
sadar sesaat setelah jauh, muntah (-), namun didapatkan adanya luka lecet di dahi sebelah
kiri, dan bekas jahitan di dahi sebelah kiri. Dari pemeriksaan MSCT kepala didapatkan
hanya gambaran perdarahan bikonkaf (SDH).
Etiologi yg paling mungkin dari kasus diatas adalah..
a. Ruptur AVM
b. Ruptur bridge vein
c. Rupture Arteri Meningea Anterior
d. Ruptur Aneurisma
e. Ruptur Arteri Meningea Media EDH

102. Pasien wanita 34 tahun dibawa oleh keluarga ke UGD dengan keluhan tiba tiba
tidak sadarkan diri saat lagi beraktivitas. Alloanamnesis: pasien terlihat pucat berkeringat,
sesaat sebelum pingsan pasien mengeluh pandangan menjadi kabur. Pasien juga sempat

158
TO UGM 17 FEBUARI 2022
mengatakan semalam begadang untuk menyelesaikan deadline satu tugas. Pasien
beberapa kali mengalami keluhan ini, kadang disertai kejang sesaat sebelum pingsan.
Manakah dibawah ini yg bukan merupakan gambaran klinis bangkitan epileptic…
a. Mengantuk setelah mengalami serangan
b. Inkontinensia saat serangan sering terjadi
c. Myoclonic jerks ( durasi singkat, pemulihan cepat)
d. Refleks cahaya saat pemeriksaan neurologis sering menurun
e. Serangan sering saat maam hari atau tidur

103. Anak umur 8 tahun datang dengan keluhan gerakan tidak terkontrol pada sisi
lengan sebelah kiri , keluhan ini berlangsung 1- 2 menit, sebelum muncul keluhan
biasanya pasien mengeluh ada rasa tidak enak pada area perut disertai rasa ingin muntah,
riwayat penyakit dahulu, pasien sering mengalami kejang demam yang berlangsung lebih
dari 5 menit dengan frekuensi 2-4 serangan dalam 24 jam
Manakah yang merupakan etiologi tersering penyebab dari jenis epilepsi yang dialami
oleh pasien ini….TLE
a. Encephalitis
b. Malformasi Vaskuler
c. Tumor
d. Sklerosis Hipokampus
e. Displasia

104. Seorang pasien Laki-laki 65 tahun mengeluh sering lupa sejak 9 bulan terakhir.
Penderita gampang menangis, sering melihat objek dan mendengar suara-suara terntentu.
Riwayat penyakit dahulu disangkal. Pemeriksaan fisik umum DBN. Pemeriksaan
skrinning MMSE adalah 17.
Lokasi tersering terjadinya mutase pada kasus diatas adalah.
a. Gen ApoE4 late onset
b.
c. Gen Presenilin -1 eaarly
d. Gen Parkin early
e. Gen ApoE 2 eaarly
f. Gen Presenilin 2 eaarly

105. Wanita 19 tahun dengan keluhan gerakan tidak terkontrol pada kedua bahu
menjalar sampai ke lengan , sering menjatuhkan barang secara tidak disengaja, serangan
biasanya muncul saat bangun tidur di pagi hari , kadang pasien juga sering bengong
selama beberapa saat tanpa disadari. Akibat hal ini pasien menjadi malu dan sulit untuk
bergaul dengan teman sebayanya, sehingga pasien menjadi selalu murung dan menjadi
tampak tidak bersemangat, mood pasien sering berubah rubah, pasien sempat dibawa ke
psikolog dan didiagnosa menderita depresi

159
TO UGM 17 FEBUARI 2022
Manakah OAE dibawah ini yang harus dihindari untuk pemberian pada pasien epilepsi
disertai dengan adanya depresi… obat JME LALEVATO
a. Clobazam
b. Lamotrigin
c. Topiramate : bikin depresi
d. Oxcarbazepine
e. Pregabalin

106. Pria 23 tahun dengan penyandang epilepsy sejak 5 tahun yll, dalam beberapa
bulan terakhir kejang makin sering terjadi, pemeriksaan laboratorium menunjukkan
SGOT : 150 dan SGPT : 230.
Manakah OAE yang tidak boleh diberikan pada kasus tersebut…..
a. Fenitoin : tidak aman
b. Pregabalin aman
c. Klonazepam aman
d. Levetiracetam aman
e. Lamotigrin
Yg dimetabolisme di hepar : Tiagabin Topiramat-LamoZonisamid, (TT-Lazo) TiTo
LaZo clona clobazam feni, as valp, Karba, etusiksimid oxcarbazepine( CloCloFAK
EO)

107. Pria 23 tahun dibawa ke IGD dengan keluhan kejang berulang kali , setiap
episode kejang berlangsung lebih dari 2 menit, kejang kelonjotan seluruh tubuh, saat tiba
di IGD kejang klonjotan seluruh tubuh, saat tiba di IGD telah berlangusng lebih dari 6
kali. Kesadaran tidak pernah membaik. Pasien 3 bulan yang lalu didiagnosa dengan
epilepsi dan telah mendapat pengobatan tetapi serangan kejang nya masih belum
berkurang. Pada pemeriksaan neurologis didapatkan lateralisasi kanan.
Pemeriksaan penunjang apa yang harus segera dilakukan saat pasien ini tiba di UGD…..
a. Pemeriksaan Darah dan Elektrolit
b. Lumbal Punksi
c. MRI Otak
d. EEG
e. CT Scan Kepala

108. Wanita 50 tahun dibawa ke IGD setelah mengalami kejang saat berada di kantor,
pasien sempat tidak sadar beberapa saat (1), setelah bangun pasien tampak kebingungan,
satu bulan lalu rekan kerja pasien mengatakan pasien pernah mengalami keluhan yang
sama.
Berapakah nilai STESS (Status Epilepticus Severity Score) pada pasien ini ….
a. 3
b. 6
c. 2
d. 1

160
TO UGM 17 FEBUARI 2022
e. 0

109. Seorang pasien datang berobat dengan keluhan gangguan tidur. Anda sebagai
dokter mencoba untuk mengases keluhan pasien dengan cara mengukur secara umum
tingkat kantuk pasien di siang hari atau kecenderungan tidur rata rata mereka dalam
kehidupan sehari hari.
Asesmen apa yang cocok digunakan sesuai dengan pemaparan yang disebutkan di atas..
a. Sleep Diaries kualitas tidur
b. Epworth Sleepiness Scale
c. Insomnia Severity Indeks Insomnia
d. Sleep Condition Indikator Insomnia
e. Pittsburgh Sleep Quality Indeks kualitas tidur (insomnia jg bs)

110. Seorang pasien datang dengan keluhan susah tidur kurang lebih 3 minggu , hal ini terjadi
setiap malam , sehingga keesokan harinya penderita merasa mengantuk konsentrasi berkurang
dalam melakukan pekerjaan nya, hal ini mempengaruhi performa kerja pasien sehingga banyak
tugasnya yang menjadi terbengkalai dan menyebabkan dia sering dimarahi atasan nya.
Apakah jenis Insomnia yang dialami oleh penderita ini….
a. Insomnia Akut derajat ringan
b. Insomnia Akut derajat sedang
c. Insomnia Kronis derajat sedang
d. Insomnia Akut derajat berat
e. Short Term insomnia derajat sedang
Akut < 1 bulan, transien : beberapa hari, short time 3-4 mgg
Kronis 1-3 bulan

111. Seorang pasien 40 tahun mengeluhkan susah untuk memulai tidur pada malam hari,
kalaupun akhirnya bisa tertidur pasien susah untuk terlelap dan biasanya akan terbangun cepat
keesokan harinya, keesokan harinya pasien merasa lemass dan lelah , kurang konsentrasi , cepat

161
TO UGM 17 FEBUARI 2022
menjadi marah dan mengantuk sepanjang hari , hal ini dialami kurang lebih 2 bulan sebelum
akhirnya pasien datang berobat
Apakah jenis Insomnia yang dialami oleh penderita ini. Menurut ICSD 3
a. Insomnia jangka pendek
b. Insomnia Kronis
c. Transient insomnia
d. Sleep Maintenance Insomnia
e. Insomnia Primer
Menurut ICSD 3 ( kronis lebih 3 bulan)

112. Seorang wanita 40 thn mengeluhkan susah tidur setelah melakukan perjalanan
menggunakan pesawat dari papua ke medan , sumatera utara , keluhan disertai nyeri kepala,
rasa mual muntah, mata terasa kering dan hidung buntu, keluhan menghilang dengan sendirinya
setelah pasien beristirahat 2-3 hari.
Berdasarkan penjelasan diatas apakah jenis gangguan tidur yang dialami oleh wanita ini..
a. Adjusment Sleep Disorder
b. Jet Lag
c. Shift work sleep disorder
d. Transient Insomnia
e. Insomnia Akut
113. Seorang pria datang dengan keluhan mengalami kejang sesaat sebelum tidur , dia
merasakan sensasi seperti terjatuh, dirasakan pada kedua kaki kadang secara bersamaan
berlangsung > 3 detik , pada pemeriksaan EEG didapatkan gambaran Arousal termasuk
didalamnya kompleks K
Apakah gangguan yang dialami oleh pria tersebut..
a. Propiospinal myoclonus
b. Sleep Starts
c. Fragmentary Myoclonus
d. Rhytmic movement Disorders
e. Gangguan Arousal

114. Anak 7 tahun dengan keluhan sering terbangun malam hari , disertai dengan teriakan
dan gelisah , dia tidak mampu mengingat mimpinya (N REM), saat bangun biasanya tampak
kebingungan ,,kadang kala disertai dengan keringat pada seluruh tubuh , dokter memberikan
diazepam untuk membantu anak ini bisa tidur
Apakah jenis gangguan tidur yang dialami anak tersebut…..
a. Nightmare disorder REM
b. REM Behaviour Disorder
c. Hipersomnia Primer
d. Sleep Walking disorder
e. Sleep Terror Disorder

162
TO UGM 17 FEBUARI 2022
115. Seorang perempuan 45 tahun, datang ke poli saraf dengan keluhan nyeri seperti
tersengat listrik, paroksismal, durasi <2 menit, berulang pada daerah wajah sisi kanan, terutama
saat tersentuh atau menggosok gigi. Pada pemeriksaan fisik diluar serangan, tidak ditemukan
defisit neurologis.
Pembuluh darah yang paling sering terlibat dalam penekanan saraf yang menyebabkan keluhan
tersebut adalah.
a. Arteri serebelaris superior
b. Arteri serebri posterior
c. Arteri basilaris
d. Arteri vertebralis
e. Arteri serebeli anterior inferior

116. Laki-laki 50 tahun datang ke poli saraf dengan keluhan nyeri pada kedua sendi lutut
sejak 2 tahun yang lalu. Awalnya terasa kaku pada pagi hari, berkurang dengan aktivitas dan saat
ini nyeri muncul tanpa dipicu oleh gerakan. Riwayat obesitas (+).
Diagnosis yang paling tepat untuk kasus tersebut..
a. Gout
b. Osteoartritis genu
c. Reumatoid artritis
d. Iliotibial band syndrome
e. Bursitis

117. Laki-laki 50 tahun, dibawa ke poli saraf dengan keluhan nyeri seolah terbakar pada
separuh tubuh kanan sejak 4 bulan lalu, riwayat stroke 7 bulan lalu pada sisi yang sama. CPSP
Terapi lini pertama untuk kasus ini.
a. Amitriptilin & lamotrigin
b. Gabapentin
c. Lamitrigin
d. Karbamezipin
e. Clobazam

118. Seorang perempuan 45 tahun, datang ke poli saraf dengan keluhan nyeri seperti
tersengat listrik, paroksismal, durasi <2 menit, berulang pada daerah wajah sisi kanan, terutama
saat tersentuh atau menggosok gigi. Pada pemeriksaan fisik diluar serangan, tidak ditemukan
defisit neurologis. Terapi lini pertama kurang efektif. Neuralgia trigeminal
Terapi farmakologis lini kedua pada kasus ini..
a. Karbamazepin
b. Baclofen
c. NSAID
d. Amtriptilin
E. Tizanidine

163
TO UGM 17 FEBUARI 2022
119. Laki-laki 40 tahun, datang ke poli saraf dengan keluhan nyeri hebat pada dada kanan
dirasakan seolah tertusuk dan terbakar sejak 3 bulan lalu. Riwayat penyakit kulit pada daerah
tersebut 5 bulan yang lalu. Neuralgia post herpetik
Dimanakah topis asal keluhan tersebut.
a. Ganglion dorsalis
b. Cornu posterior
c. Traktus spinotalamikus
d. Cornu anterior
e. Ganglion ventralis
. Pada episode herpes zoster akut setelah beberapa saat sebelumnya terinfeksi virus
Varisela dan imunitas selular menurun, maka virus yang dorman di dorsal ganglion sensori
akan teraktivasi, ber-replikasi, dan memperbanyak diri disepanjang saraf yang terkena,
sehingga mencetuskan respon imun berupa inflamasi

164
TO UGM 17 FEBUARI 2022
120. Perempuan 35 tahun datang dengan keluhan nyeri kepala unilateral, seperti tertusuk-
tusuk di sekitar mata dan pelipis, disertai mata berair, kemerahan, dan hidung tersumbat.
Serangan dialami selama 20 kali sehari selama 10-15 detik (1-600 detik), serangan dialami sudah
3 kali dalam setahun, sempat mengalami bebas nyeri kepala 1,5 bulan. Pasien telah
mengkonsumsi obat anti nyeri yang dibeli diwarung tetapi tidak terdapat perbaikan. di suatu
perusahaan. Riwayat penyakit dahulu : trauma kepala pada usia 5 tahun. Pada pemeriksaan
neurologis : tidak ditemukan defisit neurologis. Pemeriksaan umum : tekanan darah 120/80
mmHg, nadi 80 kali per menit, respirasi 20 kali per menit, suhu 36,5 C. hasil CT Scan kepala:
batas korteks dan girus jelas, system ventrikel, batang otak normal, tidak didapatkan lesi
hipodens atau hiperdens.
Apakah diagnosis penyakit pasien tersebut…
a. Sindroma Tolosa Hunt
b. Hemikrania Paroksismal
c. SUNCT
d. Migrain oftalmoplegik
e. Nyeri kepala cluster

121. Perempuan 35 tahun datang dengan keluhan nyeri kepala unilateral, seperti tertusuk-
tusuk di sekitar mata dan pelipis, disertai mata berair, kemerahan, dan hidung tersumbat.

165
TO UGM 17 FEBUARI 2022
Serangan dialami selama 20 kali sehari selama 10-15 detik, serangan dialami sudah 3 kali dalam
setahun, sempat mengalami bebas nyeri kepala 1,5 bulan. Pasien telah mengkonsumsi obat
antinyeri yang dibeli diwarung tetapi tidak terdapat perbaikan. di suatu perusahaan. Riwayat
penyakit dahulu : trauma kepala pada usia 5 tahun. Pada pemeriksaan neurologis : tidak
ditemukan defisit neurologis. Pemeriksaan umum : tekanan darah 120/80 mmHg, nadi 80 kali
per menit, respirasi 20 kali per menit, suhu 36,5 C. hasil CT Scan kepala: batas korteks dan girus
jelas, system ventrikel, batang otak normal, tidak didapatkan lesi hipodens atau hiperdens.
D/SUNCT
Apakah tatalaksana yang tepat untuk kasus di atas?
a. Terapi profilaksis prednisone
b. Terapi abortif topiramat
c. Terapi abortif lidokain iv/subcutan , profilaksis lamotrigine 25 mg/hari
d. Terapi abortif lamotigrin
e. Terapi profilaksis valproat

122. Perempuan, 24 tahun seorang pekerja kantoran datang ke poliklinik saraf dengan
keluhan nyeri kepala terutama kiri di area belakang kepala seperti tertusuk-tusuk dan
memberat jika pasien lembur kerja. Nyeri dirasakan berat dengan VAS:7 dan menjalar sampai
dengan ke pelipis. Pasien juga mengeluh mual jika nyeri kepala datang. Sebelum nyeri kepala
pasien melihat kilatan cahaya selama 5 menit kemudian nyeri kepala. Pasien sering memakai
soflens dan sering mengeluh kemerahan dan berair pada matanya. Pasien sering mekonsumsi
paracetamol jika nyeri kepala datang. Pemeriksaan neurologi: GCS: E4M6V5: Nervus cranialis :
dalam batas normal, motorik: normal, sensorik: normal. Otonom: nomal. Pemeriksaan interna
umum : dalam batas normal. MIGRAIN dngan aura
Mekanisme awal yang mendasari gejala klinis tersebut adalah..
a. Vasokonstriksi dan vasodilatasi pembuluh darah intracranial
b. Cortical spreading depression
c. Pelepasan sel inflamasi TNF alfa, IL-6, dan Il-1 beta5
d. Peningkatan glutamate pada celah post sinaps
e. Ketidakseimbangan neurotransmiter eksitasi & inhibisi

123. Perempuan, 24 tahun seorang pekerja kantoran datang ke poliklinik saraf dengan
keluhan nyeri kepala terutama kiri di area belakang kepala seperti tertusuk-tusuk dan
memberat jika pasien lembur kerja. Nyeri dirasakan berat dengan VAS:7 dan menjalar sampai
dengan ke pelipis. Pasien juga mengeluh mual jika nyeri kepala datang. Sebelum nyeri kepala
pasien melihat kilatan cahaya selama 5 menit kemudian nyeri kepala. Pasien sering memakai
soflens dan sering mengeluh kemerahan dan berair pada matanya. Pasien sering mekonsumsi
paracetamol jika nyeri kepala datang. Pemeriksaan neurologi: GCS: E4M6V5: Nervus cranialis :
dalam batas normal, motorik: normal, sensorik: normal. Otonom: nomal. Pemeriksaan interna
umum : dalam batas normal. Spesifik migrain
Terapi spesifik pada kasus di atas adalah…
a. Asam valproat

166
TO UGM 17 FEBUARI 2022
b. Amitriptilin
c. Sumatriptan
d. ASA
e. Opioid

124. Perempuan, 24 tahun seorang pekerja kantoran datang ke poliklinik saraf dengan
keluhan nyeri kepala terutama kiri di area belakang kepala seperti tertusuk-tusuk dan
memberat jika pasien lembur kerja. Nyeri dirasakan berat dengan VAS:7 dan menjalar
sampai dengan ke pelipis. Pasien juga mengeluh mual jika nyeri kepala datang. Sebelum
nyeri kepala pasien melihat kilatan cahaya selama 5 menit kemudian nyeri kepala. Pasien
sering memakai soflens dan sering mengeluh kemerahan dan berair pada matanya. Pasien
sering mekonsumsi paracetamol jika nyeri kepala datang. Pemeriksaan neurologi: GCS:
E4M6V5: Nervus cranialis : dalam batas normal, motorik: normal, sensorik: normal.
Otonom: nomal. Pemeriksaan interna umum : dalam batas normal.
Terapi profilaksis yang paling sesuai pada kasus ini adalah.
a. Propanolol 2 x 20 mg klo ada astma : asam valproat
b. Amitriptilin 1,5 mg/hari
c. Paracetamol 1.000 – 2.000 mg/hari
d. Natrium diclofenac 3 x 25 mg
e. Diazepam 2 x 2 mg

125. Seorang laki-laki 32 tahun mengeluh nyeri kepala seperti tertekan, nyeri dirasakan
tersebar di seluruh kepala, VAS: 6. Nyeri sudah dirasakan sejak 2 hari yang lalu, pasien
sudah mengalami nyeri kepala sejak 3 bulan tetapi hilang timbul. Jumlah serangan rata-
rata 3 kali dalam seminggu. Nyeri kepala tidak bertambah berat jika melakukan aktifitas
(khas pada TTH). Muntah dan mual disangkal, pada saat nyeri kepala pasien merasa
sensitive bila mendengarkan suara bising. Pemeriksaan fisik: nyeri tekan pada area
temporal dan sternokleidomastoideus. Pemeriksaan neurologi normal…..
a. TTH episodic infrekuen
b. TTH episodic frekuen
c. TTH episodic frekuen kronis
d. TTH episodic infrekuen dengan nyeri tekan pericrania
e. TTH episodic frekuen dengan nyeri tekan pericrania 12 kali di soal
Klo migrain makin beraktifitas makin memberat

126. Perempuan, 30 tahun datang ke poliklinik saraf dengan keluhan nyeri kepala
terutama kiri di area belakang kepala berdenyut dan memberat terutama saat menstruasi.
Nyeri dirasakan berat dengan VAS:7 dan menjalar sampai dengan ke pelipis. Pasien juga
mengeluh mual jika nyeri kepala datang. Sebelum nyeri kepala pasien melihat kilatan
cahaya selama 5 menit kemudian nyeri kepala. Pasien sering mekonsumsi paracetamol
jika nyeri kepala datang. Pemeriksaan neurologi: GCS: E4M6V5: Nervus cranialis :
dalam batas normal, motorik: normal, sensorik: normal. Otonom: nomal. Pemeriksaan
interna umum : dalam batas normal. Katamenial migrain
Mekanisme yang mendasari kasus di atas adalah.
a. Penurunan estrogen

167
TO UGM 17 FEBUARI 2022
b. Penurunan progesterone
c. Peningkatan LH
d. Peningkatan estrogen
e. Peningkatan FSH

127. Seorang laki-laki, 32 tahun datang ke IGD dengan keluhan nyeri kepala hebat
pada bagian kanan(unilateral), daerah sekitar mata. Nyeri kepala berlangsung 30-60
menit setiap serangan(15-180 menit), nyeri bisa dirasakan hingga 8 kali sehari. Pada
pemeriksaan fisik didapatkan : pasien gelisah, injeksi konjunctiva kanan, lakrimasi dan
kongesti nasal serta edema palpebra.
Apa terapi yang paling tepat kasus ini. DIAG/ cluster headache : OTE oksigen , triptan,
ergotamin
a. Inhalasi oksigen 7-15 liter per menit selama 15-20 menit
b. Paracetamol 500-1.000 mg + Amitriptilin 10 mg
c. Tramadol 37,5 mg + Paracetamol 500-1.000 mg
d. Kodein 10 mg
e. Diazepam 1 mg

128. Laki-laki 30 tahun di bawa ke IGD dalam keadaan tidak sadar setelah mengalami
kecelakaan lalulintas. Pasien mengendarai motor tanpa memakai helm. Pada pemeriksaan
didapatkan robek pada kepala sebelah kanan, keluar cairan bening dari telinga (otorhea)
dan memar di belakang telinga kanan (Battle sign). Bagaimanakah mekanisme terjadinya
temuan klinis pada kasus tersebut…..
A. Fraktur petrous pyramid yang merusak kanalis auditorius eksterna.
B. Fraktur basis cranii anterior yang menyebabkan darah ke jaringan periorbita racoon
eye
C. Fraktur basis cranii yang merusak membran timpani.
D. Fraktur basis cranii menyebabkan kebocoran cairan seberospinal.
E. Fraktur basis cranii yang meluas ke posterior, merusak sinus sigmoid dan jaringan di
belakang telinga
Anterior : racoon eye
Media : otorea
Posterior : battle sign
129. Perempuan 40 tahun dibawa ke unit gawat darurat dengan keluhan nyeri kepala
hebat disertai muntah proyektil lebih dari 3kali yang dialami sejak 1 jam sebelum masuk
RS akibat terjatuh dari motor. Pada pemeriksaan fisik didapatkan Tekanan Darah 140/100
mmHg, GCS E3M6V4, kaku kuduk positif. Riwayat hipertensi ada dan berobat teratur
dengan amlodipine 10 mg. Riwayat Diabetes Mellitus disangkal.
Berdasarkan kriteria Hunt and Hess, termasuk klasifikasi derajat berapakah pasien ini…
a. Grade 1 asimptomatok nyeri kepala ringan, kaku kuduk ringan
b. Grade 2 : tidak ada deficit neurologis
c. Grade 3  somnolens
d. Grade 4 : stupor

168
TO UGM 17 FEBUARI 2022
e. Grade 5 : koma dalam
130. Lakilaki 48 tahun dibawa ke IGD dengan keluhan lemah kedua tungkai dialami
sejak 5 jam sebelum masuk RS. Keluhan dirasakan setelah pasien terjatuh dari
ketinggian 4 meter ketika hendak memperbaiki genteng rumahnya yang bocor. Pasien
terjatuh dengan posisi terduduk.
Bagaimanakah penatalaksanaan yang paling tepat pada pasien ini….
a. Pemberian metilprednisolon dosis 20 mg/kgBB dalam 1 jam pertama, dilanjutkan
dengan dosis 2,4 mg/kgBB/jam selama 23 jam berikutnya
b. Pemberian metilprednisolon dosis 20 mg/kgBB dalam 1 jam pertama, dilanjutkan
dengan dosis 2,4 mg/kgBB/jam selama 47 jam berikutnya
c. Pemberian metilprednisolon dosis 20 mg/kgBB dalam 1 jam pertama, dilanjutkan
dengan dosis 5,4 mg/kgBB/jam selama 12 jam berikutnya
d. Pemberian metilprednisolon dosis 30 mg/kgBB dalam 1 jam pertama, dilanjutkan
dengan dosis 5,4 mg/kgBB/jam selama 47 jam berikutnya
e. Pemberian metilprednisolon dosis 30 mg/kgBB dalam 1 jam pertama, dilanjutkan
dengan dosis 5,4 mg/kgBB/jam selama 23 jam berikutnya

131. Seorang pasien laki-laki 31 tahun mengalami kecelakaan sepeda motor 5 hari
yang lalu. Saat ini kesadaran pasien GCS E3M6V4. Pupil reaktif dan refleks kornea ada.
Withdrawal response simetris. Hasil pemeriksaan Computed Tomography (CT) scan
dinyatakan normal, dan tekanan intrakranial normal. D/Diffuse axonal injury
Apakah temuan yang paling mungkin bisa didapatkan pada pemeriksaan MRI kepala…..
a. Pelebaran Virchow robbin space
b. Chronic subdural hematom
c. Subdural hygroma
d. Tetap normal

169
TO UGM 17 FEBUARI 2022
e. Salt and papper appearance

132. Seorang wanita berusia 25 tahun, mengalami cedera kepala berat setelah
mengalami kecelakaan sepeda motor. Skor Glasgow Coma Scale E2M3V2, pupil reaktif,
refleks kornea positif, dan hasil CT scan kepala memberikan gambaran perdarahan di
area lobus temporal kiri.
Tatalaksana apakah yang paling dianjurkan…..
a. Analgetik
b. Eritropoietin
c. Progesterone
d. Analgetik Kortikosteroid dosis tinggi
e. Analgetik Pemantauan tekanan intracranial  headup , mannitol jika ada tanda2
peningkatan TIK

133. Seorang laki-laki 25 tahun datang ke poliklinik neurologi dengan keluhan


bengkak pada kelopak mata kanan, mata kanan juga terlihat menonjol dan merah, mata
kanan kabur dan semakin lama semakin berat yang diawali penglihatan ganda. Pada
pemeriksaan fisik didapatkan mata kanan berdenyut dan nyeri kepala. Parese nervus
okulomotorius dan abdusen kanan. Visus okuli dextra 0. Visus okuli sinistra normal.
Riwayat jatuh dari sepeda motor sebulan yang lalu.
Bagian apakah yang paling mungkin mengalami cedera pada pasien ini…..
a. Arteri serebri media
b. Arteri serebri anterior
c. Arteri karotis interna segmen clinoid
d. Arteri karotis interna segmen petrosus
e. Arteri karotis interna segmen cavernosus

134. Seorang wanita 50 tahun mengalami kecelakaan yang merusak wajahnya 5 hari
yang lalu. Pasien dirawat di ICU dengan kesadaran menurun, tekanan darah 140/90
mmHg, nadi 80kali/menit, suhu 36,7°C dan pernafasan ireguler namun tidak terintubasi
(2). Ketika dinilai kesadarannya dengan Full Outline of Unresponsive score didapatkan

170
TO UGM 17 FEBUARI 2022
kelopak mata pasien membuka dengan rangsangan nyeri(1), ada respon fleksi kedua
lengan dengan rangsangan nyeri tersebut(2), pupil kanan berdilatasi dan terfiksasi.
Berapakah nilai FOUR score pada pasien ini….(1 +2+3+2)
a. 5
b. 6
c. 7
d. 8
e. 9

171
TO UGM 17 FEBUARI 2022
135. Seorang laki-laki 65 tahun yang sebelumnya didiagnosis dengan strok iskemik
sedang dirawat di High Care Unit. Saat follow up hari ke dua dinilai Glasgow Coma
Scale E2M3V2, tekanan darah 150/95 mmHg, nadi 92 kali/menit, suhu 37°C, pasien
bernafas terengah-engah yang diselingi oleh periode henti nafas yang berulang (cluster
breathing).
Di manakah letak lesi yang paling mungkin ada pada pasien ini..
a. Diensefalon

172
TO UGM 17 FEBUARI 2022
b. Pons inferior
c. Pons superior (apneusis)
d. Mesensefalon
e. Medula oblongata

136. Seorang laki-laki 65 tahun yang sebelumnya didiagnosis dengan strok iskemik
sedang dirawat di Intensive Care Unit. Saat ini keadaannya memburuk dan dilakukan tes
kalorik dengan mengalirkan air dingin di salah satu telinga, cold opposite warm same :
nystagmus berlawanan dengan irigasi air dingin
Manakah pernyataan yang paling tepat mengenai prosedur pemeriksaan ini…..
a. Komponen aferen refleks ini adalah N. VIII dengan eferen N.III dan IV
b. Posisi kepala pasien saat pemeriksaan adalah elevasi 60°
c. Tindakan dilakukan pada telinga sisi lainnya dengan interval 15 menit dari telinga
sebelumnya
d. Hasil positif berupa nistagmus ke arah telinga yang diirigasi 
e. Hasil positif berupa deviasi konjugat lambat ke sisi telinga yang diirigasi (konjugat
sesisi dengan telinga yang diberi air dingin)

137. Seorang pasien wanita 23 tahun dikonsul karena dapat membuka mata secara

173
TO UGM 17 FEBUARI 2022
spontan namun tidak kontak dengan lingkungan sekitarnya. Hal ini dialami beberapa jam
sebelumnya. Tekanan darah 130/80 mmHg, nadi 110 kali/menit, pernafasan 22 kali/menit
dan suhu 37,8°C. Pasien tidak mengedipkan mata saat dilakukan refleks ancam, namun
masih ada respon optokinetik nistagmus, tidak ada kaku kuduk. Pasien menyeringai
disertai ekstremitas kaku dengan posisi tertentu. Dari hasil CT scan kepala dan
pemeriksaan laboratorium tidak ditemukan kelainan.
Keadaan apa yang paling mungkin dialami oleh pasien ini.
a. Katatonia
b. Akinetik mutism : ggg perilaku pd lobus frontal
c. Reaksi konversi : malingering mata tertutup,
d. Bangkitan absans : bengong
e. Lock in syndrome : susah pulih , lesi pada pons, pasien ICU

138. Seorang laki-laki 79 thn, dibawa ke klinik anda oleh anaknya. Dia mengatakan
ayahnya seperti orang bingung, walaupun periode bingung berfluktuatif. Dikatakan
ayahnya melihat didatangi oleh orang yang tidak Nampak. Pada pemeriksaan didapatkan
bradykinesia dan tremor saat istirahat pada kedua tangan. MMSE 10/30, ADL terganggu.
Apakah temuan khas pada pemeriksaan Histo PA sesuai kasus diatas..
a. Badan TAU
b. Lewy Body : sineklupati, MSA PDD, demensia Lewy Body
c. Alpha- synoclein
d. Presenilin -1
e. Presenilin -2
PSP, FTD alzeimer, CJB : TAUpati
139. Laki- laki 65 tahun telah didiagnosa oleh dokter Ahli saraf dengan Parkinson dan
telah diberi Levadopa selama 6 tahun. 1 bulan ini mengeluh ada Gerakan meliuk -liuk tak
terkendali, setelah 2-3 jam mengkomsumsi levodopa, leher kaku (-) D/Dyskinesia
(wearing ON)
Tata laksana yang tepat untuk pasien tersebut adalah…
a. Menngganti dengan Levadopa CR
b. Meningkatkan dosis Levadopa

174
TO UGM 17 FEBUARI 2022
c. Menurunkan dosis Levadopa
d. Menambah COMT inhibitor
e. Menambah MAO-B inhibitor

140. Laki-laki usia 60 tahun dibawa olh keluarganya ke IGD dengan keluhan jalan
melambat dan sering jatuh kebelakang sejak 1 tahun terakhir. Pada pemeriksaan fisik
didapatkan bradykinesia, pull test (+), square wave jerk eye movement (+). Diagnose PSP
Apakah gambaran MRI kepala spesifik yang diharapkan pada kasus ini …
a. Cross Bun Sign
b. Hilangnya swallow tail sign
c. Hummingbird appearance
d. Normopressure hydrocephalus
e. Hypointense pada midbrain

141. Seorang wanita umur 60 tahun, diantar oleh keluarganya ke poli saraf, karena
sejak 7 bulan terakhir sudah tidak bisa lagi melakukan aktivitas kesehariannya seperti
mandi, berpakaian, terkadang buang air besar di sembarang tempat. Awalnya ia sering
pelupa dan kesulitan dalam berkomunikasi, semakin hari semakin memberat. Pada
pemeriksaan fisis neurologis tidak ditemukan adanya deficit neurologis fokal. MMSE
10/30
Apakah diagnosis yang paling mungkin kasus tersebut di atas…
a. Mild cognitive impairment type amnestik multidomain
b. Demensia Alzheimer type late onset
c. Demensia Alzheimer type early onset (usia dibawah 65)
d. Demensia Fronto-temporal
e. Demensia Vaskular

142. Seorang laki-laki umur 76 tahun, diantar oleh keluarganya ke poli saraf, karena
sejak 9 bulan terakhir sudah tidak bisa lagi melakukan aktivitas kesehariannya seperti
mandi, berpakaian, terkadang buang air besar di sembarang tempat. Awalnya ia sering
pelupa dan kesulitan dalam berkomunikasi, semakin hari semakin memberat. Pada CT

175
TO UGM 17 FEBUARI 2022
scan kepala ditemukan adanya atropi otak regio temporo-parietal.
Pilihlah salah satu pernyatan yang benar tentang patogenesis penyakit yang diderita oleh
pasien tersebuit di atasa.
a. Pembentukan peptida β-amyloid -Aβ42 oleh enzim γ sekretase. (GAMMA)
b. Pembentukan peptida β-amyloid -Aβ42 oleh enzim α sekretase.
c. Pembentukan peptida β-amyloid -Aβ42 oleh enzim β sekretase.
d. Pembentukan peptida β-amyloid -Aβ42 oleh enzim MMP 9
e. Pembentukan peptida β-amyloid -Aβ42 oleh enzim MMP3

143. Laki-laki usia 52 tahun, diantar oleh keluarganya ke Poli Saraf dengan keluhan
mendadak tidak mampu menulis dan membaca apa yang telah dituliskannya. Pada
pemeriksaan fungsi kortikal luhur ditemukan adanya aleksia disertai agrafia.
Dimanakah letak lesi kasus tersebut….
a. Regio temporal hemisfer kiri
b. Regio oksipital hemisfer kiri
c. Regio parietal hemisfer kiri
d. Girus angularis hemisfer kiri : MCA post
e. Girus kalkarina hemisfer kiri

144. Laki-laki 40 tahun, menjalani tes genetik beberapa tahun lalu untuk kondisi
autosomal dominan, yang mengenai anggota keluarga dari ibunya selama beberapa
generasi. Dari hasil tes menunjukkan terdapat defek gen, dan sekarang dia percaya bahwa
dia menunjukkan gejala penyakit, termasuk gerakan aneh tidak bertujuan pada
ekstremitas dan menyebabkan aktivitas sehari-hari terganggu.
Bila pasian ini mendanatkan L-dopa, mana diantara berikut yang akan tercetus…..
a. Generalized Seizure
b. Partial Seizure
c. Intention tremor
d. Scanning speech
e. Writing and jerking movement of limb

176
TO UGM 17 FEBUARI 2022
145. Seorang perempuan berusia 60 tahun dibawa ke poliklinik neurologi dengan
keluhan perilaku yang tidak sesuai, gangguan fungsi social selama beberapa tahun,
disinhibisi , apati dan kehilangan simpati, keluhan disertai dengan gangguan memori dan
berbahasa. Terdapat Riwayat keluarga yang mengalami keluhan serupa
Apakah diagnosis yang mungkin pada kasus diatas….
a. Demensia semantik
b. Primary progressive aphasia
c. Dementia Lewy body
d. Dementia frontotemporal : domain ggg perilaku
e. Dementia Alzheimer

146. Seorang pria berusia 55 tahun, datang ke RS dengan keluhan gerakan berliku-liku
dan irregular pada tangan dan jari kanan, seperti gerakan bermain piano sejak 1 tahun ini
dan semakin bertambah dalam beberapa bulan belakangan ini. Gerakan gemetar dan
perlambatan dalam beijalan tidak dijumpai. Riwayat trauma kepala, hipertensi tidak
dijumpai. AKB-PKS (atetosis)/ putamen
Berdasarkan skenario kasus ini, daerah anatomi manakah yang mengalami gangguan
pada pasien tersebut….
a. Nukleus subthalamicus
b. Nukleus ventrolateral thalami
c. Korteks Serebri
d. Putamen
e. Globus Pallidus

147. Laki-laki, 72 tahun, lulusan SMA, datang dengan keluhan mudah lupa sejak 6
bulan lalu. Hasil pemeriksaan neuropsikologi MMSE 20, CDT 1, Hachinski score 4.
Aktivitas harian perlu dibantu. Demensia Alzeimer
Proses apakah yang terjadi pada otak pasien tersebut.....
a. Inklusi sitoplasmik pada substansia nigra
b. Multiple infark
c. Demielinisasi korteks serebri

177
TO UGM 17 FEBUARI 2022
d. Defisiensi vitamin Bl
e. Plak amyloid

148. Laki-laki 40 tahun, datang ke poliklinik Neurologi dengan keluhan gemetar pada
kedua tangan sejak 1 tahun yang lalu. Semula ringan pada tangan kanan dan tidak terlalu
mengganggu tetapi lama kelamaan semakin hebat pada keduanya dan bertambah jelas
pada saat mengangkat mangkok dan gelas. Gemetar dengan frekuensi 4-12 Hz serta
dijumpai tremor pada kepala, riwayat keluarga menderita keluhan yang sama (+).
Apakah diagnosis yang tepat pada kasus di atas…
a. Tremor Intensi
b. Tremor Istirahat
c. Tremor Esssensial
d. Task-specific tremor
e. Tremor Kinetik Sederhana

149. Wanita 42 tahun datang dengan demam subfebril selama 1 bulan, terdapat nyeri
kepala, fotofobia, dan kaku pada leher selama kurang lebih satu minggu, dan perubahan
status mental. Tekanan Lumbal pungsi 30 cm H2O. Hasil Ananlisis LCS terdapat 0
eritrosit, 55 leukosit dengan 70% limfosit (normal s.d 5 limfosit/uL), protein 60 mg/ dL
(normal s.d 45 mg/ dL), dan glukosa 50 mg/dL. Hasil pengecatan tinta india (+) pada
LCS. D/Meningitis kriptokokus
Apakah terapi yang paling tepat untuk kasus di atas.

a. fluconazole 200 mg per hari


b. Ampotecarin 2 mg/kgbb/hari
c. Vancomycin 0,5mg-2 gr/hari
d. Pirimetamin 200mg
e. Ceftriaxone 2x2gr

150. Seorang wanita 60 tahun dengan HIV yang didiagnosis sekitar 6 tahun yang lalu,
datang ke klinik untuk evaluasi nyeri kaki distal dan simetris dan parestesia yang

178
TO UGM 17 FEBUARI 2022
perlahan berkembang selama 12 bulan sebelumnya. Dia melaporkan bahwa dia memulai
terapi antiretroviral baru sekitar 2 bulan yang lalu ketika jumlah CD4-nya ditemukan
menurun.
Manakah dari berikut ini yang merupakan diagnosis yang paling mungkin.
a. Neuropati HIV
b. Neuropati terkait analog nukleosida
c. Poliradikulomielitis sitomegalovirus
d. Poliradikuloneuropati demielinasi inflamasi akut
e. Multipleks mononeuropati

151. Seorang wanita 79 tahun dengan diabetes, hipertensi, dan ulkus kaki kronis
mengalami sakit kepala dan demam. Pada pemeriksaan, dia memiliki leher kaku dan
kebingungan ringan. CT scan otak biasa-biasa saja. Pungsi lumbal dilakukan, dan CSF
menunjukkan 112 WBCs/μL (normal hingga 5 limfosit/μL) dengan neutrofil 81%, kadar
protein 60 mg/dL (normal hingga 45 mg/dL), dan glukosa 80 mg/dL, dengan glukosa
darah 198 mg/dL. CSF dikirim untuk kultur.
Regimen antibiotik empiris mana yang akan Anda mulai..
a. Vankomisin dan seftriakson
b. Vankomisin, seftriakson, dan ampisilin
c. Vankomisin, seftriakson, dan asiklovir
d. Ampisilin dan gentamisin
e. Penisilin dan gentamisin

152. Seorang laki-laki berusia 25 tahun terlibat dalam kecelakaan kendaraan bermotor
8 bulan yang lalu dan mengalami trauma kepala, membutuhkan beberapa operasi
kraniofasial. Ia juga memiliki riwayat sinusitis kronis. Dia sekarang datang dengan
demam dan perubahan status mental. MRI otaknya ditampilkan pada gambar

179
TO UGM 17 FEBUARI 2022
Apakah diagnosis yang paling mungkin untuk kasus tersebut.
a. Abses cerebri
b. Toxoplasma serebri
c. Tuberculoma serebri
d. Primary CNS lymphoma
e. Brain metastase

153. Laki-laki 57 tahun dibawa ke klinik oleh istrinya karena penurunan kognitif dan
berbagai gejala neurologis. Sekitar 4 bulan yang lalu, dia berhenti mengkhawatirkan
pekerjaannya dan mulai menunjukkan perilaku aneh, meninggalkan rumah tanpa mandi
atau menyikat gigi. Dia semakin menjadi apatis dan paranoid. Dia tidak bisa
memecahkan masalah matematika sederhana dan mengalami kesulitan membayar
tagihannya. Selama sebulan terakhir, ia mengalami gaya berjalan yang tidak stabil,
ataksia, dan sering jatuh. Istrinya juga melaporkan bahwa dia "menyentak." MRI
diperoleh dan ditunjukkan pada gambar.

Manakah dari berikut ini yang paling mungkin menjadi penyebab..


a. Leukoensefalopati multifokal progresif
b. CJD
c. Penyakit Alzheimer
d. Demensia AIDS
e. FTD

180
TO UGM 17 FEBUARI 2022
154. Seorang anak laki-laki berusia 5 tahun di bawa oleh kedua orang tuanya dengan
keluhan kejang di sekujur tubuh sejak 1 jam yang lalu. Setelah diberikan antipiretik,
kejang berhenti. Riwayat 3 hari sebelumnya ia menderita demam tinggi. Pada
pemeriksan fisik didapatkan temperature 400C, GCS E2M2V1, pemeriksaan liquor
serebrospinalis ditemukan cairan jernih, glukosa normal, protein positif, ringan.
Apakah kemungkinan diagnosis.
a. Meningitis viral
b. Meningitis TB
c. Ensefalitis
d. Tumor otak
e. Epilepsi

155. Seorang wanita 17 tahun, datang dibawa orang tuanya ke rumah sakit dengan
keluhan demam, nyeri kepala dan muntah. Penglihatan mata kanan menjadi kabur.
Dalam 1 minggu, visus pasien saat ini 1/300. Hasil laboratorium dijumpai lekositosis:
14.000/mm3. Gambaran EEG menunjukkan voltase tinggi dengan aktivitas lambat di
daerah temporal kanan. Hasil MRI otak, dapat dilihat gambar dibawah ini. Untuk gambar
A: Axial FLAIR MRI, gambar B: Axial T1-weighted post-contrast MRI, gambar C: Axial
diffusion-weighted MRI

Apakah diagnosis yang paling tepat pada pasien ini…


a. Toxoplasmosis Encephalitis
b. Primary CNS Lymphoma
c. Abses Serebri
d. Tuberculoma cerebri
e. Glioblastoma multifirm

156. Laki-laki usia 50 tahun diantar oleh keluarga ke Instalasi Gawat Darurat dengan
keluhan berupa flu, anoreksia dan rasa tidak enak di kerongkongan. Dua hari yang lalu
penderita ini digigit anjing di sekitar lutut kanan. Selain itu pada penderita ini dijumpai
juga adanya hiperhidrosis, hipersalivasi dan hiperlakrimasi.
Tatalaksana manakah yang paling tepat…
a. Luka dijahit tertutup

181
TO UGM 17 FEBUARI 2022
b. Pemberian injeksi steroid
c. Pemberian VAR dan SAR
d. Luka gigitan dicuci dengan air mengalir
e. Hanya diberikan SAR serum homolog dengan dosis pemberian adalah 20 IU/ kgBB

157. Seorang anak perempuan usia 10 tahun dibawa ke UGD RS karena mengalami
kejang pada seluruh tubuh sejak 2 jam yang lalu. Sebelumnya pasien demam tinggi dan
sakit kepala, batuk serta muntah. Sewaktu kecil pasien tidak pernah kejang, tidak ada
riwayat trauma sebelumnya. Pada pemeriksaan fisik didapatkan kaku kuduk (+).
Apakah pemeriksaan penunjang yang paling tepat untuk membantu menegakkan
diagnosis…
a. Pemeriksaan darah arteri
b. Pemeriksaan LCS
c. Elektroenchphalografi
d. Pemeriksaan sputum
e. CT Scan kepala

158. Perempuan 55 tahun, datang ke poliklinik Neurologi dengan keluhan nyeri


kepala sejak 1 tahun yang lalu dan memberat 1 minggu terakhir. Pasien juga mulai
sering lupa dengan pekerjaannya sebagai ibu rumah tangga dan terkadang marah-
marah dengan alasan tidak jelas. Riwayat hipertensi (-), diabetes melitus (-), riwayat KB
suntik (+). Pada pemeriksaan fisik didapatkan, tekanan darah 130/80mmHg, nadi
80x/menit, suhu: 36,6, skor MMSE 22. Hasil CT scan kepala kontras menunjukkan lesi
hiperdens yang homogen dan berbentuk bulat ukuran 2cm, melekat pada batas
duramater
Apakah diagnosis yang tepat pada kasus di atas..
a. Meningioma di ventrikel lateral
b. Meningioma parasagittal anterior
c. Meningioma di foramen magnum
d. Meningioma Falks posterior
e. Meningioma Cerebellopontine angle

159. Perempuan 55 tahun, datang ke poliklinik Neurologi dengan keluhan nyeri


kepala sejak 1 tahun yang lalu dan memberat 1 minggu terakhir. Pasien juga mulai
sering lupa dengan pekerjaannya sebagai ibu rumah tangga dan terkadang marah-
marah dengan alasan tidak jelas. Riwayat hipertensi (-), diabetes melitus (-), riwayat KB
suntik (+). Pada pemeriksaan fisik didapatkan, tekanan darah 130/80mmHg, nadi
80x/menit, suhu: 36,6, skor MMSE 22. Hasil CT scan kepala kontras menunjukkan lesi
hiperdens yang homogen dan berbentuk bulat ukuran 2cm, melekat pada batas
duramater
Patofisiologi yang sesuai pada kasus di atas…

182
TO UGM 17 FEBUARI 2022
a. Tumor memiliki vaskularisasi yang tinggi dengan mengekspresikan Brain Derived
Nerve Growth Factor (BDNF)
b. Mutasi gen neurofibromatosis 1 (NF1) yang terletak di kromosom 22
c. Inaktivasi protein Moesin-ezrin-radixin-like-protein (MERLIN)
d. Ditemukan loss of homozygot (LOH) dari gen NF1 di kromosom 22
e. Kelainan kromosom tersering pada tumor ini adalah trisomi kromosom

160. Laki-laki 59 tahun, datang ke poliklinik Neurologi dengan keluhan pusing, dia
merasa seperti bergoyang pada saat jalan dan duduk. Keluhan ini sudah dirasakan sejak
6 bulan yang lalu dan memberat 3 hari terakhir. Pasien juga mengeluhkan bila hendak
melakukan aktivitas seperti menulis atau minum secangkir kopi tangan kanan makin
bergetar. Riwayat hipertensi (+) sejak 2 bulan lalu dan berobat teratur, riwayat DM (-),
riwayat kolesterol tinggi (-). Pada pemeriksaan fisik didapatkan, tekanan darah
120/80mmHg, nadi 76x/menit, suhu: 36,5, tonus otot ekstremitas atas dan bawah
kanan menurun. Hasil CT scan kepala kontras menunjukkan lesi bulat hiperdens dengan
batas tegas dan berbentuk kistik & kalsifikasi pada area periventrikel IV
Apakah diagnose yang tepat untuk kasus di atas..
a. Meningioma
b. Ependimoma
c. Tumor Pineal
d. Tumor metastase
e. PCNSL

161. Laki-laki 59 tahun, datang ke poliklinik Neurologi dengan keluhan pusing, dia
merasa seperti bergoyang pada saat jalan dan duduk. Keluhan ini sudah dirasakan sejak
6 bulan yang lalu dan memberat 3 hari terakhir. Pasien juga mengeluhkan bila hendak
melakukan aktivitas seperti menulis atau minum secangkir kopi tangan kanan makin
bergetar. Riwayat hipertensi (+) sejak 2 bulan lalu dan berobat teratur, riwayat DM (-),
riwayat kolesterol tinggi (-). Pada pemeriksaan fisik didapatkan, tekanan darah
120/80mmHg, nadi 76x/menit, suhu: 36,5, tonus otot ekstremitas atas dan bawah
kanan menurun. Hasil CT scan kepala kontras menunjukkan lesi bulat hiperdens dengan
batas tegas dan berbentuk kistik & kalsifikasi pada area periventrikel IV
Apakah penatatalaksanaan yang tepat pada kasus diatas…..
a. Reseksi total
b. Kemoterapi
c. Reseksi total dilanjutkan kemoterapi
d. Reseksi Parsial dilanjutkan kemoterapi
e. Reseksi total dilanjutkan radioterapi

162. Laki-laki 35 tahun, datang ke poliklinik Neurologi dengan keluhan kejang sejak 1
hari yang lalu, kejang sebanyak 3x dengan lama kejang 5 menit. Kejang berupa kaku

183
TO UGM 17 FEBUARI 2022
pada tangan sebelah kanan, pasien sadar saat kejang. Pasien juga mengeluhkan nyeri
kepala yang dialami sejak 5 bulan yang lalu dan memberat 1 bulan terakhir. Riwayat
trauma (-), demam (-). Pada pemeriksaan fisik didapatkan tekanan darah 110/70mmHg,
nadi 78x/menit, suhu: 36,3, terdapat kelemahan pada ekstremitas kiri. Hasil MRI kepala
menunjukkan lesi hipointens pada T1 dan hiperintens pada T2, tidak ada penyangatan
paska pemberian kontras. Hasil histologi PA menunjukkan sel dengan ukuran kecil,
bentuk bulat, gelap dengan nucleus kompak dan sitoplasma sedikit eosinofilik, tampak
honeycomb appearance atau fried egg appearance.
Apakah diagnosis yang tepat untuk kasus di atas…
a. Astrositoma
b. Glioblastoma
c. Oligodendroglioma
d. Tumor Ependimal
e. Tumor Pleksus koroid

163. Wanita 50 tahun datang ke poliklinik neurologi dengan keluhan penurunan


pendengaran telinga kanan sejak 2 bulan yang lalu. 2 minggu terakhir ini terdengar suara
gemuruh dari telinga yang sama. Pemeriksaan Rinne positif pada kedua telinga, weber
lateralisasi ke telinga kiri. Hasil MRI menunjukkan gambaran “trumpeted internal
acusticus”
Diagnosis yang tepat untuk kasus di atas.
a. Schwannoma Vestibular
b. Tumor selubung saraf tepi maligna
c. Trombosis pada Anterior inferior cerebellar artery (AICA)
d. Meningioma di cerebellum
e. Neurofibromatosis tipe 2

164. Seorang perempuan usia 18 tahun, datang ke IRD dengan keluhan nyeri kepala
berat yang dirasakan sejak 4 bulan yang lalu, nyeri kepala memberat 4 hari terakhir.
Pasien juga mengeluh tidak haid selama 5 bulan yang lalu hingga sekarang, riwayat haid
3 bulan sebelum keluhan ini selalu teratur, pasien sebelumnya pernah periksa dan telah
dilakukan MRI kepala dengan hasil, terdapat massa di suprasellar.
Apakah diagnosis yang tepat pada kasus di atas…
a. Low grade astrocytoma
b. High grade astrocytoma
c. Adenoma Hipofisis
d. Tumor Pleksus koroid
e. Glioblastoma multiform

165. Seorang perempuan usia 18 tahun, datang ke IRD dengan keluhan nyeri kepala
berat yang dirasakan sejak 4 bulan yang lalu, nyeri kepala memberat 4 hari terakhir.
Pasien juga mengeluh tidak haid selama 5 bulan yang lalu hingga sekarang, riwayat haid

184
TO UGM 17 FEBUARI 2022
3 bulan sebelum keluhan ini selalu teratur, pasien sebelumnya pernah periksa dan telah
dilakukan MRI kepala dengan hasil, terdapat massa di suprasellar.
Pada kasus diatas, pemeriksaan hormone apakah yang dianjurkan.
a. Prolaktin
b. Somatotropin
c. Adenokortikotropin
d. Tirotropin
e. Gonadotropin

166. Perempuan usia 60 tahun datang ke IRD dengan keluhan pusing sejak 2 bulan
yang lalu, pusing dirasakan seperti sedang berdiri di atas kapal, memberat 7 hari terakhir
ini. Pasien kesulitan dalam berjalan, sering mau jatuh ke sisi kanan dan tangan sebelah
kanan gemetar, gemetar memberat saat melakukan aktivitas seperti minum atau makan.
Pasien merupakan penderita Ca Mammae yang sudah didiagnosa 1 tahun yang lalu.
Pemeriksaan laboratorium dalam batas normal.
Pemeriksaan penunjang selanjutnya yang dianjurkan pada pasien ini adalah….
a. CT Scan Kepala non kontras
b. CT Scan Kepala dengan kontras
c. MRI kepala non kontras
d. MRI kepala dengan kontras
e. EEG

167. Seorang laki-laki berusia 37 tahun, datang ke poli saraf karena mengalami
kelopak mata kanan yang tertutup disertai dengan ukuran pupil yang mengecil pada
kedua matanya. Pasien juga merasakan hanya wajah sisi kanannya yang tidak
berkeringat. Tidak ditemukan deficit neurologis laiinnya.
Apakah obat yang dapat digunakan untuk menegakkan kemungkinan diagnosis kasus
tersebut diatas…..
a. Hydroxyamphetamin 1%
b. Pilokarpin 0,1%
c. Pilokarpin 1%
d. Mecholyl 5%
e. Cocaine 4%

168. Seorang laki-laki, 47 tahun datang ke poliklinik saraf dengan keluhan gangguan
pergerakan bola mata. Pada pemeriksaan fisik ditemukan hasil sebagai berikut :

185
TO UGM 17 FEBUARI 2022
Dimanakah letak topis pada kasus di atas…..

a. Frontal Gaze Center


b. Medula Oblongata
c. Mesensefalon
d. Cerebellum
e. Pons

169. Seorang laki-laki, 47 tahun datang ke poliklinik saraf dengan keluhan gangguan
pergerakan bola mata. Pada pemeriksaan fisik ditemukan hasil sebagai berikut :

Apakah diagnosis pada kasus di atas…


a. Internuclear Opthalmoplegia Kanan
b. Internuclear Opthalmoplegia Kiri
c. One-and-a-half Syndrome
d. Parinaud Syndrome
e. Foville Syndrome

170. Seorang perempuan berusia 43 tahun datang ke poliklinik saraf dengan keluhan
pusing berputar hilang-timbul yang dirasakan semakin memberat sejak 1 tahun terakhir.
Keluhan disertai dengan penurunan pendengaran progresif dan telinga berdenging pada
sisi kanan. Demam tidak ada. Pada pemeriksaan fisik ditemukan nistagmus horizontal,
tuli sensorineural nada tinggi, hipestesi wajah sisi kanan, dan intention tremor sisi kanan.
Apakah kemungkinan diagnosis pasien tersebut..
a. Benign Paroxysmal Positional Vertigo
b. Tumor Cerebellopontine Angle
c. Neuritis Vestibularis
d. Meniere’s Disease
e. Labirinthitis

171. Seorang perempuan berusia 43 tahun datang ke poliklinik saraf dengan keluhan
pusing berputar hilang-timbul yang dirasakan semakin memberat sejak 1 tahun terakhir.
Keluhan disertai dengan penurunan pendengaran progresif dan telinga berdenging pada
sisi kanan. Demam tidak ada. Pada pemeriksaan fisik ditemukan nistagmus horizontal,
tuli sensorineural nada tinggi, hipestesi wajah sisi kanan, dan intention tremor sisi kanan.
Apakah kemungkinan yang paling tepat dari patogenesis etiologi kasus di atas..
a. Sekuestrasi sel epitelial pada kapsula otik

186
TO UGM 17 FEBUARI 2022
b. Defek pada gen supressor tumor NF2 (MENINGIOMA DAN SCHWANOMA)
c. Gangguan penutupan neural tube
d. Misdiferensiasi meningens
e. Malformasi kongenital

172. Seorang perempuan berusia 43 tahun datang ke poliklinik saraf dengan keluhan
pusing berputar hilang-timbul yang dirasakan semakin memberat sejak 1 tahun terakhir.
Keluhan disertai dengan penurunan pendengaran progresif dan telinga berdenging pada
sisi kanan. Demam tidak ada. Pada pemeriksaan fisik ditemukan nistagmus horizontal,
tuli sensorineural nada tinggi, hipestesi wajah sisi kanan, dan intention tremor sisi kanan.
Apakah penyebab yang paling sering ditemukan pada diagnosis kasus di atas..
a. Meningioma Cerebellopontine Angle
b. Vestibular Schwannoma
c. Kista Epidermoid
d. Kista Arakhnoid
e. Lipoma

173. Seorang perempuan berusia 43 tahun datang ke poliklinik saraf dengan keluhan
pusing berputar hilang-timbul yang dirasakan semakin memberat sejak 1 tahun terakhir.
Keluhan disertai dengan penurunan pendengaran progresif dan telinga berdenging pada
sisi kanan. Demam tidak ada. Pada pemeriksaan fisik ditemukan nistagmus horizontal,
tuli sensorineural nada tinggi, hipestesi wajah sisi kanan, dan intention tremor sisi kanan.
Pada pemeriksaan MRI kepala menunjukkan gambaran hiperintens pada sekuens T2WI
yang meluas ke kanalis auditorik eksterna dengan ukuran 4 cm.
Pada stadium berapakah diagnosis kasus di atas berdasarkan klasifikasi Koos….
a. 1
b. 2
c. 3
d. 4
e. 5

174. Seorang perempuan berusia 35 tahun datang ke poliklinik saraf dengan keluhan
merasa melayang dan tidak seimbang, yang dirasakan terutama bila pasien berada di
tempat keramaian. Sebelumnya pasien tidak pernah merasakan keluhan seperti ini.
Perasaan melayang tidak berhubungan dengan posisi tubuh maupun kepala. Riwayat
diabetes mellitus disangkal.
Apakah kemungkinan diagnosis kasus tersebut di atas….
a. Benign Paroxysmal Positional Vertigo
b. Cervicogenic Dizziness
c. Vertigo Psikogenik
d. Vertigo Visual
e. Presyncope

187
TO UGM 17 FEBUARI 2022
175. Seorang perempuan berusia 35 tahun datang ke poliklinik saraf dengan keluhan
merasa melayang dan tidak seimbang, yang dirasakan terutama bila pasien berada di
tempat keramaian. Sebelumnya pasien tidak pernah merasakan keluhan seperti ini.
Perasaan melayang tidak berhubungan dengan posisi tubuh maupun kepala. Riwayat
diabetes mellitus disangkal.
Apakah pemeriksaan yang dapat dilakukan untuk menunjang diagnosis kasus di atas.
a. Situational Characteristics Questionnaire
b. Hamilton Depression Rating Scale
c. Dix-hallpike Maneuver
d. Head-up Tilt-Table
e. Neck Torsion Test

176. Seorang perempuan berusia 35 tahun datang ke poliklinik saraf dengan keluhan
merasa melayang dan tidak seimbang, yang dirasakan terutama bila pasien berada di
tempat keramaian. Sebelumnya pasien tidak pernah merasakan keluhan seperti ini.
Perasaan melayang tidak berhubungan dengan posisi tubuh maupun kepala. Riwayat
diabetes mellitus disangkal.
Apakah tatalaksana yang tepat untuk diagnosis kasus di atas…..
a. Pemberian NSAID dan Muscle Relaxant
b. Pemberian Antidepresan Trisiklik
c. Pemberian Vestibulosupressan
d. Pemberian Stoking Elastik
e. Latihan Virtual Reality

177. Seorang pria 50 tahun datang ke poli saraf dengan keluhan lemah kedua
ekstremitas bawah sejak 2 minggu lalu. Sebelumnya pasien mengeluh nyeri pada
punggung dan diikuti dengan sedikit demam dan kekauan pada daerah nuchal. Kemudian
3 hari kemudian kelemahan muncul.
Hasil MRI ditunjukkan seperti gambar dibawah ini

Apa Diagnosa pada pasien ini.


a. Abses Spinal Epidural
b. Sarcoid Myelitis

188
TO UGM 17 FEBUARI 2022
c. Viral Myelitis
d. Vertebral Bacterial Osteomyelitis
e. Tuberculous Spinal Osteomyelitis

178. Seorang pria 50 tahun datang ke poli saraf dengan keluhan lemah kedua
ekstremitas bawah sejak 2 minggu lalu. Sebelumnya pasien mengeluh nyeri pada
punggung dan diikuti dengan sedikit demam dan kekauan pada daerah nuchal. Kemudian
3 hari kemudian kelemahan muncul.
Hasil MRI ditunjukkan seperti gambar dibawah ini

Etiologi bakteri yang menjadi penyebab penyakit tersebut adalah…


a. Eschericia Coli
b. Streptococcus Mutans
c. Staphylococcus Aureus
d. Bacillus Subtilis
e. Clostridium Perfringens

179. Seorang pria 50 tahun datang ke Poli saraf dengan sering mengalami kejang sejak
1 bulan lalu. Tipe kejang tersentak sentak tidak beraturan. Pasien sering mengeluh nyeri
kepala sejak 1 tahun terakhir. Pasien merupakan seorang peternak babi dan merawatnya
di sebelah rumahnya.
Hasil Ct scan kepala non kontras ditunjukkan seperti gambar dibawah ini

189
TO UGM 17 FEBUARI 2022
Apa diagnose dari kasus diatas…..
a. Tuberkuloma
b. Abses serebral
c. Toxoplasmosis serebral
d. Metastase tumor kepala
e. Neurosistiserkosis

180. Seorang anak laki laki berusia 2 tahun dibawa orang tuanya ke dokter spesialis
saraf dengan keterlambatan perkembangan dan kejang. Anak baru bisa tengkurap usia 1
tahun, tapi hingga sekarang belum dapat duduk. Kejang mulai sering terjadi sejak 3 bulan
terakhir. Riwayat kehamilan ibu normal. Riwayat persalinan per vaginam, dengan lilitan
tali pusat. Bayi tidak langsung menangis dan sempat biru. Dari pemeriksaan fisik
ditemukan anak belum dapat berbicara, tetraparese spastik, scissor sign (+), babinski
positif bilteral
Apakah kemungkinan diagnosis pasien tersebut di atas.
a. Cerebral Palsy
b. Retardasi Mental
c. Spinal Muscular Atrophy tipe II
d. Spinal Muscular Atrophy tipe III
e. Spinal Muscular Atrophy tipe IV

181. Seorang anak laki laki usia 1 tahun dibawa orang tuanya ke dokter saraf dengan
keterlambatan perkembangan dan kejang. Jika diberi susu anak sering tersedak, kejang
sering terjadi 3 bulan terakhir dengan kepala menoleh ke kiri dan kaku keempat anggota
gerak. Dari pemeriksaan fisik ditemukan mikrosefal, tetraparese spastik, scissor sign (+).
babinsky positif bilateral

190
TO UGM 17 FEBUARI 2022
Pemeriksaan penunjang yang paling tepat untuk menilai kerusakan otak pada pasien ini
adalah…
a. CT scan kepala non kontras
b. CT Scan kepala dengan kontras
c. MRI kepala
d. EEG
e. BERA

182. Seorang gadis 10 tahun datang ke IGD dengan keluhan nyeri kepala, mual, dan
muntah yang semakin memburuk selama 2 minggu. Pasien juga mengalami mati rasa
pada wajah dan lengan diikuti bicara yang tidak jelas. Riwayat kelemahan atau kejang
disangkal. Dua tahun sebelumnya dia menjalani pengangkatan katarak kongenital kanan.
CT kepala menunjukkan massa hiperdens di regio frontoparietal kiri dengan hiperostosis
terkait dari calvaria yang berdekatan dan pergeseran garis tengah 2 mm untuk
meningioma.
Diagnosis pada pasien tersebut adalah.MENINGIOMA
a. Neurofibromatosis tipe 2
b. Neurofibromatosis tipe 1
c. Tuberous Sclerosis
d. Sindrom Noonan
e. Sturge-Weber syndrome

183. Seorang wanita usia 40 tahun datang ke poliklinik saraf dengan keluhan utama
pandangan dobel. Keluhan ini dirasakan sejak 3 bulan yang lalu, hilang timbul, terutama
sore hari dan diperberat dengan aktivitas fisik. Dari pemeriksaan neurologi didapatkan
kelemahan sedang pada keempat anggota gerak dan gangguan ringan pada proses
menelan. Tes Wartenberg positif. Pemeriksaan Repetitive Nerve Stimulation
menunjukkan decrement gelombang 1 dan 4 lebih dari 10%. Setelah diberikan
pengobatan dengan pyridostigmin dosis maksimal, hasilnya tidak ada perbaikan.
Terapi apa yang perlu ditambahkan untuk kasus ini..
a. Obat imunosupresan
b. Kortikosteroid
c. Timektomi
d. Imunoglobulin
e. Kombinasi kortikosteroid dan imunosupresan

Piridostigmin, kortikosteroid, imunosupresan, timektomi

184. Seorang laki-laki 25 tahun datang ke Poliklinik Saraf dengan keluhan kelemahan
saat ekstensi pergelangan tangan dan jari-jari. Pada pemeriksaan fisik didapatkan
kelemahan m. extensor carpi ulnaris dan extensor digitorum comunis. Pemeriksaan
sensorik dalam batas normal.

191
TO UGM 17 FEBUARI 2022
Di manakah kemungkinan letak lesi pada kasus tersebut..
a. Spiral Groove
b. N. Interosseus Posterior
c. Aksila
d. Elbow
e. Pleksus brachialis

185. Seorang laki-laki 20 tahun datang ke Poliklinik Saraf dengan keluhan kelemahan
telapak kaki kiri sulit diangkat sejak 3 minggu yang lalu setelah terpeleset. Dari status
neurologis didapatkan kelemahan M. Tibialis Anterior, M. Peroneus Longus, dan M.
Ekstensor Digitorum Brevis sinistra, hipestesi di daerah lateral tungkai kiri bawah dan
dorsum pedis sinistra. CMAP N. Tibialis abnormal.
Apakah hasil pemeriksaan NCV – EMG yang diharapkan.
a. CMAP N. Peroneus Profundus dan Superfisialis abnormal
b. CMAP N. Tibialis abnormal
c. SNAP N. Suralis abnormal
d. EMG: terdapat aktivitas spontan pada M. Biceps Femoris
e. EMG : terdapat aktivitas spontan pada M. Gastrocnemius

186. Laki-laki 44 tahun datang ke IGD dengan keluhan mata kiri jatuh terutama sore
setelah beraktivitas. Keluhan ini membaik bila istirahat. Pada pemeriksaan ditemukan
ptosis OS, kekuatan motorik ekstremitas atas dan bawah dalam batas normal.
Dalam klasifikasi klinis berdasarkan Myastenia Gravis Foundation of America (MGFA),
kelas berapa pasien ini.
a. I
b. IIIa
c. IIb
d. IIIa
e. IIIb

187. Seorang laki-laki 56 tahun datang ke Poli Saraf dengan diagnosis myasthenia
gravis. Pada pemeriksaan fisik didapatkan ptosis dan diplopia segera, bicara sering
sengau, lelah mengunyah makanan lunak, jarang tersedak, terengah saat aktivitas.
Pada pemeriksaan penutupan mata didapatkan mudah dibuka, tidak ada kelemahan fleksi
atau ekstensi leher, abduksi bahu, dan fleksi pinggul.
Berapa skor MG composite dalam kasus ini..
a. 15
b. 17
c. 19
d. 21
e. 23

192
TO UGM 17 FEBUARI 2022
188. Seorang perempuan 40 tahun datang ke Poliklinik Saraf dengan keluhan nyeri
seperti ditusuk-tusuk jarum pada jari 4 dan 5 tangan kanan(ulnaris). Keluhan ini
seringkali terasa bila bangun pagi. Terdapat kelemahan tangan kanan sehingga pasien
sulit melebarkan jari-jari tangan dan menjepit dengan jempol. Terdapat gangguan
sensibilitas pada sisi palmar jari 4 dan 5. Pemeriksaan KHS: n. Ulnaris cutaneus normal.
Apakah diagnosis dari pasien tersebut….
a. Cubital fe
b. Carpal tunnel syndrome
c. Ulnar tunnel syndrome
d. Guyon canal syndrome
e. Saturday night palsy

189. Perempuan 20 tahun datang dengan sesak nafas hebat sejak 2 jam SMRS. Lima
hari sebelumnya pasien mengalami batuk pilek dan memperoleh antibiotik azitromisin
dari dokter. Riwayat alergi positif. Pasien telah terdiagnosis miastenia gravis. Pada
pemeriksaan fisik diperoleh counting test 10, disfagia dan kesan kelemahan ke-empat
ekstremitas. Pasien direncanakan dirawat di ICU dengan ventilator mekanik.
Apakah tatalaksana medikamentosa yang paling tepat diberikan pada pasien ini…
a. Piridostigmin
b. IV Methylprednisolon
c. IVIG
d. Siklofosfamid
e. Neostigmin

190. Seorang perempuan berusia 56 tahun datang ke poliklinik saraf dengan keluhan
kelemahan keempat anggota gerak sejak 2 minggu yang lalu. Kelemahan ini berlangsung
perlahan-lahan diawali rasa kesemutan pada keempat ekstremitas bagian distal kemudian
diikuti kelemahan sampai otot proksimal. Otot-otot bulbar dalam batas normal.
Pemeriksaan KHS menunjukkan multifokal blok konduksi, perlambatan kecepatan hantar
saraf, dan penurunan amplitudo CMAP yang minimal. Pemeriksaan CSF menunjukkan
peningkatan protein dengan jumlah sel yang normal.
Apakah kemungkinan diagnosis kasus di atas..
a. AIDP
b. AMSAN
c. CIDP
d. Miller-Fisher sindrome
e. AMAN

191. Laki-laki 31 tahun datang ke poliklinik saraf dengan keluhan kelemahan tungkai
kiri. Pada pemeriksaan fisik, pasien mengalami kelemahan saat ekstensi lutut kiri. Pasien
masih dapat melakukan fleksi lutut kiri, aduksi dan abduksi paha kiri. Refleks patela kiri
negatif.
Apakah diagnosis yang tepat pada pasien ini…

193
TO UGM 17 FEBUARI 2022
a. Radikulopati L3 - L4
b. Neuropati obturator
c. Neuropati femoral
d. Neuropati skiatika
e. Pleksopati lumbal

192. Seorang wanita usia 49 tahun mengalami nyeri leher yang dirasakan sampai
lengan bawah kanan bagian medial. Dari pemeriksaan fisik didapatkan kelemahan otot-
otot instrinsik tangan, refleks ticeps menurun. Dari hasil pemeriksaan MRI kemungkinan
terdapat penekanan diskus ke radiks medula spinalis.
Segmen radikulopati manakah yang paling tepat menggambarkan klinis pasien
tersebut…..
a. C3 – 4
b. C4 – 5
c. C5 – 6
d. C6 – 7
e. C7 - Th1

193. Seorang wanita 42 tahun dirawat di RS dengan diagnosis GBS. Saat penilaian
awal, didapatkan GBS disability score 2. Saat dirawat inap, pasien mengalami
perburukan gejala klinis.
Terapi apa yang harus dilakukan untuk kasus ini…..
a. IVIG 0,4 mg/kgBB selama 3 hari
b. IVIG 0,5 mg/kgBB selama 3 hari
c. IVIG 0,5 mg/kgBB selama 5 hari
d. Plasmafaresis 100-150 ml/kgBB/kali selama 2 minggu
e. Plasmafaresis 200-250 ml/kgBB/kali selama 2 minggu

194. Seorang wanita 30 tahun datang ke IGD dengan kelemahan pada keempat
ekstremitas predominan proksimal sejak 2,5 bulan yang lalu. Kelemahan ini dirasakan
semakin lama semakin progresif. Didapatkan defisit sensorik pada bagian distal. Pasien
memiliki kontraindikasi terhadap pemberian steroid.
Terapi apa yang diperlukan untuk kasus ini…
a. Plasmafaresis
b. IVMP
c. IVIG
d. Siklofosfamid
e. Metilprednisolon

194
TO UGM 17 FEBUARI 2022
REMED JULI 21
1. Perempuan, 35 tahun dirujuk dengan dugaan meningoencefalitis. Pada hasil pemeriksaan CT
Scan kepala didapatkan gambaran edema cerebri dan pasien diberikan injeksi dexametasone.
Mekanisme manakah yang tepat menjelaskan cara kerja obat tersebut?
a. Mengikat dan mematikan bakteri
b. Menyerap cairan di ekstraselluler
c. Memperbaiki membransel
d. Memperbaiki permeabilitas kapiler
e. Mencegah inflamasi
2. 30. Pasien laki-laki 40 tahun dikonsulkan oleh ICU dengan penurunan kesadaran mendadak 1
jam pasca CABG. Pada pemeriksaan fisik didapatkan tekanan darah 170/90 mmHg. Pemeriksaan
neurologis didapatkan GCS E1M1VETT, pupil bulat anisokor dan refleks cahaya negatif, doll’s eye
movement negatif, refleks kornea negatif dan hemiparese dekstra. Pada CT scan kepala tampak
infark teritorial MCA kiri. Pemeriksaan laboratorium GDS 150 g/dl, elektrolit N/K/Cl 120/3.0/1.2
meq/L, Hb 12 g/dl. Apakah prosedur selanjutnya yang paling tepat ?
A. Memberikanmanitol
B. Mendiagnosis mati batang otak (MBO)
C. Melakukan ekstubasi

D. Melakukan tes apneu

E. Melakukan tes kalori

3. Seorang wanita 45 tahun datang ke poli saraf membawa hasil CT scan kepala tanpa kontras
dengan hasil normal. Keluhan wanita tersebut adalah sering mengalami pusing berputar yang
hilang timbul disertai telinga berdenging dengan durasi sekitar 1 menit. Keluhan dirasakan saat
pasien menggerakkan kepalanya. Tidak ada mual muntah. Dari hasil pemeriksaan neurologi tid
4. ak ada kelainan, tidak ada riwayat penyakit serius. Terapi utama pada pasien tersebut adalah :
a. Karbamazepin 200-600 mg/hari

b. Betahistin mesilat 48 mg/hari


c. Flunarizin 10 mg/hari
d. Skopolamin 0.6 mg 3 kali/hari

e. Diazepam 10 mg/hari

5. Bayi laki-laki usia 2.5 bulan mengalami kejang berupa spasme tonik 10 detik, frekuensi serangan
10-300 kali dalam 24 jam. Gambaran pemeriksaan EEG yang mungkin didapatkan pada bayi
tersebut adalah :
A. Spike and double spike wave complex : lena
B. Slow spike wave complex : lennox

195
TO UGM 17 FEBUARI 2022
C. Hypssarythmia : west
D. Burst suppression asimetris
E. Gambaran EEG normal

6. Seorang anak 10 tahun sering mengalami jeritan saat tidur, diikuti dengan ketakutan yang
ekstrim, menangis dan sangat mengacaukan, namun pasien tidak merasa bermimpi. Keadaan
apakah yang mungkin terjadi pada pasien tersebut :
A. Confusional arousal
B. Somnabolism
C. Sleep terrors
D. Varian parasomnia non REM
E. Sleep walking

7. Seorang laki-laki 27 tahun datang dengan keluhan rasa tidak nyaman pada anggota gerak bawah
yang muncul ketika pasien duduk, berbaring/diam, memburuk pada malam hari. Pasien tersebut
mengalami gerakan periodik selama tidur. Gerakan pada tungkai dapat mengurangi rasa tidak
nyaman sebagian atau seluruhnya. Disebut apakah gangguan pada pasien tersebut :
A. Insomnia primer
B. Cataplexy
C. Sleep paralysis
D. Narkolepsi
E. Restless leg syndrome

8. Seorang laki-laki 27 tahun datang dengan keluhan rasa tidak nyaman pada anggota gerak bawah
yang muncul ketika pasien duduk, berbaring/diam, memburuk pada malam hari. Pasien tersebut
mengalami gerakan periodik selama tidur. Gerakan pada tungkai dapat mengurangi rasa tidak
nyaman sebagian atau seluruhnya. Terapi yang sesuai dengan keluhan tersebut adalah :
A. Dopamin agonis
B. Levodopa
C. Modafinil 200 mg
D. Metilfenidat 5 mg
E. Amitriptilin 25 mg

9. Seorang wanita usia 50 tahun, ibu rumah tangga, datang ke Poli Saraf RSHAM, dengan keluhan
sejak 2 tahun kesemutan di kedua tangan terutama tangan kanan, jari-jari tangan terutama jari
1 – 4, terasa kebas dan kaku, nyeri (+).Tinel’s sign (+), phalen’s sign (+).
Apa hasil pemeriksaan neurologis lain yang diharapkan?

A. Wartenberg’s sign (+)


B. Flick’s sign (+)
C. Froment’s sign (+)
D. RF Brachioradialis (-)
E. Atrofi otot hipotenar

10. Perempuan 45 tahun dibawa ke IGD dalam kondisi tidak sadar sejak 2 hari, dan semakin
memberat. Pasien juga mengeluhkan nyeri kepala memberat sejak 3 bulan disertai pandangan

196
TO UGM 17 FEBUARI 2022
dobel dan jalan menyeret. Didapatkan riwayat hipertensi. Pemeriksaan neurologis : GCS 10,
pupil bulat anisokor, refleks cahaya negatif, dan hemiparese dekstra. Apakah mekanisme yang
mendasari peningkatan tekanan intrakranial pada kasus di atas?
A. MAP > CPP
B. CPP > MAP
C. Peningkatancomplianceintrakranial
D. Peningkatan elastance intrakranial
E. Gangguan autoregulasi

11. Laki-laki, 60 tahun datang ke poliklinik saraf dengan keluhan kelemahan pada jari-jari kaki kanan
dan kiri disertai rasa nyeri seperti ditusuk dan kesemutan sejak 6 bulan yang lalu. Pada saat
pemeriksaan didapatkan hilangnya sensasi getar dan penurunan reflex fisiologis pada extremitas
bawah. Pada pemeriksaan penunjang diperoleh kadar gula darah 450mg/dl dan hasil
pemeriksaan ENMG didapatkan pemanjangan NCV. Gejala maupun hasil pemeriksaan manakah
yang menunjukkan gejala small fibre neuropathy? The attacks usually consist of pain described
as stabbing or burning, or abnormal skin sensations such as tingling or itchiness
A. Nyeri seperti ditusuk
B. Hilangnyasensasigetar
C. Kelemahan
D. Penurunan reflex
E. Pemanjangan NCV

12. Seorang laki-laki usia 28 tahun, datang ke Poli Saraf RSHAM, dengan keluhan kelemahan
ekstensi pergelangan tangan dan jari-jari tangan kanan, serta ekstensi dan fleksi pada siku.
Sebelumnya pasien tertidur dengan posisi kepalanya menekan lengan atas kanan. Pada status
neurologis didapatkan hipestesi pada dorsal tangan dan pergelangan tangan, refleks
brachioradialis (-).
Apa hasil EMG yang diharapkan?

A. EMG pada M. Extensor Digitorum Brevis: lesi neurogenik dengan tanda denervasi
B. EMG pada M. Abductor Pollicis Brevis: lesi neurogenik dengan tanda denervasi
C. EMG pada M. Interosseus Dorsalis I: lesi neurogenik dengan tanda denervasi
D. EMG pada M. Biceps: lesi neurogenik dengan tanda denervasi
E. EMG pada M. Deltoid: lesi neurogenik dengan tanda denervasi

13. Laki-laki,12 tahun, dikonsulkan dari bagian anestesi, dengan diagnosis cedera kepala berat,
setelah dirawat selama 5 hari diminta untuk melakukan tes mati batang otak (MBO). GCS 3,Tensi
130/80 mmHg, N: 88 x/menit, t: 36oC, RR: 24 x/menit on ventilator. Laboratorium dalam batas
normal. Hasil tes batang otak : reflek pupil (-), reflek kornea (-), tes kalorik (-), gag reflek (-),
reflek primitive (-). Tes apneu:hasil AGD (analisis gas darah) ulang, PaCO2 naik 10 mmHg
dibandingkandengan AGD awal. Langkah apakah yang paling tepat dilakukan selanjutnya?
A. Tes AGD ulangsetelah 10 menit
B. Tes AGD ulangsetelah 30 menit
C. Tes ulang seluruh prosedur MBO dalam waktu 12 jam
D. Tes ulang seluruh prosedurMBO dalam waktu 24 jam
E. Tes ulang seluruh prosedur MBO dalam waktu 48 jam

197
TO UGM 17 FEBUARI 2022
14. Laki-laki, 60 tahun datang dengan keluhan sulit menelan mendadak, disertai pelo. Pemeriksaan
fisik = ipsilateral gangguan cerebellar, Horner’s syndrome, gangguan nyeri menyilang antara
wajah dan badan (face-body dissociation). Arteri manakah yang mengalami gangguan pada
pasien tersebut ?
A. Arteri cerebellar anterior inferior
B. Arteri cerebellar superior
C. Arteriponsparamedian
D. Arteri cerebellar posterior inferior; wallenberg
E. Arterivertebro-basiler

15. Wanita 57 tahun, kelemahan 2 tungkai,sejak 1 tahun yg lalu. tonus spastik, fasikulasi (+). respon
plantar ekstensor +. hiperrefleksia. maka lesi pada:
A. dorsal spinal root
B. ventral spinal root
C. fasikulus arcuatus
D. motor neuron
E. purkinje

16. Perempuan 45 tahun, dg gangguan visual, diabetes insipidus & gangguan tumbuh kembang.
Diagnosis px tersebut:

A. Adenoma pituitari
B. Craniopharyngioma
C. Meningioma
D. Meduloblastoma
E. Retinoblastoma

198
TO UGM 17 FEBUARI 2022
17. Laki2, 8tahun, nyeri kepala, muntah 1 bulan memberat 1 minggu. dari hasil pemeriksaan fisik
ditemukan gerak leher terbatas, ataxia (+), nistagmus (+). Diagnosis px ini
A. Craniopharyngioma
B. Meduloblastoma
C. Glioma optik
D. Meningioma
E. Astrositoma lobus frontal

18. Penderita wanita 33 tahun dengan keluhan hoyong saat berdiri. Saat duduk, berbaring tidak ada
keluhan, lain-lain dalam batas normal. Keluhan diatas dapat disebabkan oleh karena :
A. Disequilibrium
B. Vertigo non vestibuler
C. Vertigo perifer
D. Vertigo central
E. Syncope

19. Seorang pria berusia 55 tahun merasa seperti panas terbakar yang menetap kadang ada rasa
menusuk pada tubuh dan anggota gerak sebelah kanan sejak 3 minggu yang lalu. Keluhan
bertambah jika diraba. Dua bulan yang lalu pasien mengalami kelumpuhan tubuh sebelah kanan
secara tiba-tiba sehingga dirawat. Sekarang telah terdapat perbaikan. Pada berkas yang dibawa,
kesimpulan CT Scan Kepala : Infark serebri kortikal-subkortikal di daerah A. serebri media kiri.
Apakah obat pilihan yang paling tepat untuk keluhan nyeri pada kasus ini?
A. Analgetik asetaminofen+aspirin
B. Analgetik meloksikam+aspirin
C. Antidepressan trisiklik amitriptilin
D. Antikonvulsan pregabalin =neuropati sentral; 150 mg 2x1
E. Kodein

20. Anak laki-laki 8 tahun dibawa ke poliklinik saraf karena diduga gangguan tidur. Pada saat tidur,
pasien terlihat adanya gerakan-gerakan pada bibir kanan yang diikuti suara merintih seperti
berkumur-kumur dan salivasi. Durasi sekitar 1 menit. Kejadian sudah 2 kali dalam 6 bulan ini dan
selalu terjadi saat tidur. Pasien dilakukan pemeriksaan EEG.
Apakah gambaran EEG yang paling sesuai dengan kasus di atas ?

A. Hypsarrhytmia
B. Gelombang paku daerah sentro-temporal dengan tangential dipole
C. Gelombangpolyspike10-16Hz
D. Gelombang paku ombak 3 Hz generalisata
E. Slow spike wave complex (SSWC) generalisata dengan irama dasar lambat

21. Seorang pasien berusia 41 tahun mengeluhkan tangannya gemetaran sejak 3 tahun terakhir.
Awalnya muncul di tangan kanan, namun dalam 6 bulan terakhir gejala memberat dan muncul
di kedua tangan pasien. Gejala ini juga memberat bila pasien sedang stress, minum kopi, dan
berpuasa. Ayah pasien juga mengeluhkan hal yang sama. Pada pemeriksaan fisik ditemukan

199
TO UGM 17 FEBUARI 2022
rigiditas di tangan kanan. Apakah tatalaksana medikamentosa lini pertama yang paling tepat
untuk pasien ini?
A. Propanolol
B. Pramipeksol
C. Trihexyphenidil
D. Clonazepam
E. Clozapin

22. Seorang laki-laki usia 45 tahun, guru SD, datang ke dokter umum dengan keluhan susah untuk
tidur di malam hari. Keluhan ini sudah dirasakan sejak 1 tahun lalu. Ia mengalami kesulitan
untuk memulai tidur, dimana sekitar 1 jam kemudian baru tertidur. Selain itu ia juga mudah
terbangun dari tidur akibat suara, dan lama tidurnya kurang dari 8 jam. Keluhan ini sudah
mengganggu aktivitas, dimana ia menjadi sangat mengantuk di siang hari. Ia memiliki kebiasaan
minum kopi pagi dan siang hari.
Apakah anjuran yang diberikan untuk pasien ini?

A. Buat suasana lingkungan tidur yang nyaman


B. Hindariminumkopisebelumtidur
C. Hindaristressakibatkerja
D. Olahraga teratur
E. Dietteratur

23. Seorang pria berusia 55 tahun, datang ke UGD dengan keluhan nyeri kepala secara tiba-tiba
sekitar 2 jam sebelumnya. Dijumpai tekanan darah 200/100 mmHg. Pada pemeriksaan
funduskopi dijumpai gambaran fundus hipertensif. Pemeriksaan motorik dan kranialis lainnya
normal.
Gambaran apakah yang ditunjukkan pada funduskopi ini

A. Mikroaneurisma
B. Perdarahan retina
C. Cotton wool spots
D. Hard exudates
E. Macular star

24. Pasien perempuan 28 tahun datang dengan sakit kepala sejak 6jam yang lalu. Sakit kepala
dirasakan di sisi kanan, berdenyut dan hebat hingga mengganggu aktivitas. Sebelum serangan
pasien melihat kilatan cahaya. Sakit kepala seperti ini cukup sering, hampir tiap 1-2 bulan per

200
TO UGM 17 FEBUARI 2022
kali, terutama jika mengkonsumsi junk food. Biasanya sakit kepala sekitar setengah hari dan
kurang respon dengan parasetamol. Riwayat hipertensi, DM, asma disangkal. Pemeriksaan fisik
dalam batas normal.
A. Apakah terapi farmakologis yang paling tepat diberikan untuk pasien ini?
B. Terapi abortif dengan sumatriptan 6 mg PO dan profilaksis dengan topiramat 25 mg PO
C. Terapi abortif dengan sumatriptan 50 mg PO dan profilaksis dengan topiramat 25 mg PO
D. Terapi abortif saja dengan sumatriptan 6 mg PO
E. Terapi abortif saja dengan sumatriptan 50 mg PO

25. Pasien laki-laki 30 tahun datang ke poliklinik saraf dengan keluhan nyeri kepala sejak 1 bulan
yang lalu. Nyeri terutama sekitar mata dan pelipis kanan, tidak berdenyut namun intensitas
hebat. Nyeri berlangsung sekitar 30 menit dan dalam sehari dapat hingga 5 kali. Nyeri disertai
kelopak mata kanan turun, mata kanan merah dan lakrimasi serta hidung tersumbat. Nyeri
seperti ini sudah pernah terjadi dan sudah bebas serangan selama 3 minggu.
Apakah terapi profilaksis yang tepat untuk kasus tersebut?

A. Prednison
B. Verapamil
C. asamvalproat
D. indometasin
E. topiramat

26. Anak laki-laki 8 tahun diduga gangguan tidur. Pada saat tidur, pasien terlihat gerakan gerakan
pada bibir kanan diikuti suara merintih seperti berkumur-kumur dan salivasi. Durasi 1 menit.
Kejadian sudah 2 kali dalam 6 bulan dan selalu terjadi saat tidur. sindrom
epilepsi ?

A. Lennox-Gestaut
B. Rolandic
C. JME
D. Landau kleffner

27. Seorang pasien berusia 41 tahun mengeluhkan tangannya gemetaran sejak 3 tahun terakhir.
Awalnya muncul di tangan kanan, namun dalam 6 bulan terakhir gejala memberat dan muncul
di kedua tangan pasien. Gejala ini juga diikuti dengan kepala bergerak-gerak. Ayah pasien juga
mengeluhkan hal yang sama. Apa terapi yang tepat pada pasien tersebut:
Propanolol dan pirimidone

201
TO UGM 17 FEBUARI 2022
28. Lokasi kelainan :

A. Basilar tip
B. Serebriposterior
C. Comunicansposterior
D. Comunicans anterior

29. Gambar DSA : AVM. Yang mempengaruhi tatalaksana terhadap kelainan tersebut:
A. Ukuran, lokasi, drainage vena
B. Feeding artery, ukuran, lokasi
C. Feeding artery, lebar leher, jumlah perdarahan

30. Seorang laki-laki umur 44 tahun mengalami penurunan kesadaran secara tiba-tiba sejak 24 jam
yang lalu. Pada pemeriksaan fisik saat awal pasien masuk rumah sakit didapatkan tekanan darah
170/100 mmHg dengan GCS E3 M5 V4. Setelah 24 jam kesadaran pasien menjadi GCS E2 M4 V3.
Pada pasien ini dilakukan pemeriksaan brain MRI dan MRA dan didapatkan gambaran seperti di
bawah ini. Penatalaksanaan yang paling tepat pada pasien ini adalah:

A. Pemberian double antiplatelet


B. Pemberian obat antikejang
C. Pemberian obat neuroprotektan
D. Carotid endarterectomy

202
TO UGM 17 FEBUARI 2022
E. Kraniotomi dekompresi

31. Perempuan 45 tahun, datang ke UGD dengan kesadaran menurun sejak 72 jam yang lalu secara
mendadak, Riwayat nyeri kepala hebat (+), TD : 200/120 RR: 24x/mnt, t : 37 ‘C. Selang 30 menit
OS tampaklebih tenang tapi tidak responsif dan kesadaran pasien menurun secara tajam.
Pemeriksaan kaku kuduk (+) dan refleks babinski (+). Untuk mengetahui prognosis maka dapat
digunakan :
A. Hunt and Hess
B. Fisher
C. World Federation of Neurosurgeons
D. Skor ICH
E. GCS

32. Seorang wanita usia 34 tahun datang dengan keluhan gerakan berkedut-kedut tanpa disadari
oleh pasien secara tiba-tiba pada daerah wajah dan sekitar mulut. Gerakan semakin bertambah
bila pasien mengalami kelelahan dan stress. Dijumpai juga gangguan tingkah laku pada pasien
ini. Berdasarkan skenario kasus diatas, diagnosis apakah yang tepat untuk kasus tersebut ?
A. Chorea Sydenham
B. Sindroma Tourette
C. Dystonia
D. Athetosis
E. Dyskinesia

33. Wanita 35 tahun datang dengan keluhan nyeri kepala disertai mata berair dan kemerahan
selama 10-15 detik. Diagnosis pada pasien ini yang paling memungkinkan, adalah
A. Sindroma Tolosa-Hunt
B. Hemikrania paroksismal
C. SUNCT
D. Migrain oftalmoplegik
E. Nyeri kepala klaster

34. Seorang laki-laki 51 tahun sering mengeluh pusing sering mengeluh pusing saat berada di
keramaian. Keluhan dirasakan terus-menerus dan apabila berjalan seperti akan jatuh. Tidak ada
mual muntah maupun gangguan pendengaran. Dari pemeriksaan penunjang didapatkan GDS
234 mg/dL. Diagnosis banding yang paling mungkin adalah :
A. Migraine associated dizziness
B. Iskemia vertebrobasiler
C. Vertigo non vestibuler
D. Neuritis vestibularis
E. Vestibuler paroxysmia

35. Wanita, 45 tahun, nyeri kepala hebat, disertai nyeri tengkuk & muntah. TD 160/90. Pemeriksaan
untuk menilai vasospasme adalah:
A. Carotid doppler

203
TO UGM 17 FEBUARI 2022
B. TCD
C. EEG
D. CT scan
E. Brain MRI

36. pada pasien SAH, mencegah rebleeding dapat diberikan tx apa? Injtranexamid acid
37. Terapi untuk gangguan eksekutif: Neurolinguistik, neurocognitive, rehabilitasi, okupasi
38. Kelainan apa pada gangguan ekekutif: nama benda,nama tempat, arti peribahasa
39. Tujuan pemeriksaan tcd pada psa
40. OAE yang dapat memperburuk bangkitan Lena: oxcarbazepine
41. Soal infeksi, perdarahan pada lobus temporal: HSV, varicella zooster
42. Anak suka ikuti gerakan orang lain
43. Ring enhancement pada fossa posterior: abses Cerebri
44. Lesi bulat gray and white matter: metastasis
45. Anak ada chorioretinitis: toxoplasmosis
46. Malaria cerebral, ada riwayat ke Papua, apa terapinya: artemisin
47. Pusing selama 1 menit, Terapi vertigo paroxysmia: carbamazepine 200-600 mg
48. Anak2 nyeri kepala, mual,muntah, nyeri abdomen: migrain abdominal
49. Terapi abortif migrain: lupaaa
50. Ada soal tentang medication overused kalo nda salah
51. 45 th, ada iskemik dan stenosis 50%: apa terapinya
52. Buta pada kedua mata, Dimana lesinya
53. Yang benar tentang hipertensi intrakranial Idiopatik
54. Kasus PIS, dimna letak lesi: arteri lentikulostriata jawabanku
55. Letak lesi penyebab pandangan ganda: n.VI kanan
56. Trauma medula spinalis onset <2 jam: sy kasi metilprednisolone
57. Nystagmus kanan komponen cepat ke kanan; apa diagnosanya
58. Ada demensia lewy body kyknya
59. Nyeri pada tangan pasien IMA: reffred pain

Yang belum

1. Remed okto 2020/2021

204
TO UGM 17 FEBUARI 2022
205
TO UGM 17 FEBUARI 2022

Anda mungkin juga menyukai